Chapter: General Physiology; Topic: Nerve-Muscle Physiology; Subtopic: Golgi Tendon Organ and Inverse Stretch Reflex
Key Definitions & Concepts
Inverse Stretch Reflex: Also known as the Golgi Tendon Reflex; a protective mechanism where excessive tension causes muscle relaxation (Autogenic Inhibition).
Golgi Tendon Organ (GTO): The sensory receptor for this reflex, located "in series" with muscle fibers in the tendon; monitors muscle tension.
Ib Afferent Fibers: Fast-conducting myelinated sensory fibers that carry signals from the GTO to the spinal cord.
Bisynaptic (Disynaptic) Reflex: A reflex arc involving two synapses and one interneuron interposed between the sensory and motor neurons.
Ib Inhibitory Interneuron: Located in the spinal cord; receives input from Ib afferents and releases Glycine to inhibit the alpha motor neuron.
Autogenic Inhibition: The physiological result of the reflex: inhibition (relaxation) of the same muscle that generated the tension.
Reciprocal Excitation: Simultaneous contraction of the antagonist muscle during the inverse stretch reflex to stabilize the joint.
Clasp-Knife Phenomenon: A clinical sign of UMN lesions (spasticity) where resistance to stretch suddenly collapses; mediated by the inverse stretch reflex.
Monosynaptic Reflex: A reflex with only one synapse (no interneuron), such as the Stretch Reflex (Deep Tendon Reflex).
Adequate Stimulus: For GTOs, the most effective stimulus is active muscle contraction (which pulls the tendon), rather than passive stretch.
[Image of Reflex arc diagram stretch reflex]
Lead Question - 2016
Inverse stretch reflex is a?
a) Monosynaptic reflex
b) Bisynaptic reflex
c) Polysynaptic reflex
d) Nonsynaptic reflex
Explanation: The Inverse Stretch Reflex (Golgi Tendon Reflex) is mediated by the Golgi Tendon Organ. The reflex arc consists of the following steps: 1) Ib afferent fibers enter the spinal cord. 2) They synapse on an Inhibitory Interneuron (Synapse #1). 3) The interneuron synapses on the Alpha Motor Neuron (Synapse #2). 4) The motor neuron is inhibited, causing muscle relaxation. Because there is exactly one interneuron interposed between the afferent and efferent limbs, there are Two Synapses in the pathway. Therefore, it is anatomically classified as a Bisynaptic (or Disynaptic) reflex. The stretch reflex is monosynaptic. The withdrawal reflex is polysynaptic. Therefore, the correct answer is b) Bisynaptic reflex.
1. The physiological term "Autogenic Inhibition" describes the outcome of the Inverse Stretch Reflex. This means:
a) Excitation of the antagonist muscle
b) Inhibition of the homonymous (agonist) muscle
c) Inhibition of the contralateral muscle
d) Excitation of the Gamma motor neuron
Explanation: The Golgi Tendon Reflex functions as a tension-feedback system. When a muscle develops excessive tension (force), the GTOs fire. This signal travels via Ib fibers to the spinal cord, where it activates inhibitory interneurons. These interneurons specifically inhibit the alpha motor neurons of the Same muscle (Homonymous muscle) that generated the force. This "self-inhibition" is termed Autogenic Inhibition. Its protective role is to prevent the muscle from tearing away from its insertion due to overload. Therefore, the correct answer is b) Inhibition of the homonymous (agonist) muscle.
2. Which neurotransmitter is responsible for the inhibition of the alpha motor neuron during the Inverse Stretch Reflex?
a) Glutamate
b) Acetylcholine
c) Glycine
d) Substance P
Explanation: The reflex arc involves an inhibitory interneuron (the Ib inhibitory interneuron). While the sensory neuron releases Glutamate (excitatory) to activate the interneuron, the interneuron itself must release an inhibitory transmitter to stop the motor neuron from firing. In the spinal cord, the primary inhibitory neurotransmitter released by interneurons (such as Renshaw cells and Ia/Ib interneurons) is Glycine. Glycine binds to receptors on the alpha motor neuron, opening Chloride channels, causing hyperpolarization (IPSP) and preventing contraction. Therefore, the correct answer is c) Glycine.
3. While the agonist muscle relaxes during the Golgi Tendon Reflex, the antagonist muscle contracts. This phenomenon is known as:
a) Reciprocal Inhibition
b) Reciprocal Excitation
c) Crossed Extensor Reflex
d) Gamma Loop activation
Explanation: Reflexes often coordinate opposing muscle groups. In the Stretch Reflex, the agonist contracts and the antagonist is inhibited (Reciprocal Inhibition). In the Inverse Stretch Reflex (GTO reflex), the opposite occurs: the agonist is inhibited (relaxed) to relieve tension, and the Antagonist muscle is Excited (contracts). This is called Reciprocal Excitation. It helps stabilize the joint and assists in unloading the strained agonist muscle. This requires an excitatory interneuron in the pathway to the antagonist. Therefore, the correct answer is b) Reciprocal Excitation.
4. The Golgi Tendon Organ is particularly sensitive to:
a) Passive stretch of the muscle
b) Active contraction of the muscle
c) Changes in muscle length only
d) Vibration
Explanation: Anatomically, GTOs are arranged "in series" with the muscle fibers. While passive stretch can stimulate them if intense enough, their threshold is extremely high for passive stretch. However, when the muscle fibers contract actively, they pull directly on the tendon, compressing the nerve endings in the GTO. Thus, the GTO has a very low threshold for and is exquisitely sensitive to Active Contraction. It functions as a dynamometer, continuously monitoring muscle force (tension), not just length. Therefore, the correct answer is b) Active contraction of the muscle.
5. Which clinical sign is a manifestation of the Inverse Stretch Reflex in patients with spasticity (UMN lesions)?
a) Cog-wheel rigidity
b) Lead-pipe rigidity
c) Clasp-knife spasticity
d) Pendular knee jerk
Explanation: In spasticity, there is hypertonia (resistance to stretch) due to a hyperactive Stretch Reflex. However, if the clinician continues to stretch the muscle forcefully, the tension in the tendon eventually reaches the threshold of the Golgi Tendon Organs. Once activated, the GTOs trigger the Inverse Stretch Reflex (autogenic inhibition), causing the muscle resistance to suddenly "melt away" or collapse. This sudden release of tone is called the Clasp-knife phenomenon (resembling a pocket knife snapping shut). Therefore, the correct answer is c) Clasp-knife spasticity.
6. The afferent limb of the Inverse Stretch Reflex is formed by which group of nerve fibers?
a) Group Ia
b) Group II
c) Group Ib
d) Group III
Explanation: Sensory fibers from muscle receptors are classified by diameter. Group Ia fibers innervate the Primary Muscle Spindle endings (Length/Velocity). Group II fibers innervate Secondary Spindle endings (Static Length). Group Ib fibers innervate the Golgi Tendon Organs (Tension). Both Ia and Ib fibers are large, myelinated A-alpha type fibers with fast conduction velocities (70-120 m/s), ensuring rapid reflex responses. Therefore, the correct answer is c) Group Ib.
7. Tetanus toxin (Tetany) produces muscle rigidity and spasms by specifically blocking:
a) Acetylcholine release at the neuromuscular junction
b) Glutamate release from afferent fibers
c) Glycine release from inhibitory interneurons
d) Calcium uptake in the Sarcoplasmic Reticulum
Explanation: Tetanospasmin is a neurotoxin that travels retrogradely to the spinal cord. It targets the inhibitory interneurons (Renshaw cells and Ib inhibitory interneurons). The toxin cleaves synaptobrevin, preventing the exocytosis of inhibitory neurotransmitters (GABA and Glycine). By blocking the release of Glycine, the alpha motor neurons are "disinhibited" (lose their braking mechanism). They fire uncontrollably, leading to sustained muscle contraction (tetanus) and spasms. This effectively disables the Inverse Stretch Reflex inhibition. Therefore, the correct answer is c) Glycine release from inhibitory interneurons.
8. How many neurons are involved in the simplest pathway of the Inverse Stretch Reflex arc?
a) 2 neurons
b) 3 neurons
c) 4 neurons
d) Many neurons
Explanation: To count the neurons in a reflex arc: 1. Afferent Neuron: The Ib sensory fiber bringing the signal. 2. Interneuron: The Ib inhibitory interneuron in the spinal cord. 3. Efferent Neuron: The Alpha motor neuron executing the command. Since there are three distinct neurons involved in the chain, it is a 3-neuron arc. This corresponds to a bisynaptic (2 synapse) reflex. The stretch reflex involves only 2 neurons (afferent + efferent). Therefore, the correct answer is b) 3 neurons.
9. The Golgi Tendon Reflex is an example of a:
a) Positive feedback mechanism
b) Negative feedback mechanism
c) Feed-forward mechanism
d) Learned reflex
Explanation: Homeostatic reflexes usually function as negative feedback loops to maintain stability. In the GTO reflex: Input: Increased Muscle Tension. Response: Muscle Relaxation (decreased tension). Because the response (relaxation) opposes and reduces the original stimulus (tension), it is a classic Negative Feedback mechanism. It acts as a servo-control system to keep muscle tension within safe physiological limits and smoothen contraction force. Positive feedback would increase tension further (vicious cycle). Therefore, the correct answer is b) Negative feedback mechanism.
10. Unlike the Golgi Tendon Organ, the Muscle Spindle is arranged:
a) In series with extrafusal fibers
b) In parallel with extrafusal fibers
c) Within the tendon
d) Outside the muscle capsule
Explanation: The anatomical arrangement is crucial for function. GTOs are "In Series" (at the end) to detect tension. Muscle Spindles are embedded within the muscle belly and attach to the endomysium of adjacent fibers. This places them In Parallel with the main force-generating (extrafusal) fibers. This parallel arrangement means that when the muscle stretches, the spindle stretches (detecting length). When the muscle contracts (shortens), the spindle unloads (shortens). Therefore, the correct answer is b) In parallel with extrafusal fibers.
Chapter: General Physiology; Topic: Nerve-Muscle Physiology; Subtopic: Golgi Tendon Organ and Inverse Stretch Reflex
Key Definitions & Concepts
Golgi Tendon Organ (GTO): A specialized sensory receptor located at the junction between the muscle fibers and the tendon (musculotendinous junction); it is arranged "in series" with the muscle fibers.
Inverse Stretch Reflex: A protective reflex where excessive tension on a muscle causes it to relax (Autogenic Inhibition), preventing structural damage.
Ib Afferent Fibers: Large, fast-conducting myelinated sensory fibers that transmit tension signals from the GTO to the spinal cord.
Disynaptic Reflex: A reflex arc that involves two synapses and one interneuron interposed between the sensory and motor neurons.
Ib Inhibitory Interneuron: A specific interneuron in the spinal cord that releases inhibitory neurotransmitters (Glycine) to inhibit the alpha motor neuron of the agonist muscle.
Autogenic Inhibition: The physiological process where the muscle generating the force is inhibited (relaxed) by its own sensory feedback (from GTOs).
Clasp-Knife Phenomenon: A clinical sign of spasticity (UMN lesion) where resistance to passive stretch suddenly gives way; mediated by the overactive Golgi tendon reflex.
"In Series" Arrangement: GTOs are aligned in line with the force vector, making them sensitive to the force (tension) generated by muscle contraction.
Dynamic vs. Static Response: Like spindles, GTOs have a dynamic response (reacting to sudden increases in tension) and a static response (steady-state tension).
Glycine: The primary inhibitory neurotransmitter released by the spinal interneurons in the GTO reflex arc.
[Image of Reflex arc diagram stretch reflex]
Lead Question - 2016
Spinal cord has how many synapses in golgi tendon reflex?
a) 1
b) 2
c) 3
d) 4
Explanation: The Golgi tendon reflex (Inverse Stretch Reflex) is an inhibitory reflex mediated by the Golgi Tendon Organ. The neuronal pathway involves: 1) The afferent Ib sensory fiber enters the spinal cord. 2) The Ib fiber synapses on an Inhibitory Interneuron (Synapse #1). 3) The Inhibitory Interneuron synapses on the Alpha Motor Neuron (Synapse #2). Because there is an interneuron inserted between the afferent and efferent limbs, the circuit contains Two Synapses within the spinal cord. This classifies it as a Disynaptic Reflex. In contrast, the Stretch Reflex is monosynaptic (1 synapse), and the Withdrawal Reflex is polysynaptic (>2 synapses). Therefore, the correct answer is b) 2.
1. Unlike the muscle spindle, which is arranged in parallel, the Golgi Tendon Organ is arranged "in series" with the extrafusal fibers. This anatomical arrangement makes the GTO predominantly sensitive to:
a) Muscle Length
b) Velocity of stretch
c) Muscle Tension (Force)
d) Joint angle position
Explanation: The physical arrangement of a receptor dictates its adequate stimulus. Muscle spindles are "in parallel" and are stretched when the muscle lengthens (detecting length). Golgi Tendon Organs are located in the tendon, "in series" with the muscle fibers. When the muscle contracts, it pulls on the tendon, creating tension. Even isometric contraction (where length doesn't change but tension increases) stimulates the GTO. Therefore, the GTO functions as a Force or Tension monitor. It protects the muscle from tearing by sensing excessive force. Therefore, the correct answer is c) Muscle Tension (Force).
2. A patient with a stroke exhibits spasticity. When the physician passively stretches the patient's arm, there is initial resistance followed by a sudden release ("melting away") of tone. This "Clasp-Knife" reflex is mediated by the:
a) Muscle Spindle (Ia fibers)
b) Golgi Tendon Organ (Ib fibers)
c) Pain receptors (Group III fibers)
d) Joint capsule receptors
Explanation: The Clasp-Knife phenomenon has two phases. The initial resistance is due to the hyperactive Stretch Reflex (spasticity) mediated by Muscle Spindles. However, as tension builds up in the tendon during the stretch, the high threshold Golgi Tendon Organs are eventually activated. The GTO signals (via Ib fibers) trigger the Inverse Stretch Reflex, causing Autogenic Inhibition (sudden relaxation) of the muscle. This sudden loss of resistance is the "blade closing" effect. Therefore, the release phase is mediated by the GTO. Therefore, the correct answer is b) Golgi Tendon Organ (Ib fibers).
3. The inhibitory interneuron in the Golgi tendon reflex arc releases which neurotransmitter onto the alpha motor neuron?
a) Glutamate
b) Acetylcholine
c) Glycine
d) Substance P
Explanation: The afferent Ib fiber releases Glutamate (excitatory) to activate the interneuron. However, the function of this reflex is to relax the muscle. Therefore, the interneuron itself must be inhibitory. The primary inhibitory neurotransmitter released by interneurons in the spinal cord (such as Renshaw cells and Ia/Ib inhibitory interneurons) is Glycine. Glycine opens chloride channels on the alpha motor neuron, hyperpolarizing it (IPSP) and preventing contraction. Strychnine poisoning blocks these glycine receptors, leading to convulsions. Therefore, the correct answer is c) Glycine.
4. While the Stretch Reflex causes contraction of the agonist muscle, the Golgi Tendon Reflex causes:
a) Contraction of the agonist muscle
b) Relaxation of the agonist muscle (Autogenic Inhibition)
c) Contraction of the antagonist muscle (Reciprocal Excitation)
d) Both b and c
Explanation: Reflexes coordinate agonist/antagonist pairs. 1. Stretch Reflex: Excites agonist (contraction), inhibits antagonist (Reciprocal Inhibition). 2. Golgi Tendon Reflex: Operates in reverse. It senses too much tension. To relieve tension, it inhibits the agonist motor neuron, causing Relaxation (Autogenic Inhibition). Simultaneously, through excitatory interneurons, it stimulates the antagonist muscle to contract (Reciprocal Excitation). This dual action unloads the strained muscle and stabilizes the joint. Therefore, the correct answer is d) Both b and c.
5. Tetanus toxin causes severe muscle spasms and rigidity. The mechanism involves the blockade of neurotransmitter release from which cell type involved in the tendon reflex?
a) Alpha motor neurons
b) Ib Afferent sensory neurons
c) Inhibitory Interneurons (Renshaw / Ib interneurons)
d) Gamma motor neurons
Explanation: Tetanospasmin (Tetanus toxin) is a neurotoxin that undergoes retrograde transport to the spinal cord. It specifically cleaves synaptobrevin in inhibitory nerve terminals, preventing the release of inhibitory neurotransmitters (Glycine and GABA). This affects Renshaw cells and the Ib Inhibitory Interneurons of the Golgi tendon reflex. Without this essential inhibition ("disinhibition"), the motor neurons fire uncontrollably, resulting in spastic paralysis, lockjaw, and opisthotonus. The protective "brake" of the GTO reflex is disabled. Therefore, the correct answer is c) Inhibitory Interneurons (Renshaw / Ib interneurons).
6. Which of the following nerve fibers carries the afferent signal from the Golgi Tendon Organ?
a) Type Ia
b) Type Ib
c) Type II
d) Type III
Explanation: Sensory fibers from muscle receptors are classified by diameter (Lloyd-Hunt). Group Ia: Primary muscle spindle ending (Length/Velocity). Group Ib: Golgi Tendon Organ (Tension). Group II: Secondary muscle spindle ending (Static length). Group III/IV: Free nerve endings (Pain/Chemicals). Both Ia and Ib fibers fall under the A-alpha category in the Erlanger-Gasser system (fastest conduction, 70-120 m/s), ensuring rapid reflex adjustments. Therefore, the correct answer is b) Type Ib.
7. The Jendrassik maneuver (reinforcement) enhances deep tendon reflexes. Physiologically, this maneuver affects the reflex arc by:
a) Inhibiting the Golgi Tendon Organs
b) Directly depolarizing alpha motor neurons
c) Increasing Gamma motor neuron drive to muscle spindles
d) Increasing the release of Glycine
Explanation: While this question relates to the stretch reflex, it is important to contrast it with GTO function. Jendrassik's maneuver (clenching teeth/pulling hands) increases the central excitatory drive. This descends to the spinal cord and activates Gamma motor neurons. High gamma activity pre-stretches the muscle spindles, sensitizing them to the tap. It does not primarily involve the GTO pathway. In fact, GTO activation would inhibit the reflex. Understanding the distinction (Sensitizing Spindle vs. Inhibiting GTO) is key to reflex physiology. Therefore, the correct answer is c) Increasing Gamma motor neuron drive to muscle spindles.
8. In the classification of reflexes based on the number of synapses, the Flexor Withdrawal Reflex (e.g., stepping on a tack) is classified as:
a) Monosynaptic
b) Disynaptic
c) Polysynaptic
d) Asynaptic
Explanation: 1. Stretch Reflex: Monosynaptic (1 synapse). 2. Golgi Tendon Reflex: Disynaptic (2 synapses). 3. Flexor Withdrawal Reflex: This involves cutaneous pain receptors (A-delta/C fibers). The signal travels through multiple interneurons (excitatory and inhibitory) across several spinal segments to coordinate the contraction of flexors and relaxation of extensors in the affected limb (and often the opposite limb via the Crossed Extensor Reflex). Since it involves a chain of interneurons (>2 synapses), it is Polysynaptic. Therefore, the correct answer is c) Polysynaptic.
9. The "Adequate Stimulus" for the Golgi Tendon Organ is:
a) Passive stretch of the muscle
b) Vibration
c) Active contraction of the muscle
d) Muscle fatigue
Explanation: GTOs have a high threshold for passive stretch; you have to stretch a muscle very hard to activate them passively. However, they have a very low threshold for Active Contraction. When a muscle contracts, the force is transmitted directly through the tendon, squeezing the GTO nerve endings interlaced with the collagen fibers. Thus, GTOs are exquisitely sensitive monitors of active force generation, providing minute-to-minute feedback to the CNS to regulate muscle tension. Therefore, the correct answer is c) Active contraction of the muscle.
10. A laboratory experiment demonstrates that stimulating a specific nerve fiber causes an Inhibitory Postsynaptic Potential (IPSP) in the alpha motor neuron. The fiber stimulated was most likely a:
a) Group Ia afferent
b) Group Ib afferent
c) Alpha motor neuron axon
d) Gamma motor neuron axon
Explanation: Group Ia stimulation -> EPSP in agonist motor neuron (Stretch Reflex). Group Ib stimulation -> IPSP in agonist motor neuron (Inverse Stretch Reflex). This inhibition is not direct; the Ib fiber excites an interneuron, and the *interneuron* causes the IPSP. However, the net effect of stimulating the Ib afferent pathway is inhibition of the homonymous motor neuron. This physiological signature distinguishes tendon organ afferents from spindle afferents. Therefore, the correct answer is b) Group Ib afferent.
Chapter: General Physiology; Topic: Nerve-Muscle Physiology; Subtopic: The Stretch Reflex and Fiber Types
Key Definitions & Concepts
Stretch Reflex (Myotatic Reflex): A monosynaptic reflex arc primarily responsible for maintaining muscle tone and resisting passive stretch.
Type Ia Fibers (Primary Afferents): Large, myelinated sensory fibers innervating the primary (annulospiral) endings of the muscle spindle. They detect the rate of change in muscle length (velocity).
Type II Fibers (Secondary Afferents): Medium, myelinated sensory fibers innervating the secondary (flower-spray) endings. They detect the static length of the muscle.
Erlanger-Gasser Classification: Classifies fibers as A, B, C.
A-alpha (Aα): Corresponds to Group Ia and Ib (proprioception).
A-beta (Aβ): Corresponds to Group II (touch/pressure/secondary spindle).
A-gamma (Aγ): Motor to muscle spindle (fusimotor).
Group Ia: Equivalent to A-alpha sensory fibers; the fastest conducting sensory fibers in the body (70-120 m/s).
Monosynaptic Connection: The Ia afferent synapses directly onto the Alpha Motor Neuron in the spinal cord without an interneuron.
Muscle Spindle: The receptor organ for the stretch reflex, located in parallel with muscle fibers.
Alpha Motor Neuron: The efferent limb of the reflex arc, causing contraction of the extrafusal muscle fibers.
Gamma Loop: The mechanism where gamma motor neurons stretch the spindle to activate Ia afferents, indirectly causing muscle contraction.
Dynamic Response: The burst of firing from Ia fibers during the actual stretching movement (phasic stretch reflex).
[Image of Reflex arc diagram stretch reflex]
Lead Question - 2016
Afferents for stretch reflexes are carried by which fibers?
a) A alpha (Aa)
b) A gamma (Ay)
c) Type B
d) Type C
Explanation: The stretch reflex is initiated by the muscle spindle. The primary sensory endings of the muscle spindle are innervated by Group Ia fibers. In the Erlanger-Gasser classification (Letter system), Group Ia fibers correspond to the A-alpha (Aα) sensory fibers. These are the largest and fastest conducting sensory fibers, essential for the rapid monosynaptic response required for posture and reflexes. Type II fibers (A-beta) also contribute to the static component, but the primary high-velocity signal comes from Ia (A-alpha). A-gamma fibers are motor to the spindle. Type B are autonomic. Type C are pain. Therefore, the correct answer is a) A alpha (Aa).
1. Which specific component of the muscle spindle is innervated by the Group Ia afferent fiber?
a) The ends of the nuclear chain fibers only
b) The center of the nuclear bag and nuclear chain fibers (Primary ending)
c) The Golgi tendon organ
d) The flower-spray ending only
Explanation: The muscle spindle contains intrafusal fibers (Nuclear Bag and Nuclear Chain). The sensory innervation is organized into primary and secondary endings. The Group Ia afferent wraps spirally around the equatorial (center) region of both the Nuclear Bag and Nuclear Chain fibers. This structure is called the Primary (Annulospiral) ending. Because it innervates the dynamic nuclear bag fibers, the Ia fiber is uniquely sensitive to the velocity (rate of change) of stretch. The secondary (flower-spray) ending is innervated by Group II fibers. Therefore, the correct answer is b) The center of the nuclear bag and nuclear chain fibers (Primary ending).
2. The conduction velocity of the afferent fibers mediating the knee-jerk reflex is approximately:
a) 0.5 - 2 m/s
b) 3 - 15 m/s
c) 30 - 70 m/s
d) 70 - 120 m/s
Explanation: The knee-jerk reflex is a phasic stretch reflex mediated by Group Ia afferents. Group Ia fibers are equivalent to A-alpha sensory fibers. These are the largest diameter (12-20 µm) and most heavily myelinated fibers in the peripheral nervous system. Consequently, they conduct impulses at the highest physiological speeds, ranging from 70 to 120 meters per second. This extreme speed minimizes the latency of the reflex arc, allowing for immediate postural corrections. C fibers conduct at ~1 m/s. Therefore, the correct answer is d) 70 - 120 m/s.
3. While the A-alpha afferent carries the signal to the spinal cord, the efferent (motor) signal that causes the muscle to contract travels via:
a) A-gamma motor neurons
b) A-alpha motor neurons
c) B fibers
d) C fibers
Explanation: The stretch reflex arc has two main neural components. The afferent limb is the Ia sensory fiber (A-alpha sensory). This fiber synapses in the anterior horn. The efferent limb, which carries the command back to the main muscle mass (extrafusal fibers) to cause contraction, is the Alpha Motor Neuron (A-alpha motor). Thus, both the sensory input and the motor output of the stretch reflex utilize the fastest fiber type (A-alpha) to ensure speed. Gamma motor neurons innervate the spindle itself, not the main muscle force generators. Therefore, the correct answer is b) A-alpha motor neurons.
4. Group II afferent fibers from the muscle spindle secondary endings correspond to which fiber type in the Erlanger-Gasser classification?
a) A-alpha
b) A-beta
c) A-delta
d) A-gamma
Explanation: Group II fibers innervate the secondary (flower-spray) endings of the muscle spindle, located adjacent to the primary ending. They are predominantly responsible for detecting the static length of the muscle (tone). In the Erlanger-Gasser system, Group II fibers fall into the A-beta (Aβ) category (some sources place them at the A-beta/gamma border, but A-beta is the standard sensory equivalent for mechanoreception/secondary spindle). They are slower (30-70 m/s) than the Group Ia (A-alpha) fibers. Therefore, the correct answer is b) A-beta.
5. The Golgi Tendon Organ (GTO), which mediates the Inverse Stretch Reflex, sends afferent signals via which fiber group?
a) Group Ia
b) Group Ib
c) Group II
d) Group III
Explanation: The Golgi Tendon Organ is a tension receptor located in the tendon. Its afferent fiber is classified as Group Ib. Like Ia fibers, Ib fibers are large and myelinated (A-alpha type), conducting at 70-120 m/s. However, physiologically they are distinct: Ia fibers (spindle) excite the agonist muscle (stretch reflex), whereas Ib fibers (GTO) activate inhibitory interneurons to relax the agonist muscle (autogenic inhibition) to prevent damage from excessive force. Therefore, the correct answer is b) Group Ib.
6. A-gamma (γ) fibers are part of the stretch reflex system, but they are classified as:
a) Sensory Afferents
b) Motor Efferents
c) Autonomic Preganglionic
d) Interneurons
Explanation: It is crucial to distinguish between the fibers leaving the spindle and those going to it. A-alpha (Ia) and A-beta (II) are sensory afferents *leaving* the spindle. A-gamma fibers are Motor Efferents leaving the spinal cord to innervate the contractile poles of the intrafusal fibers *within* the spindle. By contracting these poles, they maintain spindle sensitivity during muscle shortening (Alpha-Gamma Coactivation). They do not carry sensory information themselves. Therefore, the correct answer is b) Motor Efferents.
7. Which fiber type is NOT involved in the monosynaptic stretch reflex arc?
a) A-alpha sensory (Ia)
b) A-alpha motor
c) C fibers
d) All are involved
Explanation: The monosynaptic stretch reflex arc is the simplest reflex. It consists of: 1. Receptor (Spindle). 2. Afferent Neuron: Group Ia (A-alpha sensory). 3. Synapse: Monosynaptic in ventral horn. 4. Efferent Neuron: Alpha Motor Neuron (A-alpha motor). 5. Effector (Muscle). C fibers (unmyelinated, slow pain) are not part of this proprioceptive reflex loop. They are involved in polysynaptic withdrawal reflexes (flexor reflex) in response to pain. Therefore, the correct answer is c) C fibers.
8. The dynamic response of the muscle spindle (detecting velocity) is transmitted exclusively by:
a) Group II fibers
b) Group Ia fibers
c) Group Ib fibers
d) Group III fibers
Explanation: The muscle spindle provides two types of information: Static (length) and Dynamic (rate of change). Group II (secondary) endings are purely static; they fire at a constant rate proportional to length. Group Ia (primary) endings are both static and dynamic. However, they are the exclusive carriers of the Dynamic response. When the muscle is actively being stretched, Ia firing increases dramatically to signal the velocity. Once the length stabilizes, firing decreases to a steady static rate. This velocity sensitivity allows for predictive motor control. Therefore, the correct answer is b) Group Ia fibers.
9. A section of the dorsal root (rhizotomy) abolishes the stretch reflex because it interrupts the:
a) Gamma motor output
b) Alpha motor output
c) Ia afferent input
d) Renshaw cell inhibition
Explanation: The reflex arc requires an intact afferent and efferent limb. The dorsal root carries sensory (afferent) information into the spinal cord. This includes the Ia afferent input from the muscle spindle. If the dorsal root is cut (rhizotomy), the CNS no longer receives the signal that the muscle is being stretched. Consequently, the alpha motor neuron is not activated reflexively, and the stretch reflex (and muscle tone) is lost (areflexia). The motor output (ventral root) remains intact, so voluntary movement is possible, but the reflex is gone. Therefore, the correct answer is c) Ia afferent input.
10. In the numerical classification, Group III fibers (equivalent to A-delta) typically carry sensory information from muscle regarding:
a) Muscle length
b) Muscle tension
c) Pain and temperature (free nerve endings)
d) Vibration
Explanation: While Group I and II are specialized proprioceptors, muscle nerves also contain smaller fibers. Group III (myelinated, small diameter = A-delta) and Group IV (unmyelinated = C fibers) innervate free nerve endings within the muscle connective tissue. They serve as muscle nociceptors (detecting Pain, e.g., from ischemia/cramps) and ergoreceptors (detecting metabolic products). They do not carry length or tension data for the stretch reflex. Therefore, the correct answer is c) Pain and temperature (free nerve endings).
Chapter: General Physiology; Topic: Nerve-Muscle Physiology; Subtopic: Muscle Spindle and Stretch Reflex
Key Definitions & Concepts
Muscle Spindle: A specialized sensory receptor located within the belly of skeletal muscles that detects changes in muscle length (stretch) and the rate of change.
Intrafusal Fibers: Specialized muscle fibers inside the spindle (Nuclear Bag and Nuclear Chain fibers) that are innervated by gamma motor neurons.
Extrafusal Fibers: The main force-generating muscle fibers outside the spindle, innervated by alpha motor neurons.
Muscle Tone: The continuous and passive partial contraction of the muscles, or the muscle's resistance to passive stretch during resting state; maintained by the stretch reflex.
Stretch Reflex (Myotatic Reflex): A monosynaptic reflex arc where stretching a muscle leads to its immediate contraction to resist the stretch (e.g., Knee jerk).
Alpha-Gamma Coactivation: The simultaneous firing of alpha and gamma motor neurons during voluntary contraction to ensure the spindle remains sensitive to stretch even as the muscle shortens.
Ia Afferents: Large, fast sensory fibers from the primary endings of the spindle that detect the rate of stretch (Dynamic response).
Type II Afferents: Sensory fibers from secondary endings that detect the static length of the muscle.
Reciprocal Inhibition: The mechanism where the antagonist muscle is relaxed (inhibited) when the agonist muscle contracts during a reflex.
Golgi Tendon Organ: A different receptor located in tendons that detects muscle tension (force), not length.
[Image of Muscle spindle structure]
Lead Question - 2016
Function of muscle spindle is?
a) Movement of a limb
b) Muscle tone maintenance
c) Goal oriented muscle contraction
d) All of the above
Explanation: The muscle spindle is a proprioceptor that detects muscle length and changes in length. Its primary physiological role is to mediate the Stretch Reflex. When a muscle is passively stretched, the spindle sends a signal to the spinal cord, which reflexively activates the alpha motor neurons to contract the muscle and resist the stretch. This continuous feedback loop maintains a baseline level of contraction known as Muscle Tone. While it is involved in smoothing movements, the generation of limb movement (a) and goal-oriented contraction (c) are primarily functions of the motor cortex and alpha motor neurons. Therefore, the most direct physiological function listed is tone maintenance. Therefore, the correct answer is b) Muscle tone maintenance.
1. The sensitivity of the muscle spindle to stretch is adjusted by which type of motor neuron?
a) Alpha motor neurons
b) Beta motor neurons
c) Gamma motor neurons
d) Delta motor neurons
Explanation: Intrafusal muscle fibers within the spindle have contractile elements at their poles. These poles are innervated by Gamma Motor Neurons (fusimotor neurons). When gamma neurons fire, the poles contract, stretching the central sensory region of the spindle. This "pre-stretches" the spindle, making it more sensitive to any subsequent stretch of the whole muscle. Without gamma innervation, the spindle would become slack and useless during muscle shortening. Thus, the gamma system acts as a "volume control" or sensitivity tuner for the stretch reflex. Therefore, the correct answer is c) Gamma motor neurons.
2. Which sensory nerve fiber type provides the primary (annulospiral) ending of the muscle spindle, detecting the velocity of stretch?
a) Type Ib
b) Type II
c) Type Ia
d) Type III
Explanation: The muscle spindle has two types of sensory endings. The Primary (Annulospiral) ending wraps around the center of both nuclear bag and chain fibers. It is innervated by Type Ia fibers (fastest conduction). These endings are sensitive to both the absolute length and, crucially, the rate of change (velocity) of length (Dynamic response). The Secondary (Flower-spray) ending is innervated by Type II fibers and detects only static length. Type Ib fibers innervate Golgi Tendon Organs. Therefore, the correct answer is c) Type Ia.
3. The "Knee Jerk" reflex is a classic example of a deep tendon reflex. Anatomically, this reflex arc is:
a) Polysynaptic
b) Monosynaptic
c) Bisynaptic
d) Asynaptic
Explanation: The stretch reflex (myotatic reflex) is unique in the nervous system because it is Monosynaptic. The afferent neuron (Type Ia fiber from the muscle spindle) enters the spinal cord dorsal horn and synapses directly onto the alpha motor neuron of the same muscle in the anterior horn. There are no interneurons involved in the excitatory pathway to the agonist muscle. This allows for the extremely rapid response time observed in deep tendon reflexes. (Note: The inhibition of the antagonist muscle is polysynaptic). Therefore, the correct answer is b) Monosynaptic.
4. During voluntary muscle contraction, both alpha and gamma motor neurons are activated simultaneously. This phenomenon, known as Alpha-Gamma Coactivation, ensures that:
a) The muscle generates maximum force
b) The muscle relaxes quickly
c) The muscle spindle remains sensitive to stretch during shortening
d) The Golgi tendon organ is inhibited
Explanation: When extrafusal fibers contract (alpha activation), the muscle shortens. If the intrafusal fibers did not also contract, the spindle would go slack (unloaded) and stop sending signals. Alpha-Gamma Coactivation ensures that as the muscle shortens, the poles of the spindle also contract, keeping the central sensory region taut. This maintains the spindle's sensitivity, allowing the CNS to continue monitoring muscle length and make adjustments throughout the contraction. Therefore, the correct answer is c) The muscle spindle remains sensitive to stretch during shortening.
5. Unlike the muscle spindle which is arranged in parallel with muscle fibers, the Golgi Tendon Organ (GTO) is arranged in series. The GTO primarily detects:
a) Muscle Length
b) Muscle Tension (Force)
c) Joint Angle
d) Rate of stretch
Explanation: The anatomical arrangement dictates the function. Muscle spindles are "in parallel" and stretch when the muscle length increases. Golgi Tendon Organs (GTOs) are located in the tendon, "in series" with the muscle fibers. When the muscle contracts, it pulls on the tendon, increasing tension. Thus, GTOs are sensitive detectors of Muscle Tension (Force). Activation of GTOs (via Ib fibers) triggers the Inverse Stretch Reflex (Autogenic Inhibition), causing the muscle to relax to prevent damage from excessive force. Therefore, the correct answer is b) Muscle Tension (Force).
6. The Nuclear Bag fibers of the muscle spindle are specifically responsible for the:
a) Static response only
b) Dynamic response (rate of change)
c) Metabolic support
d) Transmission of pain
Explanation: Intrafusal fibers are divided into Nuclear Bag and Nuclear Chain fibers. Nuclear Chain fibers detect static length. Nuclear Bag fibers are further divided into static and dynamic types. The Dynamic Nuclear Bag fibers are crucial for the Dynamic response. They respond vigorously while the muscle length is changing (detecting velocity), causing a burst of firing in Ia afferents. This allows the nervous system to predict where the limb will be. The Static response is mediated by both Chain and Static Bag fibers. Therefore, the correct answer is b) Dynamic response (rate of change).
7. Clonus, a series of rhythmic involuntary muscle contractions, is a clinical sign of:
a) Lower Motor Neuron lesion
b) Cerebellar disease
c) Upper Motor Neuron lesion (Hyperactive stretch reflex)
d) Basal Ganglia disorder
Explanation: Clonus occurs when the stretch reflex is highly hyperexcitable. A sudden stretch (e.g., dorsiflexion of the foot) triggers a reflex contraction. In a hyperactive state, the relaxation following the contraction stretches the spindle again, triggering another reflex, creating an oscillating cycle. This hyper-reflexia is a hallmark of an Upper Motor Neuron (UMN) lesion (e.g., stroke, spinal cord injury), where the descending inhibitory control over the spinal reflex arc is lost. LMN lesions abolish reflexes (areflexia). Therefore, the correct answer is c) Upper Motor Neuron lesion (Hyperactive stretch reflex).
8. The phenomenon of "Reciprocal Inhibition" during the knee jerk reflex involves:
a) Contraction of the Hamstrings
b) Inhibition of the Quadriceps motor neurons
c) Inhibition of the Hamstrings motor neurons via an interneuron
d) Direct inhibition of the Hamstrings by the Ia afferent
Explanation: For the quadriceps (agonist) to extend the knee effectively during the reflex, the opposing hamstrings (antagonist) must relax. The Ia afferent from the quadriceps spindle splits in the spinal cord. One branch excites the quadriceps alpha motor neuron directly. Another branch synapses on an Inhibitory Interneuron (Ia inhibitory interneuron). This interneuron releases Glycine to Inhibit the alpha motor neurons of the Hamstrings. This coordinated relaxation of the antagonist is called Reciprocal Inhibition. It is polysynaptic. Therefore, the correct answer is c) Inhibition of the Hamstrings motor neurons via an interneuron.
9. Jendrassik's maneuver (clenching teeth or pulling hands apart) facilitates deep tendon reflexes by:
a) Increasing alpha motor neuron excitability directly
b) Increasing gamma motor neuron drive to the spindles
c) Decreasing muscle tension
d) Inhibiting the Golgi tendon organ
Explanation: If a patient's reflexes are difficult to elicit, Jendrassik's maneuver is used. The mechanism involves the reduction of descending inhibition and an increase in the general state of arousal of the spinal cord. Specifically, it increases the descending drive to the Gamma motor neurons. Increased gamma activity pre-stretches the muscle spindles, making them more sensitive to the tap of the reflex hammer. This lowers the threshold for the reflex, making it easier to elicit (Reinforcement). Therefore, the correct answer is b) Increasing gamma motor neuron drive to the spindles.
10. Muscle tone is essentially a state of partial tetanus maintained by:
a) Continuous firing of Golgi Tendon Organs
b) Asynchronous firing of motor units driven by the stretch reflex
c) Synchronous firing of all motor units
d) Passive elasticity of Titin only
Explanation: Muscle tone is the resistance of muscle to passive stretch. It has a passive component (titin/connective tissue) but the active component is neural. It is maintained by the feedback loop from the muscle spindles (stretch reflex). To prevent fatigue and jerky movements, the CNS stimulates different motor units at different times. This Asynchronous firing of motor units creates a smooth, continuous, low-level tension in the muscle. If the dorsal roots are cut (interrupting the reflex loop), active muscle tone is lost (flaccidity). Therefore, the correct answer is b) Asynchronous firing of motor units driven by the stretch reflex.
Chapter: General Physiology; Topic: Nerve-Muscle Physiology; Subtopic: Skeletal Muscle Fiber Types
Key Definitions & Concepts
Red Muscle Fibers (Type I): Slow-twitch, oxidative fibers rich in myoglobin and mitochondria; specialized for endurance and posture (e.g., Soleus).
White Muscle Fibers (Type IIb): Fast-twitch, glycolytic fibers with low myoglobin content; specialized for rapid, powerful bursts of movement but fatigue easily (e.g., Extraocular muscles).
Myoglobin: The oxygen-binding heme protein present in muscle; its high concentration gives Red fibers their color and oxygen storage capacity.
Sarcoplasmic Reticulum (SR): More extensive in White fibers to facilitate rapid Calcium release and uptake for fast contraction cycles.
Henneman's Size Principle: Motor units are recruited in order of size: Small (Type I) first, followed by Medium (Type IIa), and finally Large (Type IIb) for maximal force.
Myosin ATPase: The enzyme on the myosin head determining contraction speed; White fibers have high ATPase activity (Fast), Red fibers have low activity (Slow).
Glycolysis: The anaerobic metabolic pathway predominantly used by White fibers, leading to lactic acid accumulation and fatigue.
Postural Muscles: Antigravity muscles (back, legs) composed largely of fatigue-resistant Red fibers to maintain standing posture.
Soleus vs. Gastrocnemius: A classic comparison; Soleus is predominantly Red (posture), while Gastrocnemius is mixed/White (sprinting/jumping).
Capillary Density: High in Red fibers to supply oxygen; low in White fibers as they rely on anaerobic glycogen stores.
[Image of Red vs White muscle fiber comparison table]
Lead Question - 2016
White fibers are present in which muscle?
a) Calf muscles
b) Back muscles
c) Gluteal muscles
d) Hand muscles
Explanation: Muscle fibers are classified into Type I (Red, Slow) and Type II (White, Fast). Red fibers are fatigue-resistant and adapted for tonic, postural sustained contractions; therefore, they are abundant in the deep muscles of the back, the gluteal muscles, and the soleus (deep calf muscle). White fibers are adapted for phasic, rapid, fine, or powerful movements but fatigue quickly. The intrinsic muscles of the Hand (and the extraocular muscles) require rapid, precise movements and contain a high proportion of White fibers (Type II). While most human muscles are a mix, the functional requirement of the hand dictates a dominance of fast-twitch units. Therefore, the correct answer is d) Hand muscles.
1. Which histological feature is more prominent in White (Type IIb) muscle fibers compared to Red (Type I) fibers?
a) Number of Mitochondria
b) Capillary density
c) Concentration of Myoglobin
d) Extensive Sarcoplasmic Reticulum
Explanation: White fibers are specialized for speed. To achieve rapid contraction and relaxation cycles, they require very fast cycling of Calcium ions. Anatomically, this is supported by a much more Extensive Sarcoplasmic Reticulum and a higher density of T-tubules compared to red fibers. This allows for rapid release and re-uptake of calcium. In contrast, Red fibers rely on aerobic metabolism and thus have higher mitochondrial density, capillary density, and myoglobin content (which gives them the red color). White fibers sacrifice these aerobic structures for the machinery of speed. Therefore, the correct answer is d) Extensive Sarcoplasmic Reticulum.
2. A marathon runner typically has a higher proportion of Type I (Red) fibers in their quadriceps compared to a sprinter. Which enzyme would be found in high concentration in the biopsy of the marathon runner's muscle?
a) Phosphofructokinase
b) Succinate Dehydrogenase
c) Glycogen Phosphorylase
d) Myosin ATPase (Fast isoform)
Explanation: Type I (Red) fibers are oxidative. They generate ATP via the Krebs cycle and Oxidative Phosphorylation. Therefore, they are rich in mitochondrial enzymes involved in these pathways. Succinate Dehydrogenase (SDH) is a key enzyme of the Krebs cycle and a marker for oxidative capacity. A marathon runner, whose muscles are adapted for endurance, will have high levels of SDH. Conversely, a sprinter (Type II dominant) would have high levels of glycolytic enzymes like Phosphofructokinase and Glycogen Phosphorylase, and high Fast Myosin ATPase activity for speed. Therefore, the correct answer is b) Succinate Dehydrogenase.
3. According to Henneman's Size Principle, during a graded muscle contraction, which motor units are recruited last?
a) Type I (Slow Oxidative)
b) Type IIa (Fast Oxidative-Glycolytic)
c) Type IIb (Fast Glycolytic)
d) Gamma motor neurons
Explanation: The central nervous system recruits motor units in a specific order based on the size of the alpha motor neuron soma. Small motor neurons (innervating Type I fibers) have the lowest threshold and are recruited first for weak contractions. As the required force increases, larger neurons are recruited. The largest motor neurons, which innervate Type IIb (Fast Glycolytic) white fibers, have the highest threshold and are recruited last. These fibers generate the greatest force but fatigue rapidly, so they are reserved for maximal effort (like lifting a heavy weight). Therefore, the correct answer is c) Type IIb (Fast Glycolytic).
4. The Soleus muscle is often cited as a classic example of a "Red" muscle. What is its primary physiological role that necessitates this fiber type composition?
a) Explosive jumping movements
b) Fine motor control of the ankle
c) Maintenance of standing posture against gravity
d) Rapid dorsiflexion
Explanation: The Soleus lies deep to the Gastrocnemius. While the Gastrocnemius is mixed and used for running/jumping, the Soleus is a tonic, antigravity muscle. Its primary function is to prevent the body from falling forward at the ankle during standing. This requires continuous, low-level contraction for hours without fatigue. Type I (Red) fibers are perfectly suited for this role because they are fatigue-resistant and efficient at aerobic metabolism. If the soleus were composed of white fibers, we would collapse from fatigue shortly after standing up. Therefore, the correct answer is c) Maintenance of standing posture against gravity.
5. Which metabolic characteristic is the primary cause of the rapid fatigue seen in White (Type IIb) muscle fibers?
a) Depletion of fatty acids
b) Accumulation of Lactic Acid
c) Lack of Calcium
d) Excess Myoglobin
Explanation: White fibers rely almost exclusively on Anaerobic Glycolysis for ATP production because they lack sufficient mitochondria for oxidative phosphorylation. They contain large stores of glycogen which are rapidly broken down. The end product of anaerobic glycolysis is pyruvate, which is converted to Lactate (Lactic Acid) to regenerate NAD+. The rapid Accumulation of Lactic Acid (and protons) lowers the intracellular pH, which inhibits glycolytic enzymes (like PFK) and interferes with actin-myosin interaction, leading to rapid muscle fatigue. Red fibers oxidize pyruvate, avoiding this buildup. Therefore, the correct answer is b) Accumulation of Lactic Acid.
6. In terms of innervation, the alpha motor neurons supplying Type IIb (White) fibers differ from those supplying Type I fibers in that they:
a) Have smaller cell bodies
b) Have slower conduction velocities
c) Innervate a larger number of muscle fibers (High innervation ratio)
d) Are tonically active at rest
Explanation: A "Motor Unit" consists of one motor neuron and all the muscle fibers it innervates. Motor units governing fine control (eye/hand) are small. However, when comparing fiber types generally, the fast-twitch (Type IIb) motor units are large. The alpha motor neuron has a large cell body, a fast conducting axon, and crucially, it branches to innervate a Larger number of muscle fibers (often hundreds or thousands). This High Innervation Ratio generates a large amount of force when the unit fires ("Gross motor"). Type I units have fewer fibers per neuron. Therefore, the correct answer is c) Innervate a larger number of muscle fibers (High innervation ratio).
7. Which of the following muscles is predominantly composed of Fast Glycolytic (White) fibers to facilitate extremely rapid movements?
a) Soleus
b) Erector Spinae
c) Extraocular muscles
d) Tibialis Anterior
Explanation: The Extraocular muscles (e.g., Lateral Rectus) are required to perform saccadic eye movements, which are among the fastest movements in the body. To achieve this high velocity, they are composed mainly of Type II (White) fibers with extremely fast myosin ATPase activity and extensive sarcoplasmic reticulum. While they also have a unique "non-twitch" fiber type for holding gaze, the bulk of the muscle involved in saccades is fast-twitch. Postural muscles like the Erector Spinae and Soleus are predominantly Red. Therefore, the correct answer is c) Extraocular muscles.
8. A muscle biopsy is stained for Myosin ATPase at alkaline pH (pH 9.4). The fibers that stain darkly are classified as:
a) Type I fibers
b) Type II fibers
c) Intermediate fibers
d) Denervated fibers
Explanation: Histochemistry is the gold standard for fiber typing. The assay exploits the different pH sensitivities of the Myosin ATPase enzyme isoforms. The "Fast" ATPase found in Type II fibers is stable and active at alkaline pH (basic). The "Slow" ATPase found in Type I fibers is acid-stable but alkali-labile. Therefore, when pre-incubated at pH 9.4 (alkaline), the Type II (White) fibers retain activity and stain darkly, while Type I fibers appear light. Conversely, at acidic pH (4.3), Type I fibers stain darkly. Therefore, the correct answer is b) Type II fibers.
9. After amputation of a limb, the remaining stump muscles often undergo atrophy. If a nerve from a slow muscle is cross-innervated to a fast muscle experimentally, what happens to the fiber type?
a) No change occurs
b) The muscle becomes necrotic
c) The fast muscle transforms into a slow muscle
d) The slow nerve changes its firing pattern to match the muscle
Explanation: This refers to the classic "Cross-Innervation Experiments" (Buller, Eccles, and Eccles). The firing pattern of the motor neuron dictates the phenotype of the muscle fiber. If a nerve that normally drives a slow muscle (tonic, low frequency firing) is surgically connected to a fast muscle, the fast muscle undergoes gene expression changes (myosin isoform switching) and Transforms into a slow muscle. This proves that neural input (trophic factors and firing frequency) determines fiber type differentiation (" The nerve dictates the muscle"). Therefore, the correct answer is c) The fast muscle transforms into a slow muscle.
10. Type IIa fibers are often called "Intermediate" fibers. They differ from Type IIb (White) fibers because Type IIa fibers possess:
a) No mitochondria
b) Very slow contraction speed
c) Significant oxidative capacity and fatigue resistance
d) Only anaerobic metabolism
Explanation: Type IIa fibers represent a hybrid physiology. Like Type IIb, they have Fast Myosin ATPase and contract rapidly (Fast Twitch). However, unlike the purely glycolytic Type IIb, Type IIa fibers contain a significant number of mitochondria and myoglobin (making them appear red/pink). This gives them a Significant oxidative capacity (Fast Oxidative-Glycolytic). Consequently, they are moderately resistant to fatigue, unlike the easily fatigable Type IIb fibers. They are recruited after Type I but before Type IIb. Therefore, the correct answer is c) Significant oxidative capacity and fatigue resistance.
Chapter: General Physiology; Topic: Nerve-Muscle Physiology; Subtopic: Structure of the Sarcomere
Key Definitions & Concepts
Sarcomere: The functional contractile unit of a muscle fiber; defined as the segment between two consecutive Z lines (disks).
A Band (Anisotropic): The dark band containing the entire length of the Thick filaments (Myosin) and the overlapping parts of the Thin filaments. Its length remains constant during contraction.
I Band (Isotropic): The light band containing only Thin filaments (Actin) and no thick filaments. It spans across two adjacent sarcomeres, bisected by the Z line.
H Zone: The central region of the A Band containing only thick filaments (no actin overlap). It shortens/disappears during contraction.
Z Line: The dark line bisecting the I Band; anchors the thin filaments.
M Line: The dark line in the center of the H zone; anchors the thick filaments.
Sarcomere Length Formula: Length = Length of A Band + (2 x Half-I Band). Since the "I Band" usually refers to the *full* light band spanning two sarcomeres, the portion *within* one sarcomere is two halves, equaling the total width of one I band. Thus, Sarcomere = A Band + I Band (width).
Sliding Filament Theory: During contraction, thin filaments slide over thick filaments. The Z lines move closer together, shortening the I band and H zone, but the A band length is unchanged.
Optimal Length: The sarcomere length (2.0-2.2 µm) at which maximal tension can be generated due to optimal overlap.
Titin: A giant elastic protein connecting the Z line to the M line, stabilizing the thick filament.
Lead Question - 2016
In a muscle fiber at rest, the length of the I band is 1 mm and A band is 1.5 mm. What is the length of the sarcomere?
a) 0.5 mm
b) 2.5 mm
c) 3.5 mm
d) 5 mm
Explanation: A single sarcomere extends from one Z-line to the next Z-line. The structure consists of one full A Band in the center. Flanking the A band on either side is half of an I Band (since the full I band is bisected by the Z-line and shared between two sarcomeres). Therefore, the total length of one sarcomere = Length of A Band + (Length of ½ I Band + Length of ½ I Band). Mathematically, this simplifies to: Sarcomere Length = Length of A Band + Length of one full I Band. Given: A Band = 1.5 mm (micrometers usually, but mm in question), I Band = 1.0 mm. Calculation: 1.5 + 1.0 = 2.5 mm. Therefore, the correct answer is b) 2.5 mm.
1. Which band of the sarcomere disappears completely during maximal skeletal muscle contraction?
a) A Band
b) Z Line
c) H Zone
d) M Line
Explanation: During contraction, the thin filaments (actin) slide over the thick filaments (myosin) toward the center of the sarcomere (M line). The H Zone is the central part of the A band that, in a resting state, contains only myosin and no actin overlap. As the actin filaments slide inward, they invade the H zone. At maximal contraction, the actin filaments from opposite sides meet (and may even overlap) at the center, causing the H Zone to disappear completely. The I band also narrows significantly but doesn't technically disappear until extreme shortening. The A band remains constant. Therefore, the correct answer is c) H Zone.
2. The "Z-line" (Zwischenscheibe) creates the physical boundary of the sarcomere. Which protein is the primary structural component anchoring Actin filaments to the Z-line?
a) Myomesin
b) Alpha-actinin
c) Titin
d) Desmin
Explanation: The Z-line is a dense protein disc. The thin filaments (Actin) are anchored here. The primary protein responsible for cross-linking the actin filaments to the Z-disc lattice is Alpha-actinin. Myomesin is found at the M-line (anchoring myosin). Titin connects the Z-line to the M-line (providing elasticity). Desmin forms intermediate filaments that connect adjacent Z-lines (sarcomeres) to each other laterally, keeping myofibrils aligned. Therefore, the correct answer is b) Alpha-actinin.
3. Which protein extends from the Z-line to the M-line and is responsible for the passive elasticity (resting tension) of the muscle?
a) Nebulin
b) Dystrophin
c) Titin
d) Tropomyosin
Explanation: Titin (Connectin) is a giant protein (the largest known). It spans half the sarcomere, anchoring the thick filament to the Z-line and extending to the M-line. A portion of the Titin molecule in the I-band region acts like a molecular spring. When the muscle is stretched, Titin uncoils, generating Passive Tension (elastic recoil) that prevents overstretching and helps the muscle return to its resting length. Nebulin acts as a ruler for actin length. Dystrophin connects the cytoskeleton to the membrane. Therefore, the correct answer is c) Titin.
4. The "M Line" (Mittelscheibe) is located in the center of the sarcomere. It serves as the attachment point for:
a) Thin filaments
b) Thick filaments
c) Z-discs
d) T-tubules
Explanation: The sarcomere has structural symmetry. While the Z-line anchors the thin filaments at the ends, the M Line acts as the anchor point for the Thick filaments (Myosin) in the center of the sarcomere. Proteins like Myomesin and C-protein are found here, holding the myosin bundles in a hexagonal lattice. This anchoring ensures that the thick filaments remain centered during the sliding process of contraction. The enzyme Creatine Kinase is also located at the M-line to regenerate ATP. Therefore, the correct answer is b) Thick filaments.
5. Which band of the sarcomere contains the enzyme Creatine Kinase to rapidly regenerate ATP?
a) Z Line
b) I Band
c) M Line
d) Actin-Myosin Overlap zone
Explanation: Muscle contraction requires immediate ATP availability. The enzyme Creatine Kinase (MM isoform) transfers a phosphate group from Phosphocreatine to ADP to regenerate ATP. Structurally, this enzyme is bound to the M Line (within the A band) of the sarcomere. This strategic location places the ATP-regenerating machinery right next to the Myosin heads (which are the ATPases) in the center of the thick filament array, ensuring efficient energy supply during contraction. Therefore, the correct answer is c) M Line.
6. In the cross-section of a myofibril at the A-I overlap zone, each Thick filament is surrounded by how many Thin filaments?
a) 2
b) 3
c) 4
d) 6
Explanation: The myofilaments are arranged in a precise hexagonal lattice. In the region where actin and myosin overlap (A band), each Thick filament (Myosin) is surrounded by a hexagonal array of 6 Thin filaments (Actin). Conversely, each Thin filament sits in the center of a triangle formed by 3 Thick filaments. This 1:6 (or 2:1 ratio overall) geometry allows each myosin head to interact with multiple actin filaments, maximizing force generation. Therefore, the correct answer is d) 6.
7. If the length of a sarcomere is stretched beyond 3.65 micrometers, the active tension developed drops to zero because:
a) Titin breaks
b) There is no overlap between thick and thin filaments
c) The H zone disappears
d) Calcium cannot bind to Troponin
Explanation: The Length-Tension relationship is fundamental to muscle physiology. Active tension depends on the number of cross-bridges formed. At a sarcomere length of ~3.65 µm (in frog muscle, often cited as the limit), the actin filaments are pulled completely out of the A band. Consequently, There is no overlap between thick and thin filaments. Without overlap, myosin heads cannot bind to actin, no cross-bridges can form, and the active tension generated is zero. The muscle is "disengaged." Therefore, the correct answer is b) There is no overlap between thick and thin filaments.
8. Which of the following proteins serves as a "molecular ruler" to regulate the length of the Thin (Actin) filament?
a) Titin
b) Nebulin
c) Myomesin
d) Obscurin
Explanation: The precise length of filaments is critical for the crystalline structure of the sarcomere. Nebulin is a large, non-elastic filamentous protein that runs along the entire length of the Thin Filament (Actin). It is anchored at the Z-line. It is thought to act as a "Molecular Ruler" that dictates exactly how long the actin polymer grows during muscle development. Titin performs a similar ruler/scaffold function for the Thick filament (Myosin). Therefore, the correct answer is b) Nebulin.
9. During isotonic contraction, which measurement changes?
a) Length of the A band
b) Length of the Myosin filament
c) Length of the I band
d) Length of the Actin filament
Explanation: In isotonic contraction, the muscle shortens. Based on the sliding filament theory: 1. Actin and Myosin filament lengths remain constant. 2. The A band (Myosin length) remains constant. 3. The filaments slide past each other. 4. The Z-lines move closer together. 5. This shortening of the sarcomere occurs at the expense of the I Band and the H Zone, which both decrease in length (shorten). Therefore, the visible change is in the I band. Therefore, the correct answer is c) Length of the I band.
10. The region of the sarcomere containing ONLY Actin filaments is the:
a) A Band
b) H Zone
c) I Band
d) M Line
Explanation: Definitions are key here. A Band = Length of Myosin (includes Actin overlap). H Zone = Center of A Band with Myosin ONLY. I Band = The light band spanning the Z-line, containing ONLY Actin (Thin) filaments (no Myosin). M Line = Center anchor for Myosin. Therefore, the region exclusively containing actin is the I Band. Therefore, the correct answer is c) I Band.
Chapter: General Physiology; Topic: Nerve-Muscle Physiology; Subtopic: Ultrastructure of Cardiac vs. Skeletal Muscle
Key Definitions & Concepts
T-Tubules (Transverse Tubules): Invaginations of the sarcolemma (cell membrane) that penetrate deep into the muscle fiber, allowing the action potential to reach the interior quickly.
Z Line (Disk): The boundary of the sarcomere; in cardiac muscle, T-tubules are located here.
A-I Junction: The region where the A band (thick filaments) and I band (thin filaments) overlap; in skeletal muscle, T-tubules are located here.
Diad (Dyad): The structure formed in cardiac muscle by one T-tubule and one terminal cisterna of the Sarcoplasmic Reticulum (SR).
Triad: The structure formed in skeletal muscle by one T-tubule and two terminal cisternae of the SR.
Calcium-Induced Calcium Release (CICR): The mechanism in cardiac muscle where the influx of extracellular Ca2+ through L-type channels triggers the release of stored Ca2+ from the SR.
L-Type Calcium Channel (DHP Receptor): Voltage-gated channels in the T-tubule; in cardiac muscle, they act as functional Ca2+ channels essential for contraction.
Ryanodine Receptor (RyR): The Calcium release channel on the SR; RyR2 is the cardiac isoform, while RyR1 is skeletal.
Intercalated Discs: Specialized cell junctions connecting cardiac myocytes, containing desmosomes (mechanical strength) and gap junctions (electrical continuity).
Phospholamban: A regulatory protein in cardiac muscle SR that, when phosphorylated (by sympathetic stimulation), increases SERCA activity and relaxation rate.
Lead Question - 2016
In cardiac muscles, T-tubules are present at?
a) Z lines
b) A lines
c) I lines
d) A-I junction
Explanation: There is a distinct anatomical difference between skeletal and cardiac muscle regarding the location of the Transverse Tubules (T-tubules). In mammalian Skeletal Muscle, the T-tubules are located at the A-I Junctions (the overlap between actin and myosin). Since there are two A-I junctions per sarcomere, skeletal muscle has two T-tubules per sarcomere. In contrast, in Cardiac Muscle (ventricular myocytes), the T-tubules are located at the Z lines (Z disks). Because there is only one Z line per sarcomere end (sharing with the next), there is effectively one T-tubule per sarcomere. Cardiac T-tubules are also significantly wider (larger diameter) than skeletal ones. Therefore, the correct answer is a) Z lines.
1. Which of the following structural arrangements represents the Calcium release unit in mammalian Cardiac muscle?
a) Triad located at the Z line
b) Triad located at the A-I junction
c) Diad located at the Z line
d) Diad located at the A-I junction
Explanation: The interaction between the T-tubule and the Sarcoplasmic Reticulum (SR) differs between muscle types. In skeletal muscle, one T-tubule is flanked by two terminal cisternae of the SR, forming a "Triad." In Cardiac muscle, the SR is less well-developed and does not form extensive terminal cisternae. Instead, small expansions of the SR make contact with the T-tubule. Usually, a single T-tubule associates with a single subsarcolemmal SR cisterna, forming a Diad (or Dyad). As established in the lead question, these cardiac T-tubules (and thus the Diads) are located at the Z line. Therefore, the correct answer is c) Diad located at the Z line.
2. The mechanism of Excitation-Contraction coupling in cardiac muscle differs from skeletal muscle because cardiac muscle is dependent on:
a) Mechanical coupling between DHP and RyR
b) Influx of extracellular Calcium (Calcium-Induced Calcium Release)
c) Sodium influx only
d) Complete independence from extracellular ions
Explanation: In skeletal muscle, the DHP receptor acts as a mechanical voltage sensor that physically pulls open the Ryanodine receptor (RyR1). Extracellular Ca2+ is not required for the release signal. In Cardiac muscle, the DHP receptor acts as a true Calcium channel. Depolarization opens these L-type channels, allowing a small amount of Extracellular Calcium to enter the cell. This entering Calcium ("trigger calcium") binds to the Ryanodine Receptors (RyR2) on the SR, causing them to open and release a massive amount of stored Calcium. This process is called Calcium-Induced Calcium Release (CICR). Therefore, the correct answer is b) Influx of extracellular Calcium (Calcium-Induced Calcium Release).
3. A patient with heart failure is prescribed a drug that inhibits the Na+/K+ ATPase. This leads to increased contractility. This mechanism works because the resulting intracellular Sodium accumulation directly affects the function of which transporter near the T-tubule?
a) SERCA pump
b) Na+/Ca2+ Exchanger (NCX)
c) L-type Calcium Channel
d) Ryanodine Receptor
Explanation: This describes the mechanism of Digoxin (Cardiac Glycosides). The Na+/K+ ATPase normally keeps intracellular Na+ low. Inhibition leads to increased intracellular Na+. This reduces the gradient for the Na+/Ca2+ Exchanger (NCX), which is located on the sarcolemma and T-tubules. The NCX normally pumps Ca2+ out using the energy of Na+ coming in. If intracellular Na+ is high, the gradient is weaker, and the NCX works less effectively (or reverses). Consequently, less Calcium is extruded, leading to higher intracellular Calcium, increased SR loading, and stronger contractions (positive inotropy). Therefore, the correct answer is b) Na+/Ca2+ Exchanger (NCX).
4. Structurally, how do the T-tubules of cardiac muscle compare to those of skeletal muscle?
a) Cardiac T-tubules are narrower and more numerous
b) Cardiac T-tubules are wider and fewer in number
c) Cardiac T-tubules are absent
d) Cardiac T-tubules are arranged longitudinally
Explanation: Anatomical adaptations reflect function. Cardiac T-tubules are significantly Wider (larger diameter, about 5 times wider) than those in skeletal muscle. This allows for easier diffusion of ions and nutrients into the deep myofibrils and provides a reservoir of extracellular calcium (glycocalyx helps trap Ca2+ here). However, because they are located only at the Z lines (1 per sarcomere) rather than the A-I junctions (2 per sarcomere), there are Fewer T-tubules in cardiac muscle compared to skeletal muscle. Therefore, the correct answer is b) Cardiac T-tubules are wider and fewer in number.
5. The Ryanodine Receptor isoform found in the Sarcoplasmic Reticulum of cardiac muscle is:
a) RyR1
b) RyR2
c) RyR3
d) IP3 Receptor
Explanation: The Calcium release channels on the Sarcoplasmic Reticulum are known as Ryanodine Receptors (RyR). There are distinct isoforms. RyR1 is the skeletal muscle isoform, which is physically coupled to the DHP receptor. RyR2 is the Cardiac muscle (and brain) isoform, which is activated by Calcium binding (CICR). RyR3 is found in the brain and other tissues. Mutations in RyR1 are associated with Malignant Hyperthermia, while mutations in RyR2 are associated with Catecholaminergic Polymorphic Ventricular Tachycardia (CPVT). Therefore, the correct answer is b) RyR2.
6. A calcium channel blocker like Verapamil exerts its negative inotropic effect by acting on which protein located in the T-tubule membrane?
a) Ryanodine Receptor
b) Na+/Ca2+ Exchanger
c) L-type Calcium Channel (DHP receptor)
d) T-type Calcium Channel
Explanation: Verapamil and Diltiazem are non-dihydropyridine calcium channel blockers. They target the L-type Voltage-Gated Calcium Channels located on the sarcolemma and within the T-tubules. In cardiac muscle, the influx of calcium through these channels is the absolute requirement to trigger contraction (Trigger Calcium). Blocking these channels reduces the amount of trigger calcium entering during the plateau phase (Phase 2), thereby reducing the amount of calcium released from the SR, leading to decreased force of contraction (Negative Inotropy). Therefore, the correct answer is c) L-type Calcium Channel (DHP receptor).
7. Phospholamban is a regulatory protein found in the SR membrane of cardiac muscle. When phosphorylated by Protein Kinase A (during sympathetic stimulation), it:
a) Inhibits the L-type Calcium channel
b) Inhibits SERCA, slowing relaxation
c) Relieves inhibition of SERCA, accelerating relaxation
d) Closes the Ryanodine Receptor
Explanation: In the resting state, unphosphorylated Phospholamban acts as a "brake" or inhibitor of the SERCA pump (Sarcoplasmic Endoplasmic Reticulum Calcium ATPase). During sympathetic stimulation (fight or flight), Beta-adrenergic stimulation leads to PKA activation. PKA phosphorylates Phospholamban. This phosphorylation removes the inhibition on SERCA. Consequently, SERCA pumps Calcium back into the SR faster and more efficiently. This accelerates muscle relaxation (Positive Lusitropy) and increases the SR calcium stores for the next beat (contributing to Positive Inotropy). Therefore, the correct answer is c) Relieves inhibition of SERCA, accelerating relaxation.
8. In skeletal muscle, the T-tubules are located at the A-I junction. This means there are how many T-tubules per sarcomere?
a) One
b) Two
c) Three
d) Four
Explanation: A sarcomere extends from one Z line to the next Z line. It contains one full A band in the center and two half I bands on either side. The boundary between the A band and the I band is the A-I Junction. Since there is an A-I junction on both sides of the A band within a single sarcomere, there are Two A-I junctions. Consequently, in skeletal muscle where T-tubules are located at these junctions, there are Two T-tubules per sarcomere. This contrasts with the single T-tubule (at the Z line) in cardiac muscle. Therefore, the correct answer is b) Two.
9. The electrical continuity between adjacent cardiac muscle cells, allowing them to function as a syncytium, is provided by:
a) T-tubules
b) Desmosomes
c) Gap Junctions
d) Fascia adherens
Explanation: Cardiac muscle cells are connected end-to-end by specialized structures called Intercalated Discs. These discs contain three types of junctions. Fascia adherens and Desmosomes provide mechanical strength, holding the cells together during contraction. Gap Junctions (Connexons) provide low-resistance electrical pathways that allow ions to flow freely from one cell to the next. This ensures that the action potential spreads rapidly across the entire heart muscle, allowing the ventricles to contract as a single coordinated unit or functional syncytium. Therefore, the correct answer is c) Gap Junctions.
10. Unlike skeletal muscle, cardiac muscle contraction cannot be tetanized (sustained contraction without relaxation). This is primarily due to:
a) The absence of Troponin
b) The very short refractory period
c) The long Absolute Refractory Period overlapping with contraction
d) The lack of T-tubules
Explanation: Tetanization requires high-frequency stimulation where new action potentials are generated before the muscle has relaxed from the previous twitch. In cardiac muscle, the Action Potential is very long (due to the L-type Ca2+ plateau). Consequently, the Absolute Refractory Period (time when Na+ channels are inactivated) is also extremely long (almost as long as the mechanical contraction itself). By the time the membrane is excitable again, the muscle has already begun to relax. This protective mechanism prevents summation and tetanus, which would be fatal for the heart's pumping function (filling requires relaxation). Therefore, the correct answer is c) The long Absolute Refractory Period overlapping with contraction.
Chapter: General Physiology; Topic: Nerve-Muscle Physiology; Subtopic: Ultrastructure of Cardiac vs. Skeletal Muscle
Key Definitions & Concepts
T-Tubules (Transverse Tubules): Invaginations of the sarcolemma (cell membrane) that penetrate deep into the muscle fiber, allowing the action potential to reach the interior quickly.
Z Line (Disk): The boundary of the sarcomere; in cardiac muscle, T-tubules are located here.
A-I Junction: The region where the A band (thick filaments) and I band (thin filaments) overlap; in skeletal muscle, T-tubules are located here.
Diad (Dyad): The structure formed in cardiac muscle by one T-tubule and one terminal cisterna of the Sarcoplasmic Reticulum (SR).
Triad: The structure formed in skeletal muscle by one T-tubule and two terminal cisternae of the SR.
Calcium-Induced Calcium Release (CICR): The mechanism in cardiac muscle where the influx of extracellular Ca2+ through L-type channels triggers the release of stored Ca2+ from the SR.
L-Type Calcium Channel (DHP Receptor): Voltage-gated channels in the T-tubule; in cardiac muscle, they act as functional Ca2+ channels essential for contraction.
Ryanodine Receptor (RyR): The Calcium release channel on the SR; RyR2 is the cardiac isoform, while RyR1 is skeletal.
Intercalated Discs: Specialized cell junctions connecting cardiac myocytes, containing desmosomes (mechanical strength) and gap junctions (electrical continuity).
Phospholamban: A regulatory protein in cardiac muscle SR that, when phosphorylated (by sympathetic stimulation), increases SERCA activity and relaxation rate.
Lead Question - 2016
In cardiac muscles, T-tubules are present at?
a) Z lines
b) A lines
c) I lines
d) A-I junction
Explanation: There is a distinct anatomical difference between skeletal and cardiac muscle regarding the location of the Transverse Tubules (T-tubules). In mammalian Skeletal Muscle, the T-tubules are located at the A-I Junctions (the overlap between actin and myosin). Since there are two A-I junctions per sarcomere, skeletal muscle has two T-tubules per sarcomere. In contrast, in Cardiac Muscle (ventricular myocytes), the T-tubules are located at the Z lines (Z disks). Because there is only one Z line per sarcomere end (sharing with the next), there is effectively one T-tubule per sarcomere. Cardiac T-tubules are also significantly wider (larger diameter) than skeletal ones. Therefore, the correct answer is a) Z lines.
1. Which of the following structural arrangements represents the Calcium release unit in mammalian Cardiac muscle?
a) Triad located at the Z line
b) Triad located at the A-I junction
c) Diad located at the Z line
d) Diad located at the A-I junction
Explanation: The interaction between the T-tubule and the Sarcoplasmic Reticulum (SR) differs between muscle types. In skeletal muscle, one T-tubule is flanked by two terminal cisternae of the SR, forming a "Triad." In Cardiac muscle, the SR is less well-developed and does not form extensive terminal cisternae. Instead, small expansions of the SR make contact with the T-tubule. Usually, a single T-tubule associates with a single subsarcolemmal SR cisterna, forming a Diad (or Dyad). As established in the lead question, these cardiac T-tubules (and thus the Diads) are located at the Z line. Therefore, the correct answer is c) Diad located at the Z line.
2. The mechanism of Excitation-Contraction coupling in cardiac muscle differs from skeletal muscle because cardiac muscle is dependent on:
a) Mechanical coupling between DHP and RyR
b) Influx of extracellular Calcium (Calcium-Induced Calcium Release)
c) Sodium influx only
d) Complete independence from extracellular ions
Explanation: In skeletal muscle, the DHP receptor acts as a mechanical voltage sensor that physically pulls open the Ryanodine receptor (RyR1). Extracellular Ca2+ is not required for the release signal. In Cardiac muscle, the DHP receptor acts as a true Calcium channel. Depolarization opens these L-type channels, allowing a small amount of Extracellular Calcium to enter the cell. This entering Calcium ("trigger calcium") binds to the Ryanodine Receptors (RyR2) on the SR, causing them to open and release a massive amount of stored Calcium. This process is called Calcium-Induced Calcium Release (CICR). Therefore, the correct answer is b) Influx of extracellular Calcium (Calcium-Induced Calcium Release).
3. A patient with heart failure is prescribed a drug that inhibits the Na+/K+ ATPase. This leads to increased contractility. This mechanism works because the resulting intracellular Sodium accumulation directly affects the function of which transporter near the T-tubule?
a) SERCA pump
b) Na+/Ca2+ Exchanger (NCX)
c) L-type Calcium Channel
d) Ryanodine Receptor
Explanation: This describes the mechanism of Digoxin (Cardiac Glycosides). The Na+/K+ ATPase normally keeps intracellular Na+ low. Inhibition leads to increased intracellular Na+. This reduces the gradient for the Na+/Ca2+ Exchanger (NCX), which is located on the sarcolemma and T-tubules. The NCX normally pumps Ca2+ out using the energy of Na+ coming in. If intracellular Na+ is high, the gradient is weaker, and the NCX works less effectively (or reverses). Consequently, less Calcium is extruded, leading to higher intracellular Calcium, increased SR loading, and stronger contractions (positive inotropy). Therefore, the correct answer is b) Na+/Ca2+ Exchanger (NCX).
4. Structurally, how do the T-tubules of cardiac muscle compare to those of skeletal muscle?
a) Cardiac T-tubules are narrower and more numerous
b) Cardiac T-tubules are wider and fewer in number
c) Cardiac T-tubules are absent
d) Cardiac T-tubules are arranged longitudinally
Explanation: Anatomical adaptations reflect function. Cardiac T-tubules are significantly Wider (larger diameter, about 5 times wider) than those in skeletal muscle. This allows for easier diffusion of ions and nutrients into the deep myofibrils and provides a reservoir of extracellular calcium (glycocalyx helps trap Ca2+ here). However, because they are located only at the Z lines (1 per sarcomere) rather than the A-I junctions (2 per sarcomere), there are Fewer T-tubules in cardiac muscle compared to skeletal muscle. Therefore, the correct answer is b) Cardiac T-tubules are wider and fewer in number.
5. The Ryanodine Receptor isoform found in the Sarcoplasmic Reticulum of cardiac muscle is:
a) RyR1
b) RyR2
c) RyR3
d) IP3 Receptor
Explanation: The Calcium release channels on the Sarcoplasmic Reticulum are known as Ryanodine Receptors (RyR). There are distinct isoforms. RyR1 is the skeletal muscle isoform, which is physically coupled to the DHP receptor. RyR2 is the Cardiac muscle (and brain) isoform, which is activated by Calcium binding (CICR). RyR3 is found in the brain and other tissues. Mutations in RyR1 are associated with Malignant Hyperthermia, while mutations in RyR2 are associated with Catecholaminergic Polymorphic Ventricular Tachycardia (CPVT). Therefore, the correct answer is b) RyR2.
6. A calcium channel blocker like Verapamil exerts its negative inotropic effect by acting on which protein located in the T-tubule membrane?
a) Ryanodine Receptor
b) Na+/Ca2+ Exchanger
c) L-type Calcium Channel (DHP receptor)
d) T-type Calcium Channel
Explanation: Verapamil and Diltiazem are non-dihydropyridine calcium channel blockers. They target the L-type Voltage-Gated Calcium Channels located on the sarcolemma and within the T-tubules. In cardiac muscle, the influx of calcium through these channels is the absolute requirement to trigger contraction (Trigger Calcium). Blocking these channels reduces the amount of trigger calcium entering during the plateau phase (Phase 2), thereby reducing the amount of calcium released from the SR, leading to decreased force of contraction (Negative Inotropy). Therefore, the correct answer is c) L-type Calcium Channel (DHP receptor).
7. Phospholamban is a regulatory protein found in the SR membrane of cardiac muscle. When phosphorylated by Protein Kinase A (during sympathetic stimulation), it:
a) Inhibits the L-type Calcium channel
b) Inhibits SERCA, slowing relaxation
c) Relieves inhibition of SERCA, accelerating relaxation
d) Closes the Ryanodine Receptor
Explanation: In the resting state, unphosphorylated Phospholamban acts as a "brake" or inhibitor of the SERCA pump (Sarcoplasmic Endoplasmic Reticulum Calcium ATPase). During sympathetic stimulation (fight or flight), Beta-adrenergic stimulation leads to PKA activation. PKA phosphorylates Phospholamban. This phosphorylation removes the inhibition on SERCA. Consequently, SERCA pumps Calcium back into the SR faster and more efficiently. This accelerates muscle relaxation (Positive Lusitropy) and increases the SR calcium stores for the next beat (contributing to Positive Inotropy). Therefore, the correct answer is c) Relieves inhibition of SERCA, accelerating relaxation.
8. In skeletal muscle, the T-tubules are located at the A-I junction. This means there are how many T-tubules per sarcomere?
a) One
b) Two
c) Three
d) Four
Explanation: A sarcomere extends from one Z line to the next Z line. It contains one full A band in the center and two half I bands on either side. The boundary between the A band and the I band is the A-I Junction. Since there is an A-I junction on both sides of the A band within a single sarcomere, there are Two A-I junctions. Consequently, in skeletal muscle where T-tubules are located at these junctions, there are Two T-tubules per sarcomere. This contrasts with the single T-tubule (at the Z line) in cardiac muscle. Therefore, the correct answer is b) Two.
9. The electrical continuity between adjacent cardiac muscle cells, allowing them to function as a syncytium, is provided by:
a) T-tubules
b) Desmosomes
c) Gap Junctions
d) Fascia adherens
Explanation: Cardiac muscle cells are connected end-to-end by specialized structures called Intercalated Discs. These discs contain three types of junctions. Fascia adherens and Desmosomes provide mechanical strength, holding the cells together during contraction. Gap Junctions (Connexons) provide low-resistance electrical pathways that allow ions to flow freely from one cell to the next. This ensures that the action potential spreads rapidly across the entire heart muscle, allowing the ventricles to contract as a single coordinated unit or functional syncytium. Therefore, the correct answer is c) Gap Junctions.
10. Unlike skeletal muscle, cardiac muscle contraction cannot be tetanized (sustained contraction without relaxation). This is primarily due to:
a) The absence of Troponin
b) The very short refractory period
c) The long Absolute Refractory Period overlapping with contraction
d) The lack of T-tubules
Explanation: Tetanization requires high-frequency stimulation where new action potentials are generated before the muscle has relaxed from the previous twitch. In cardiac muscle, the Action Potential is very long (due to the L-type Ca2+ plateau). Consequently, the Absolute Refractory Period (time when Na+ channels are inactivated) is also extremely long (almost as long as the mechanical contraction itself). By the time the membrane is excitable again, the muscle has already begun to relax. This protective mechanism prevents summation and tetanus, which would be fatal for the heart's pumping function (filling requires relaxation). Therefore, the correct answer is c) The long Absolute Refractory Period overlapping with contraction.
Chapter: General Physiology; Topic: Nerve-Muscle Physiology; Subtopic: Contractile and Regulatory Proteins of Muscle
Key Definitions & Concepts
Myosin: The contractile protein that forms the Thick Filaments. It is a molecular motor that converts chemical energy (ATP) into mechanical force.
Actin: The globular protein (G-actin) that polymerizes to form the double-helical Thin Filaments (F-actin). It contains the binding sites for myosin.
ATPase Activity: A localized enzymatic function on the Myosin Head (S1 fragment) that hydrolyzes ATP into ADP and inorganic phosphate (Pi) to energize the cross-bridge.
Tropomyosin: A regulatory filamentous protein that lies in the groove of the actin helix, covering the myosin-binding sites in the resting state.
Troponin Complex: A heterotrimer consisting of Troponin T (binds Tropomyosin), Troponin I (Inhibits binding), and Troponin C (binds Calcium).
Cross-Bridge Cycle: The cyclical attachment, power stroke, detachment, and re-cocking of myosin heads interacting with actin.
Power Stroke: The conformational change where the myosin head pivots, pulling the actin filament toward the center of the sarcomere (M-line); triggered by the release of Phosphate.
ATP Binding: Essential for the detachment of the myosin head from the actin filament after the power stroke.
Titin: A giant elastic protein connecting the Z-disk to the M-line, maintaining the alignment of the thick filament.
Sliding Filament Theory: Muscle contraction occurs by the sliding of thin filaments over thick filaments, shortening the sarcomere without changing filament length.
[Image of Myosin molecule structure]
Lead Question - 2016
True about myosin?
a) Thin filament
b) Covers active site of action
c) Has ATPase activity
d) Ca+ binding protein
Explanation: Myosin is the major contractile protein comprising the Thick Filament (Actin forms the Thin filament). It does not cover the active site; that is the function of Tropomyosin in the resting state. It does not bind Calcium directly to initiate contraction; Calcium binds to Troponin C on the thin filament. The defining functional characteristic of the Myosin head is that it acts as an enzyme: it possesses ATPase activity. It hydrolyzes ATP to ADP and Pi, storing the energy required for the power stroke and muscle contraction. Therefore, the correct answer is c) Has ATPase activity.
1. The detachment of the myosin cross-bridge from the actin filament is strictly dependent on the binding of:
a) Calcium ions
b) ADP
c) ATP
d) Magnesium
Explanation: The cross-bridge cycle involves distinct steps governed by ATP processing. The hydrolysis of ATP "cocks" the myosin head. The release of Inorganic Phosphate (Pi) triggers the Power Stroke. The release of ADP leaves the myosin head attached to actin in a low-energy "rigor" state. For the myosin head to detach and begin a new cycle, a new molecule of ATP must bind to the nucleotide-binding pocket on the myosin head. In the absence of ATP (as in death), detachment cannot occur, leading to Rigor Mortis. Therefore, the correct answer is c) ATP.
2. Which structural component of the sarcomere remains constant in length during skeletal muscle contraction?
a) I Band
b) H Zone
c) A Band
d) Distance between Z lines
Explanation: According to the Sliding Filament Theory, contraction involves the interdigitation of filaments, not the shortening of the filaments themselves. The A Band corresponds to the entire length of the Thick filaments (Myosin). Since the myosin filaments do not shorten, the width of the A Band remains constant. The I Band (thin filaments only) and the H Zone (thick filaments only) both shorten as the overlap increases. The distance between Z lines (the sarcomere length) decreases. Therefore, the correct answer is c) A Band.
3. A 10-year-old boy presents with progressive muscle weakness and calf pseudohypertrophy. He is diagnosed with Duchenne Muscular Dystrophy. This condition is caused by a defect in Dystrophin, a protein that links actin to:
a) The Z-disk
b) The M-line
c) The Transverse Tubules
d) The Sarcolemma and Extracellular Matrix
Explanation: Dystrophin is a massive cytoskeletal protein located on the inner surface of the muscle cell membrane (sarcolemma). Its crucial function is to mechanically link the internal cytoskeleton (specifically the Actin filaments) to the transmembrane Dystrophin-Glycoprotein Complex (DGC), which anchors the fiber to the Extracellular Matrix (laminin). This linkage stabilizes the sarcolemma during contraction. Absence of dystrophin leads to membrane fragility, calcium influx, and necrosis of muscle fibers (DMD). It does not link to Z-disks (Titin/Alpha-actinin do that). Therefore, the correct answer is d) The Sarcolemma and Extracellular Matrix.
4. During the resting state of skeletal muscle, the interaction between actin and myosin is physically blocked by:
a) Troponin C
b) Titin
c) Tropomyosin
d) Myosin Light Chains
Explanation: Regulation of contraction occurs on the thin filament. In the absence of Calcium (resting state), the filamentous protein Tropomyosin lies directly over the myosin-binding groove on the actin helix. This creates "steric hindrance," physically blocking the myosin heads from attaching to actin. When Calcium is released, it binds to Troponin C, causing a conformational change in the Troponin complex that pulls Tropomyosin aside, uncovering the active sites and allowing cross-bridge formation. Therefore, the correct answer is c) Tropomyosin.
5. The myosin molecule is a hexamer composed of:
a) 1 Heavy chain and 5 Light chains
b) 2 Heavy chains and 4 Light chains
c) 4 Heavy chains and 2 Light chains
d) 6 Heavy chains
Explanation: The myosin II molecule (skeletal muscle myosin) is a large protein complex. It is composed of Two Heavy Chains and Four Light Chains (Total = 6 polypeptide chains). The heavy chains coil around each other to form the "tail" and diverge to form the two globular "heads." Each head is associated with two light chains: one Essential Light Chain (stabilizes the head) and one Regulatory Light Chain (regulates ATPase activity, especially in smooth muscle). Therefore, the correct answer is b) 2 Heavy chains and 4 Light chains.
6. In the length-tension relationship of skeletal muscle, the active tension is maximal when:
a) The muscle is fully stretched (no overlap)
b) The muscle is fully shortened (actin overlap)
c) There is optimal overlap between thick and thin filaments
d) The passive tension is zero
Explanation: The force generated by a muscle fiber is directly proportional to the number of cross-bridges formed between actin and myosin. This depends on the sarcomere length. At Optimal Length (L0) (usually 2.0 - 2.2 micrometers), there is maximal overlap between the myosin heads and actin filaments, allowing the maximum number of cross-bridges to form. If the muscle is stretched too far (no overlap) or shortened too much (actin filaments overlap each other and hit Z-disks), the number of effective cross-bridges decreases, reducing active tension. Therefore, the correct answer is c) There is optimal overlap between thick and thin filaments.
7. Which protein acts as a "molecular ruler" determining the precise length of the actin (thin) filaments during muscle development?
a) Titin
b) Nebulin
c) Desmin
d) Alpha-actinin
Explanation: The sarcomere contains giant accessory proteins that maintain structural integrity. Nebulin is a large, non-elastic protein that runs along the entire length of the thin filament. It is thought to act as a "Molecular Ruler," dictating the precise length of the actin polymer during assembly. In contrast, Titin is the molecular ruler and spring for the thick filament and provides resting elasticity. Alpha-actinin anchors actin to the Z-disk. Desmin connects Z-disks of adjacent myofibrils. Therefore, the correct answer is b) Nebulin.
8. The "S1 fragment" obtained by proteolytic cleavage of myosin contains which functional domains?
a) Tail region for filament assembly
b) Binding sites for Actin and ATP
c) Binding sites for Troponin and Tropomyosin
d) Hinge region only
Explanation: Proteolytic digestion of myosin (using papain or trypsin) yields specific fragments: Light Meromyosin (LMM) and Heavy Meromyosin (HMM). HMM is further split into S1 and S2. The S1 fragment corresponds to the globular Myosin Head. This is the functional "business end" of the molecule. It contains the Binding site for Actin and the ATPase enzymatic pocket (binding site for ATP). The tail (LMM) is for self-assembly into filaments. The S2 is the flexible neck. Therefore, the correct answer is b) Binding sites for Actin and ATP.
9. Which subunit of the Troponin complex mediates the inhibition of the actin-myosin interaction in the absence of calcium?
a) Troponin C
b) Troponin T
c) Troponin I
d) Calmodulin
Explanation: The Troponin complex has three distinct subunits with specific functions. Troponin T (TnT) binds the complex to Tropomyosin. Troponin C (TnC) is the calcium-binding sensor. Troponin I (TnI) is the Inhibitory subunit. In the resting state, TnI binds strongly to actin, holding the tropomyosin in the blocking position and inhibiting ATPase activity. Upon calcium binding to TnC, the interaction between TnI and actin is weakened, allowing tropomyosin to move. TnI levels in blood are markers for cardiac muscle damage. Therefore, the correct answer is c) Troponin I.
10. Smooth muscle myosin differs from skeletal muscle myosin in that its interaction with actin is primarily regulated by:
a) Troponin C binding to Calcium
b) Phosphorylation of the Regulatory Light Chain
c) Direct binding of Calcium to the Heavy chain
d) Removal of Tropomyosin
Explanation: This is a fundamental difference between muscle types. Skeletal muscle is "thin-filament regulated" (Calcium binds Troponin C). Smooth muscle lacks Troponin. Instead, it is "thick-filament regulated." Calcium binds to Calmodulin, and the Ca-Calmodulin complex activates an enzyme called Myosin Light Chain Kinase (MLCK). MLCK then Phosphorylates the Regulatory Light Chain on the myosin head. Only the phosphorylated form of smooth muscle myosin can interact with actin to cause contraction. Therefore, the correct answer is b) Phosphorylation of the Regulatory Light Chain.
Chapter: Head & Neck Anatomy; Topic: Muscles of Mastication; Subtopic: Attachments around Maxilla
Keyword Definitions:
Maxillary Tuberosity: Rounded posterior part of maxilla giving attachment to medial pterygoid.
Medial Pterygoid: Muscle assisting mastication; attaches to tuberosity & lateral pterygoid plate.
Lateral Pterygoid: Protrudes mandible; no attachment to maxillary tuberosity.
Masseter: Inserts on ramus of mandible; no attachment to tuberosity.
Temporalis: Inserts on coronoid process; unrelated to tuberosity.
1) Lead Question – 2016
Maxillary tubercle gives attachment to?
a) Lateral pterygoid
b) Medial pterygoid
c) Temporalis
d) Masseter
Answer: b) Medial pterygoid
Explanation: The medial pterygoid muscle arises from the medial surface of the lateral pterygoid plate and the maxillary tuberosity. This posterior maxillary prominence provides a strong anchor point for this elevator of the mandible. None of the other muscles—masseter, temporalis, or lateral pterygoid—attach to the maxillary tuberosity. Understanding this attachment is clinically relevant in maxillary fractures and posterior maxillary nerve blocks.
2) Fibers of medial pterygoid run in the same direction as:
a) Lateral pterygoid
b) Masseter
c) Buccinator
d) Temporalis
Answer: b) Masseter
Explanation: The medial pterygoid and masseter form a muscular sling elevating the mandible, with nearly parallel fiber orientation.
3) Medial pterygoid is supplied by:
a) Facial nerve
b) Mandibular nerve (V3)
c) Glossopharyngeal nerve
d) Hypoglossal nerve
Answer: b) Mandibular nerve (V3)
Explanation: All muscles of mastication, including the medial pterygoid, receive motor supply from V3.
4) Action of medial pterygoid includes:
a) Depression of mandible
b) Elevation and side-to-side movement
c) Retraction
d) Protraction only
Answer: b) Elevation and side-to-side movement
Explanation: Along with masseter, it elevates the mandible and produces grinding movements.
5) A blowout fracture of posterior maxilla may affect the attachment of:
a) Buccinator
b) Medial pterygoid
c) Temporalis
d) Orbicularis oris
Answer: b) Medial pterygoid
Explanation: The medial pterygoid attaches to the maxillary tuberosity; fractures here affect its function.
6) Which muscle helps in protrusion of mandible?
a) Temporalis
b) Masseter
c) Lateral pterygoid
d) Mylohyoid
Answer: c) Lateral pterygoid
Explanation: The only major muscle that protrudes the mandible is the lateral pterygoid.
7) Which structure lies medial to medial pterygoid?
a) Parotid gland
b) Tensor veli palatini
c) Pharyngeal wall
d) Mandibular ramus
Answer: c) Pharyngeal wall
Explanation: The medial pterygoid forms the lateral boundary of the pharyngeal wall.
8) Medial pterygoid forms part of:
a) Infratemporal fossa
b) Parapharyngeal space
c) Pterygopalatine fossa
d) Nasal cavity
Answer: a) Infratemporal fossa
Explanation: It occupies the infratemporal fossa and contributes to mastication mechanics.
9) Paralysis of medial pterygoid results in:
a) Inability to protrude mandible
b) Deviation of jaw to opposite side
c) Weak elevation of mandible
d) Excessive retraction
Answer: c) Weak elevation of mandible
Explanation: Medial pterygoid is a major elevator; paralysis weakens closure of mouth.
10) Which artery supplies medial pterygoid?
a) Facial artery
b) Inferior alveolar artery
c) Maxillary artery
d) Lingual artery
Answer: c) Maxillary artery
Explanation: Branches of the maxillary artery supply muscles of mastication.
11) Medial pterygoid is separated from ramus of mandible by:
a) Pterygomandibular raphe
b) Parotid duct
c) Mylohyoid line
d) Buccal fat pad
Answer: a) Pterygomandibular raphe
Explanation: The raphe forms an important landmark between buccinator and superior constrictor, lying close to the medial pterygoid.
Chapter: Head & Neck; Topic: Temporomandibular Joint (TMJ); Subtopic: Muscular Attachments of TMJ
Keyword Definitions:
TMJ (Temporomandibular Joint): A synovial joint between mandible & temporal bone.
Articular Disc: Fibrocartilaginous structure dividing TMJ into upper & lower compartments.
Lateral Pterygoid Muscle: Major muscle inserting into TMJ disc & neck of mandible.
Masseter: Elevator of mandible, no attachment to disc.
Temporalis: Inserts into coronoid process, elevates/retracts mandible.
Buccinator: Muscle of cheek, unrelated to TMJ movement.
1) Lead Question – 2016
Which muscle is attached to the disc of the temporomandibular joint?
a) Buccinator
b) Lateral pterygoid
c) Masseter
d) Temporalis
Answer: b) Lateral pterygoid
Explanation: The lateral pterygoid muscle, specifically its superior head, inserts into the articular disc and capsule of the TMJ. This attachment allows the muscle to control anterior movement of the disc during mouth opening. Other muscles like masseter and temporalis act on the mandible but do not attach to the disc. Buccinator plays no role in TMJ mechanics. This unique attachment explains why disc displacement is commonly associated with lateral pterygoid hyperactivity.
2) Which muscle primarily assists in protrusion of the mandible?
a) Medial pterygoid
b) Lateral pterygoid
c) Masseter
d) Temporalis
Answer: b) Lateral pterygoid
Explanation: The lateral pterygoid is the only muscle that pulls the mandibular condyle and disc forward, producing protrusion. Medial pterygoid assists slightly but is mainly an elevator. Temporalis retracts, and masseter mainly elevates the mandible.
3) A patient with TMJ clicking likely has dysfunction involving:
a) Masseter
b) Lateral pterygoid
c) Mylohyoid
d) Stylohyoid
Answer: b) Lateral pterygoid
Explanation: Clicking occurs when the articular disc displaces anteriorly due to imbalance or hyperactivity of the lateral pterygoid, which is attached to the disc.
4) Depression (opening) of the mouth is initiated mainly by:
a) Gravity
b) Lateral pterygoid
c) Masseter
d) Buccinator
Answer: a) Gravity
Explanation: Initial mouth opening is passive due to gravity; full opening requires contraction of lateral pterygoid.
5) Which muscle assists in retraction of the mandible?
a) Posterior fibers of temporalis
b) Buccinator
c) Medial pterygoid
d) Lateral pterygoid
Answer: a) Posterior fibers of temporalis
Explanation: Posterior temporalis fibers pull the mandible backward; pterygoids protrude instead.
6) TMJ disc is primarily composed of:
a) Hyaline cartilage
b) Fibrocartilage
c) Elastic cartilage
d) Keratinized cartilage
Answer: b) Fibrocartilage
Explanation: Unlike most synovial joints, TMJ has a fibrocartilaginous disc, allowing durability during mastication.
7) Injury to which nerve affects TMJ movements?
a) Auriculotemporal nerve
b) Facial nerve
c) Hypoglossal nerve
d) Spinal accessory nerve
Answer: a) Auriculotemporal nerve
Explanation: Auriculotemporal nerve (branch of V3) supplies sensory innervation to TMJ; V3 supplies muscles of mastication including lateral pterygoid.
8) Which artery supplies TMJ?
a) Maxillary artery
b) Facial artery
c) Lingual artery
d) Superior thyroid artery
Answer: a) Maxillary artery
Explanation: TMJ receives blood mainly from maxillary artery branches, including deep auricular artery.
9) During wide mouth opening, the condyle moves:
a) Upward
b) Downward and forward
c) Backward
d) None
Answer: b) Downward and forward
Explanation: The lateral pterygoid pulls the condyle and disc forward onto the articular eminence.
10) Teeth clenching involves strongest contraction of:
a) Masseter
b) Buccinator
c) Styloglossus
d) Superior constrictor
Answer: a) Masseter
Explanation: Masseter is the strongest muscle of mastication, responsible for forceful elevation of the jaw.
11) Pain in the preauricular region during chewing suggests pathology of:
a) TMJ
b) Stylomastoid foramen
c) Hypoglossal canal
d) Nasal cavity
Answer: a) TMJ
Explanation: TMJ dysfunction presents with preauricular pain, clicking, limited jaw movement, or deviation to one side.
Chapter: Anterior Abdominal Wall & Inguinal Canal; Topic: Cremaster Muscle; Subtopic: Fascia & Muscular Derivatives of the Inguinal Region
Keyword Definitions:
Cremaster Muscle: Thin muscle layer covering spermatic cord; elevates testes.
Internal Oblique: Middle layer abdominal muscle contributing to spermatic cord coverings.
Spermatic Cord: Cord-like structure carrying vas deferens, vessels, and nerves.
Cremasteric Reflex: Reflex elevation of testes mediated by genital branch of genitofemoral nerve.
Inguinal Canal: Oblique passage in lower abdominal wall transmitting spermatic cord or round ligament.
1) Lead Question – 2016
Cremastric muscle is formed from?
a) Fascia from internal oblique
b) Fascia from external oblique
c) Fascia from rectus abdominis
d) Fascia from transversus abdominis
Answer: a) Fascia from internal oblique
Explanation: The cremaster muscle and cremasteric fascia arise from fibers of the internal oblique muscle. As the testes descend through the inguinal canal, a portion of the internal oblique is drawn into the spermatic cord to form these structures. The external oblique forms the external spermatic fascia, whereas the transversus abdominis contributes no muscle to the cord because it arches above it. Hence, the cremaster is solely derived from internal oblique muscle fibers.
2) Cremasteric reflex afferent limb is via?
a) Ilioinguinal nerve
b) Genital branch of genitofemoral nerve
c) Femoral branch of genitofemoral nerve
d) Pudendal nerve
Answer: a) Ilioinguinal nerve
Explanation: The afferent sensory limb arises from the ilioinguinal nerve, while efferent is via genital branch of genitofemoral nerve.
3) Cremasteric reflex is absent in?
a) Upper motor neuron lesion
b) L1 nerve root injury
c) Hypothyroidism
d) Cauda equina lesion
Answer: b) L1 nerve root injury
Explanation: Reflex depends on L1–L2 segment integrity; L1 damage abolishes it.
4) Which is a derivative of external oblique aponeurosis?
a) External spermatic fascia
b) Cremaster muscle
c) Conjoint tendon
d) Tunica vaginalis
Answer: a) External spermatic fascia
Explanation: The external spermatic fascia arises from the external oblique aponeurosis.
5) Transversus abdominis contributes to spermatic cord covering as?
a) Cremaster muscle
b) Conjoint tendon
c) Transversalis fascia
d) It contributes nothing
Answer: d) It contributes nothing
Explanation: The transversus abdominis arches above the canal and gives no covering.
6) Injury to genital branch of genitofemoral nerve results in?
a) Absent cremasteric reflex
b) Loss of thigh adduction
c) Loss of scrotal sensation
d) Foot drop
Answer: a) Absent cremasteric reflex
Explanation: This branch provides the motor supply to the cremaster muscle.
7) Dartos muscle is derived from?
a) Smooth muscle of scrotal skin
b) Internal oblique
c) Transversalis fascia
d) External oblique
Answer: a) Smooth muscle of scrotal skin
Explanation: Dartos is a thin smooth-Muscle layer important for thermoregulation.
8) Deep inguinal ring is an opening in?
a) External oblique aponeurosis
b) Transversalis fascia
c) Internal oblique
d) Scarpa’s fascia
Answer: b) Transversalis fascia
Explanation: The deep ring is a defect in the transversalis fascia near midpoint of inguinal ligament.
9) Conjoint tendon is formed by fusion of?
a) External and internal oblique
b) Internal oblique and transversus abdominis
c) Rectus abdominis and external oblique
d) Transversalis fascia and internal oblique
Answer: b) Internal oblique and transversus abdominis
Explanation: Conjoint tendon supports the posterior wall of inguinal canal.
10) Covering of spermatic cord that originates from transversalis fascia?
a) Cremasteric fascia
b) Dartos muscle
c) Internal spermatic fascia
d) External spermatic fascia
Answer: c) Internal spermatic fascia
Explanation: This fascia lies deepest and arises from transversalis fascia.
11) Main action of cremaster muscle?
a) Elevation of testes
b) Depression of testes
c) Regulation of penile erection
d) Scrotal skin tightening
Answer: a) Elevation of testes
Explanation: Cremaster elevates testes during cold exposure or protective reflex.
Chapter: Anterior Abdominal Wall; Topic: Rectus Sheath; Subtopic: Anatomy & Applied Anatomy
Keyword Definitions:
Rectus sheath: Fibrous sheath enclosing rectus abdominis formed by abdominal muscle aponeuroses.
Arcuate line: A landmark below which all three aponeuroses pass anterior to rectus abdominis.
External oblique aponeurosis: Most superficial contributor to the anterior sheath throughout.
Internal oblique aponeurosis: Splits above arcuate line but passes fully anterior below it.
Transversus abdominis aponeurosis: Contributes posteriorly above arcuate line but fully anterior below it.
1) Lead Question – 2016
Anterior Rectus Sheath just above pubic symphysis is formed by ?
a) External Oblique Aponeurosis
b) The aponeurosis of three muscles including External Oblique, Internal Oblique, and Transversus Abdominis
c) Linea Alba
d) Internal Oblique only
Answer: b) The aponeurosis of three muscles including External Oblique, Internal Oblique, and Transversus Abdominis
Explanation: Below the arcuate line and particularly just above the pubic symphysis, the **anterior rectus sheath is formed by the aponeuroses of all three lateral abdominal muscles**—external oblique, internal oblique, and transversus abdominis. These fuse anteriorly, leaving no posterior sheath in this region. The linea alba is a midline fibrous structure, not the sheath itself. Internal oblique alone never forms the complete anterior sheath here. Therefore, the correct answer is the contribution of all three aponeuroses.
2) Above the arcuate line, the posterior rectus sheath is formed by:
a) External oblique only
b) Internal oblique and transversus abdominis
c) Transversus abdominis only
d) No muscle aponeurosis
Answer: b) Internal oblique and transversus abdominis
Explanation: Above the arcuate line, the internal oblique splits into anterior and posterior laminae; the posterior lamina combines with the aponeurosis of transversus abdominis to form the posterior rectus sheath.
3) Below the arcuate line, the posterior rectus sheath:
a) Becomes thicker
b) Is absent
c) Is formed by external oblique
d) Contains transversalis fascia
Answer: b) Is absent
Explanation: All three aponeuroses pass anteriorly below the arcuate line, so only transversalis fascia lies posterior to rectus abdominis.
4) A patient has a Spigelian hernia. Which layer is primarily defective?
a) External oblique
b) Internal oblique
c) Transversus abdominis
d) Transversalis fascia
Answer: c) Transversus abdominis
Explanation: Spigelian hernias occur along the semilunar line where transversus abdominis aponeurosis is weak or deficient.
5) The arcuate line corresponds to which vertebral level approximately?
a) L1
b) L2
c) S1
d) T12
Answer: a) L1
Explanation: The arcuate line is usually at the level of L1, marking the transition in sheath composition.
6) Which muscle lies inside the rectus sheath?
a) Pyramidalis
b) Transversus abdominis
c) Internal oblique
d) External oblique
Answer: a) Pyramidalis
Explanation: Pyramidalis lies anterior to rectus abdominis and inside the rectus sheath, inserting into linea alba.
7) A penetrating injury below the arcuate line risks damage to:
a) Superior epigastric artery
b) Inferior epigastric artery
c) Musculophrenic artery
d) Intercostal arteries
Answer: b) Inferior epigastric artery
Explanation: Inferior epigastric vessels ascend posterior to rectus until below the arcuate line where the sheath changes.
8) Linea alba is formed by fusion of:
a) Only external oblique
b) All three abdominal muscle aponeuroses
c) Rectus abdominis
d) Transversalis fascia
Answer: b) All three abdominal muscle aponeuroses
Explanation: The linea alba results from midline fusion of aponeuroses of external oblique, internal oblique, and transversus abdominis.
9) Rectus abdominis originates from:
a) Pubic crest
b) Xiphoid process
c) Costal margin
d) Iliac crest
Answer: a) Pubic crest
Explanation: Rectus abdominis arises from the pubic crest and symphysis and inserts into the 5th–7th costal cartilages.
10) Which structure passes between the layers of the rectus sheath?
a) Thoracodorsal vessels
b) Superior epigastric vessels
c) Short gastric arteries
d) Pudendal nerve
Answer: b) Superior epigastric vessels
Explanation: Superior epigastric artery descends within the sheath supplying the upper rectus region.
11) The semilunar line marks the lateral border of:
a) Transversus abdominis
b) External oblique
c) Rectus abdominis
d) Psoas major
Answer: c) Rectus abdominis
Explanation: The semilunar line is a curved tendinous margin forming the lateral border of the rectus abdominis muscle.
Chapter: Back & Thorax; Topic: Thoracolumbar Fascia; Subtopic: Muscular Relations
Keyword Definitions:
Thoracolumbar fascia: A multilayered fascial complex in the lumbar region enclosing deep back and abdominal wall muscles.
Anterior layer: The deepest layer, covering quadratus lumborum anteriorly.
Middle layer: Lies posterior to quadratus lumborum, separating it from erector spinae.
Quadratus lumborum: Muscle extending from iliac crest to 12th rib, stabilizing the spine.
Psoas major: Hip flexor lying more medially, not between the TL fascia layers.
1) Lead Question – 2016
Muscle lying between anterior and middle layer of thoracolumbar fascia is ?
a) Psoas major
b) Quadratus Lumborum
c) Obdurator internus
d) External oblique
Answer: b) Quadratus lumborum
Explanation: The thoracolumbar fascia has three layers: anterior, middle, and posterior. The **quadratus lumborum** is located between the anterior and middle layers. The anterior layer covers the muscle from the front, while the middle layer lies posterior to it and separates it from the erector spinae group. Psoas major lies medial to these layers, and external oblique is superficial and not part of the fascial compartment. Thus, only quadratus lumborum fits the anatomical location precisely.
2) Which layer of thoracolumbar fascia forms the lateral raphe?
a) Anterior
b) Middle
c) Posterior
d) Deep investing fascia
Answer: c) Posterior
Explanation: The posterior layer thickens laterally to form the lateral raphe where abdominal wall muscles attach.
3) Quadratus lumborum receives nerve supply from–
a) Subcostal nerve
b) Ilioinguinal nerve
c) Genitofemoral nerve
d) L5 root
Answer: a) Subcostal nerve
Explanation: QL is supplied by T12 (subcostal nerve) and L1–L4 branches of lumbar plexus.
4) A patient with lateral bending weakness likely has injury to:
a) Quadratus lumborum
b) Psoas minor
c) Piriformis
d) Gluteus minimus
Answer: a) Quadratus lumborum
Explanation: QL is the primary lumbar lateral flexor.
5) Middle layer of thoracolumbar fascia attaches to:
a) Spinous processes
b) Transverse processes
c) Iliac crest only
d) Coccyx
Answer: b) Transverse processes
Explanation: The middle layer attaches to lumbar transverse processes and separates deep muscle groups.
6) Posterior layer of thoracolumbar fascia encloses:
a) Psoas major
b) Erector spinae
c) Quadratus lumborum
d) Rectus abdominis
Answer: b) Erector spinae
Explanation: The posterior layer surrounds the erector spinae mass completely.
7) Which muscle stabilizes 12th rib during inspiration?
a) External oblique
b) Quadratus lumborum
c) Transversus abdominis
d) Internal oblique
Answer: b) Quadratus lumborum
Explanation: QL fixes the 12th rib allowing diaphragm to act efficiently.
8) Thoracolumbar fascia contributes to origin of:
a) External oblique
b) Internal oblique
c) Rectus abdominis
d) Quadratus lumborum
Answer: b) Internal oblique
Explanation: Internal oblique and transversus abdominis partly arise from thoracolumbar fascia.
9) Psoas major lies anterior to which fascial layer?
a) Anterior
b) Middle
c) Posterior
d) Transversalis fascia
Answer: a) Anterior
Explanation: Psoas major is medial and anterior to the TL fascia system.
10) Injury to thoracolumbar fascia commonly presents with:
a) Pain on shoulder abduction
b) Low-back pain
c) Knee instability
d) Ankle inversion weakness
Answer: b) Low-back pain
Explanation: TL fascia stabilizes lumbar spine; damage causes chronic low-back strain.
11) Which muscle does *not* attach to thoracolumbar fascia?
a) Latissimus dorsi
b) Internal oblique
c) Transversus abdominis
d) Pectoralis minor
Answer: d) Pectoralis minor
Explanation: Pectoralis minor is a thoracic muscle unrelated to TL fascia.
Chapter: Lower Limb Anatomy; Topic: Leg Muscles – Anterior Compartment; Subtopic: Tibialis Anterior – Actions, Nerve Supply, Relations
Keyword Definitions:
Tibialis Anterior (TA): Major dorsiflexor and inverter of the foot from anterior compartment.
Deep Peroneal Nerve: Supplies all anterior compartment muscles including TA.
Anterior Tibial Vessels: Artery and vein running deep to tibialis anterior throughout leg.
Medial Cuneiform: One of TA’s insertion sites along with base of 1st metatarsal.
Dorsiflexion: Upward movement of the foot at the ankle joint.
1) Lead Question – 2016
All of the following are true about tibialis anterior except?
A) It is supplied by the superficial peroneal nerve
B) It dorsiflexes the foot
C) It is closely related to the anterior tibial vessels
D) It inserts on the medial cuneiform
Answer: A) It is supplied by the superficial peroneal nerve
Explanation: Tibialis anterior is innervated by the deep peroneal nerve, not the superficial peroneal nerve, making option A false. It is the primary dorsiflexor of the foot and also assists in inversion. It lies in close anatomical relation to the anterior tibial vessels, which run deep to it along the anterior compartment. Its insertion is on the medial cuneiform and base of the first metatarsal. Thus, all statements except A are true.
2) Tibialis anterior also contributes to–
A) Foot eversion
B) Foot inversion
C) Toe extension
D) Knee flexion
Answer: B) Foot inversion
Explanation: TA combines dorsiflexion with inversion due to its medial insertion. Thus, B is correct.
3) Deep peroneal nerve supplies all except–
A) Tibialis anterior
B) EHL
C) EDL
D) Peroneus longus
Answer: D) Peroneus longus
Explanation: Peroneus longus is supplied by superficial peroneal nerve. Thus, D is correct.
4) A patient with deep peroneal nerve palsy will have–
A) Inability to plantarflex
B) Inability to dorsiflex
C) Inability to evert
D) Inability to flex toes
Answer: B) Inability to dorsiflex
Explanation: Deep peroneal nerve supplies all dorsiflexors. Thus, B is correct.
5) In tibialis anterior tendinopathy, pain is felt over–
A) Lateral malleolus
B) Medial cuneiform
C) Calcaneus
D) Navicular tuberosity
Answer: B) Medial cuneiform
Explanation: TA inserts on medial cuneiform/base of 1st metatarsal. Thus, B is correct.
6) The artery accompanying tibialis anterior is–
A) Posterior tibial artery
B) Anterior tibial artery
C) Fibular artery
D) Medial plantar artery
Answer: B) Anterior tibial artery
Explanation: Anterior tibial artery travels deep to TA. Thus, B is correct.
7) Dorsalis pedis artery is continuation of–
A) Posterior tibial artery
B) Anterior tibial artery
C) Fibular artery
D) Popliteal artery
Answer: B) Anterior tibial artery
Explanation: ATA continues as dorsalis pedis at ankle. Thus, B is correct.
8) TA originates mainly from–
A) Fibula
B) Tibial lateral surface
C) Tibial medial surface
D) Calcaneus
Answer: B) Tibial lateral surface
Explanation: It arises from lateral tibia and interosseous membrane. Thus, B is correct.
9) A runner with foot drop has weakness of–
A) Gastrocnemius
B) Tibialis anterior
C) Tibialis posterior
D) Soleus
Answer: B) Tibialis anterior
Explanation: TA is primary dorsiflexor; weakness leads to foot drop. Thus, B is correct.
10) Superficial peroneal nerve injury affects–
A) TA
B) EHL
C) Foot eversion
D) Foot inversion
Answer: C) Foot eversion
Explanation: SPN supplies peroneus longus/brevis—main everters. Thus, C is correct.
11) Tight TA may lead to–
A) Pes cavus
B) Excessive dorsiflexion
C) Toe flexion
D) Decreased inversion
Answer: B) Excessive dorsiflexion
Explanation: Overactive TA increases dorsiflexion force at ankle. Thus, B is correct.
Chapter: Lower Limb Anatomy; Topic: Foot Arches; Subtopic: Medial & Lateral Longitudinal Arches
Keyword Definitions:
Medial Longitudinal Arch: Higher arch of foot supported by spring ligament and intrinsic muscles.
Lateral Longitudinal Arch: Flatter arch supported mainly by plantar fascia and muscle tendons.
Plantar Fascia: Thick fibrous structure forming primary support for both arches.
Spring Ligament: Supports medial arch only, connecting calcaneus to navicular.
Intrinsic Foot Muscles: Small muscles contributing to arch stabilization.
1) Lead Question – 2016
Which of the following is common between the medial and lateral plantar arch?
A) Flexor Digitorum Brevis
B) Plantar Fascia
C) Spring Ligament
D) Deltoid Ligament
Answer: B) Plantar Fascia
Explanation: The plantar fascia, also called the plantar aponeurosis, is a strong fibrous structure extending from the calcaneus to the toes. It provides essential static support to both the medial and lateral longitudinal arches by maintaining tension during weight-bearing. Flexor digitorum brevis contributes mainly to the medial arch. Spring ligament supports only the medial arch by holding the talar head. Deltoid ligament stabilizes the ankle joint and does not participate in arch maintenance. Therefore, the only structure common to both medial and lateral arches is the plantar fascia.
2) The spring ligament connects:
A) Talus to calcaneus
B) Calcaneus to navicular
C) Cuboid to navicular
D) Talus to navicular
Answer: B) Calcaneus to navicular
Explanation: The spring ligament forms the key static support of medial arch. Thus, B is correct.
3) Primary dynamic support of the medial arch is from–
A) Tibialis posterior
B) Fibularis brevis
C) Tibialis anterior
D) Gastrocnemius
Answer: A) Tibialis posterior
Explanation: Tibialis posterior maintains medial arch height. Thus, A is correct.
4) Lateral arch mainly receives support from–
A) Flexor hallucis longus
B) Fibularis longus
C) Tibialis anterior
D) Soleus
Answer: B) Fibularis longus
Explanation: Fibularis longus tendon forms sling under the foot supporting lateral arch. Thus, B is correct.
5) A patient with flat foot likely has dysfunction of–
A) Tibialis posterior
B) Fibularis brevis
C) Flexor digitorum longus
D) Gastrocnemius
Answer: A) Tibialis posterior
Explanation: Posterior tibial tendon failure leads to arch collapse. Thus, A is correct.
6) Plantar aponeurosis originates from–
A) Metatarsal heads
B) Calcaneal tuberosity
C) Navicular
D) Talus
Answer: B) Calcaneal tuberosity
Explanation: It arises from calcaneus and extends to toes supporting both arches. Thus, B is correct.
7) Medial arch bones include all except–
A) Talus
B) Navicular
C) Cuboid
D) Cuneiforms
Answer: C) Cuboid
Explanation: Cuboid belongs to lateral arch. Thus, C is correct.
8) Lateral arch includes–
A) Talus
B) Cuboid
C) Navicular
D) All cuneiforms
Answer: B) Cuboid
Explanation: Cuboid and calcaneus form the lateral arch. Thus, B is correct.
9) Failure of plantar fascia results in–
A) Cavus foot
B) Flattened arches
C) Ankle valgus
D) Toe deformities only
Answer: B) Flattened arches
Explanation: Plantar fascia is essential for both arches; rupture causes collapse. Thus, B is correct.
10) Weight transfer during stance passes mainly through–
A) Cuboid
B) Talus
C) Navicular
D) Sesamoids
Answer: B) Talus
Explanation: Talus receives body weight and transmits to arches. Thus, B is correct.
11) In pes cavus, which structure is typically tight?
A) Tibialis posterior
B) Plantar fascia
C) Spring ligament
D) ACL
Answer: B) Plantar fascia
Explanation: Excessive arch height is linked with tight plantar fascia. Thus, B is correct.
Chapter: Upper Limb Anatomy; Topic: Forearm Muscles; Subtopic: Wrist Flexors – Insertions
Keyword Definitions:
Flexor Carpi Radialis (FCR): A wrist flexor and radial deviator inserting on bases of 2nd & 3rd metacarpals.
Metacarpal Base: Proximal part of metacarpal bone where several forearm muscles insert.
Radial Deviation: Movement of wrist toward thumb side produced by FCR.
Common Flexor Origin: Medial epicondyle giving rise to superficial flexors including FCR.
Carpal Bones: Eight small bones of wrist not involved in FCR insertion.
1) Lead Question – 2016
Flexor carpi radialis inserts into?
A) Base of 5th metatarsal
B) Base of 2nd and 3rd metacarpal
C) Scaphoid and trapezium
D) Capitate and hamate
Answer: B) Base of 2nd and 3rd metacarpal
Explanation: Flexor carpi radialis originates from the medial epicondyle of the humerus and travels along the anterior forearm. It inserts primarily on the base of the 2nd metacarpal with occasional slips to the 3rd metacarpal. This insertion enables it to flex the wrist and produce radial deviation. It does not insert on carpal bones like scaphoid or trapezium, nor on the 5th metatarsal, which belongs to the lower limb. Therefore, the correct answer is B.
2) Flexor carpi ulnaris inserts into–
A) Pisiform
B) Trapezoid
C) 2nd metacarpal
D) Capitate
Answer: A) Pisiform
Explanation: FCU inserts into pisiform and via ligaments to hamate and 5th metacarpal. Thus, A is correct.
3) FCR is innervated by–
A) Ulnar nerve
B) Median nerve
C) Radial nerve
D) Axillary nerve
Answer: B) Median nerve
Explanation: FCR is a median-nerve–supplied forearm flexor. Thus, B is correct.
4) The major action of FCR is–
A) Wrist flexion and radial deviation
B) Wrist extension
C) Finger abduction
D) Thumb extension
Answer: A) Wrist flexion and radial deviation
Explanation: FCR flexes wrist and pulls it radially. Thus, A is correct.
5) Which structure passes through FCR tunnel in flexor retinaculum?
A) FCR tendon
B) Median nerve
C) Ulnar nerve
D) FDP tendon
Answer: A) FCR tendon
Explanation: FCR has a separate fibro-osseous tunnel lateral to carpal tunnel. Thus, A is correct.
6) Pain over 2nd metacarpal base with wrist flexion suggests strain of–
A) FCU
B) FCR
C) ECRB
D) EPL
Answer: B) FCR
Explanation: FCR inserts on 2nd/3rd metacarpal; strain causes pain there. Thus, B is correct.
7) FCR originates from–
A) Lateral epicondyle
B) Medial epicondyle
C) Radius
D) Ulna
Answer: B) Medial epicondyle
Explanation: All superficial forearm flexors arise from medial epicondyle. Thus, B is correct.
8) FCR tendon is used for tendon transfer in–
A) Radial nerve palsy
B) Median nerve palsy
C) Ulnar nerve palsy
D) Axillary nerve palsy
Answer: A) Radial nerve palsy
Explanation: FCR can be transferred to restore wrist extension. Thus, A is correct.
9) The artery running close to FCR tendon at wrist is–
A) Ulnar artery
B) Radial artery
C) Anterior interosseous
D) Posterior interosseous
Answer: B) Radial artery
Explanation: Radial artery lies just lateral to FCR tendon, used for pulse palpation. Thus, B is correct.
10) The flexor retinaculum attaches laterally to–
A) Pisiform
B) Hook of hamate
C) Scaphoid tubercle
D) Ulna
Answer: C) Scaphoid tubercle
Explanation: Lateral attachments include scaphoid and trapezium. Thus, C is correct.
11) Wrist flexion is strongest when combined with–
A) Ulnar deviation
B) Radial deviation
C) Supination
D) Pronation
Answer: A) Ulnar deviation
Explanation: FCU is strongest flexor; combining ulnar deviation increases flexion force. Thus, A is correct.
Chapter: Upper Limb Anatomy; Topic: Shoulder Girdle Muscles; Subtopic: Muscles Used in Climbing
Keyword Definitions:
Latissimus Dorsi: Large muscle of the back responsible for powerful extension, adduction, and internal rotation of the humerus.
Rhomboideus: Muscle that retracts and stabilizes the scapula.
Trapezius: Muscle responsible for elevation, rotation, and retraction of the scapula.
Levator Scapulae: Elevates scapula and assists in downward rotation.
Climbing Musculature: Combination of shoulder extensors and adductors enabling pulling movements.
1) Lead Question – 2016
Which muscle helps in climbing a tree?
A) Latissimus dorsi
B) Rhomboideus
C) Trapezius
D) Levator scapulae
Answer: A) Latissimus dorsi
Explanation: Latissimus dorsi is the principal muscle used during climbing because it produces powerful extension, adduction, and internal rotation of the humerus. These actions bring the body upward by pulling the trunk toward the upper limb. It also stabilizes the posterior axillary fold and contributes strongly during activities such as rowing, swimming, and climbing. Rhomboids and levator scapulae help stabilize the scapula, while trapezius aids in scapular motion but does not generate the primary pull required for climbing. Therefore, the correct answer is A.
2) Latissimus dorsi is innervated by–
A) Axillary nerve
B) Thoracodorsal nerve
C) Dorsal scapular nerve
D) Long thoracic nerve
Answer: B) Thoracodorsal nerve
Explanation: Thoracodorsal nerve (branch of posterior cord) supplies latissimus dorsi, enabling climbing actions. Thus, B is correct.
3) A patient unable to retract the scapula likely has dysfunction of–
A) Rhomboids
B) Latissimus dorsi
C) Pectoralis minor
D) Serratus anterior
Answer: A) Rhomboids
Explanation: Rhomboids retract and stabilize the scapula. Injury impairs retraction. Thus, A is correct.
4) Trapezius paralysis leads to–
A) Winged scapula
B) Loss of scapular elevation
C) Loss of humeral extension
D) Weak elbow flexion
Answer: B) Loss of scapular elevation
Explanation: Trapezius elevates and rotates scapula. Injury causes drooping. Thus, B is correct.
5) The primary muscle for overhead abduction beyond 90° is–
A) Deltoid
B) Latissimus dorsi
C) Trapezius
D) Rhomboids
Answer: C) Trapezius
Explanation: Trapezius upwardly rotates scapula during overhead abduction. Thus, C is correct.
6) Levator scapulae is supplied mainly by–
A) Spinal accessory nerve
B) Dorsal scapular nerve
C) Thoracodorsal nerve
D) Suprascapular nerve
Answer: B) Dorsal scapular nerve
Explanation: Levator scapulae receives dorsal scapular nerve with C3–C4. Thus, B is correct.
7) During climbing, the latissimus dorsi synergizes with–
A) Deltoid
B) Teres major
C) Supraspinatus
D) Infraspinatus
Answer: B) Teres major
Explanation: Teres major assists latissimus dorsi in extension and adduction. Thus, B is correct.
8) Loss of thoracodorsal nerve results in inability to–
A) Elevate scapula
B) Extend humerus
C) Externally rotate humerus
D) Initiate abduction
Answer: B) Extend humerus
Explanation: Thoracodorsal nerve innervates latissimus dorsi which extends humerus. Thus, B is correct.
9) A rock climber with difficulty pulling upward likely has weakness of–
A) Latissimus dorsi
B) Supraspinatus
C) Biceps brachii
D) Trapezius
Answer: A) Latissimus dorsi
Explanation: Pulling the trunk upward relies heavily on latissimus dorsi. Thus, A is correct.
10) Teres major inserts on–
A) Intertubercular groove (medial lip)
B) Greater tuberosity
C) Lesser tuberosity
D) Deltoid tuberosity
Answer: A) Intertubercular groove (medial lip)
Explanation: Teres major inserts on medial lip aiding extension and adduction. Thus, A is correct.
11) A patient with difficulty hiking body upward while holding overhead bars likely has injury to–
A) Radial nerve
B) Thoracodorsal nerve
C) Axillary nerve
D) Suprascapular nerve
Answer: B) Thoracodorsal nerve
Explanation: This nerve supplies latissimus dorsi, essential for climbing. Thus, B is correct.
Chapter: Upper Limb Anatomy; Topic: Muscles of the Arm; Subtopic: Actions of Posterior Compartment Muscles
Keyword Definitions:
Anconeus: A small triangular muscle located at the posterolateral elbow assisting triceps in extension.
Elbow Extensors: Muscles primarily responsible for extension, mainly triceps brachii.
Accessory Muscles: Muscles that support but do not perform the primary action.
Posterior Arm Compartment: Contains triceps and anconeus, both innervated by radial nerve.
Stabilization Role: Anconeus helps stabilize the elbow joint during forearm movements.
1) Lead Question – 2016
What is the action of anconeus?
A) Primary elbow extensor
B) Assists extension of elbow
C) Wrist extension
D) Thumb abduction
Answer: B) Assists extension of elbow
Explanation: The anconeus muscle is a small, triangular muscle situated near the lateral epicondyle of the humerus. It contributes to elbow extension but is not the primary extensor; that role belongs to the triceps brachii. Instead, anconeus acts as a synergist assisting triceps in extension, stabilizing the elbow joint during pronation-supination, and helping in maintaining joint alignment. It does not contribute to wrist extension or thumb movements. Therefore, the correct answer is B.
2) Anconeus is innervated by–
A) Median nerve
B) Ulnar nerve
C) Radial nerve
D) Axillary nerve
Answer: C) Radial nerve
Explanation: Anconeus receives motor supply from the radial nerve, similar to triceps, reinforcing its role in elbow extension. Thus, C is correct.
3) Primary extensor of the elbow is–
A) Brachialis
B) Triceps brachii
C) Anconeus
D) Biceps brachii
Answer: B) Triceps brachii
Explanation: Triceps is the main extensor of the elbow, with anconeus acting only as an accessory muscle. Thus, B is correct.
4) Anconeus contributes to which additional function?
A) Wrist flexion
B) Elbow joint stabilization
C) Thumb opposition
D) Shoulder internal rotation
Answer: B) Elbow joint stabilization
Explanation: Anconeus stabilizes the elbow during pronation-supination. Thus, B is correct.
5) Anconeus originates from–
A) Olecranon
B) Lateral epicondyle
C) Medial epicondyle
D) Radial tuberosity
Answer: B) Lateral epicondyle
Explanation: It arises from the posterior surface of the lateral epicondyle. Thus, B is correct.
6) A patient has weak elbow extension but normal triceps strength. Which muscle is likely affected?
A) Biceps
B) Brachialis
C) Anconeus
D) Supinator
Answer: C) Anconeus
Explanation: If triceps is intact but extension is weak in terminal range, anconeus involvement is suspected. Thus, C is correct.
7) Anconeus inserts on–
A) Coronoid process
B) Olecranon and upper ulna
C) Radial head
D) Ulnar styloid
Answer: B) Olecranon and upper ulna
Explanation: Anconeus fibers attach to lateral olecranon and posterior ulna. Thus, B is correct.
8) Which movement would remain unchanged if anconeus is paralyzed?
A) Elbow extension strength
B) Elbow stability
C) Wrist extension
D) Terminal locking of elbow
Answer: C) Wrist extension
Explanation: Wrist extension is radial-nerve mediated but unrelated to anconeus. Thus, C is correct.
9) Anconeus assists which muscle during extension?
A) Brachioradialis
B) Triceps brachii
C) Deltoid
D) Latissimus dorsi
Answer: B) Triceps brachii
Explanation: Anconeus works synergistically with triceps. Thus, B is correct.
10) Which nerve root contributes to anconeus innervation?
A) C5
B) C6
C) C8
D) T1
Answer: C) C8
Explanation: Radial nerve branches supplying anconeus commonly derive from C7–C8 fibers. Thus, C is correct.
11) Pain at the posterolateral elbow with resisted extension may indicate injury to–
A) Anconeus
B) Pronator teres
C) Palmaris longus
D) FCR
Answer: A) Anconeus
Explanation: Localized pain at lateral elbow with extension implicates anconeus strain. Thus, A is correct.
Chapter: Upper Limb Anatomy; Topic: Scapular Musculature; Subtopic: Fossae of Scapula & Their Muscles
Keyword Definitions:
Infraspinous Fossa: Large depression below the spine of the scapula housing the infraspinatus muscle.
Infraspinatus: A rotator cuff muscle responsible for lateral rotation of the arm.
Supraspinous Fossa: Upper scapular depression containing supraspinatus muscle.
Subscapular Fossa: Anterior scapular surface containing subscapularis muscle.
Teres Major: Muscle located inferiorly, not in the infraspinous fossa.
1) Lead Question – 2016
Infraspinous fossa of scapula contains which of the following muscles?
A) Subscapularis
B) Infraspinatus
C) Teres major
D) Supraspinatus
Answer: B) Infraspinatus
Explanation: The infraspinous fossa is the large posterior surface of the scapula located below the spine. It exclusively contains the infraspinatus muscle, one of the rotator cuff muscles responsible for lateral rotation of the shoulder. The supraspinatus muscle lies above the spine in the supraspinous fossa. Subscapularis occupies the anterior subscapular fossa, and teres major lies inferior to the infraspinous fossa, not within it. Therefore, the correct answer is B. Understanding scapular fossae and their associated muscles is essential for interpreting shoulder injuries and muscle actions.
2) Supraspinatus originates from–
A) Infraspinous fossa
B) Supraspinous fossa
C) Subscapular fossa
D) Glenoid cavity
Answer: B) Supraspinous fossa
Explanation: Supraspinatus arises from supraspinous fossa and initiates abduction. Thus, B is correct.
3) Subscapularis muscle lies on which surface?
A) Anterior scapula
B) Posterior scapula above spine
C) Posterior scapula below spine
D) Lateral border
Answer: A) Anterior scapula
Explanation: Subscapularis occupies the subscapular fossa on anterior surface. Thus, A is correct.
4) The infraspinatus muscle is responsible for–
A) Medial rotation
B) Lateral rotation
C) Abduction
D) Adduction
Answer: B) Lateral rotation
Explanation: Infraspinatus is a major lateral rotator of the humerus. Thus, B is correct.
5) Which nerve supplies the infraspinatus?
A) Axillary nerve
B) Suprascapular nerve
C) Thoracodorsal nerve
D) Spinal accessory nerve
Answer: B) Suprascapular nerve
Explanation: Suprascapular nerve supplies both supraspinatus and infraspinatus. Thus, B is correct.
6) A patient with suprascapular nerve lesion will have weakened–
A) Lateral rotation
B) Medial rotation
C) Finger flexion
D) Elbow extension
Answer: A) Lateral rotation
Explanation: Loss of infraspinatus affects lateral rotation. Thus, A is correct.
7) Teres minor lies–
A) Above infraspinatus
B) Below infraspinatus
C) Anterior to subscapularis
D) Within supraspinous fossa
Answer: B) Below infraspinatus
Explanation: Teres minor is inferior to infraspinatus on posterior scapula. Thus, B is correct.
8) Which muscle inserts on the lesser tubercle?
A) Infraspinatus
B) Supraspinatus
C) Subscapularis
D) Teres minor
Answer: C) Subscapularis
Explanation: Subscapularis uniquely inserts on lesser tubercle. Thus, C is correct.
9) Suprascapular nerve passes through–
A) Quadrangular space
B) Suprascapular notch
C) Triangular space
D) Guyon’s canal
Answer: B) Suprascapular notch
Explanation: The nerve passes under the superior transverse ligament. Thus, B is correct.
10) In rotator cuff tears, which muscle is most commonly affected?
A) Subscapularis
B) Supraspinatus
C) Infraspinatus
D) Teres major
Answer: B) Supraspinatus
Explanation: Supraspinatus tendon is most vulnerable to impingement. Thus, B is correct.
11) A patient with difficulty initiating abduction likely has injury to–
A) Infraspinatus
B) Latissimus dorsi
C) Supraspinatus
D) Deltoid
Answer: C) Supraspinatus
Explanation: Supraspinatus initiates first 15° of abduction. Thus, C is correct.
Chapter: Upper Limb Anatomy; Topic: Intrinsic Muscles of the Hand; Subtopic: Muscles Acting on the Thumb
Keyword Definitions:
Thenar Muscles: Intrinsic hand muscles responsible for fine thumb movements.
Dual Nerve Supply: A muscle receiving innervation from two different nerves.
Recurrent Median Nerve: Branch supplying most thenar muscles.
Deep Branch of Ulnar Nerve: Innervates some intrinsic hand muscles including part of FPB.
Pollicis Muscles: Muscles specifically acting on the thumb (pollex).
1) Lead Question – 2016
Which muscle acting on the thumb has dual nerve supply?
A) Flexor Pollicis Longus
B) Flexor Pollicis Brevis
C) Adductor Pollicis
D) Opponens Pollicis
Answer: B) Flexor Pollicis Brevis
Explanation: Flexor pollicis brevis (FPB) has two heads—superficial and deep. The superficial head is supplied by the recurrent branch of the median nerve, while the deep head is supplied by the deep branch of the ulnar nerve, making FPB the classic muscle with dual innervation. Flexor pollicis longus is entirely median-nerve supplied via the anterior interosseous nerve. Adductor pollicis receives only ulnar nerve supply. Opponens pollicis is solely median-innervated. Therefore, FPB is the correct answer, and its dual innervation is clinically important in thenar nerve injury assessment.
2) Adductor pollicis is supplied by–
A) Median nerve
B) Ulnar nerve
C) Radial nerve
D) Musculocutaneous nerve
Answer: B) Ulnar nerve
Explanation: Adductor pollicis receives deep branch of ulnar nerve supply. Thus, B is correct.
3) Opponens pollicis is responsible for–
A) Thumb flexion
B) Thumb extension
C) Thumb opposition
D) Thumb adduction
Answer: C) Thumb opposition
Explanation: It brings thumb across palm for precision grip. Thus, C is correct.
4) Flexor pollicis longus is innervated by–
A) Radial nerve
B) Median nerve (AIN)
C) Ulnar nerve
D) Posterior interosseous nerve
Answer: B) Median nerve (AIN)
Explanation: The anterior interosseous nerve supplies FPL. Thus, B is correct.
5) In carpal tunnel syndrome, which thumb movement is most affected?
A) Adduction
B) Opposition
C) Extension
D) Abduction
Answer: B) Opposition
Explanation: Median nerve compression affects thenar muscles, especially opponens pollicis. Thus, B is correct.
6) Froment’s sign tests weakness of–
A) Opponens pollicis
B) Adductor pollicis
C) FPB superficial head
D) EPL
Answer: B) Adductor pollicis
Explanation: Froment’s sign indicates ulnar nerve palsy causing weak adductor pollicis. Thus, B is correct.
7) Median nerve injury leads to–
A) Loss of thumb adduction
B) Loss of thumb opposition
C) Weak extension of thumb
D) Clawing of little finger
Answer: B) Loss of thumb opposition
Explanation: Opponens pollicis is median-innervated; injury prevents opposition. Thus, B is correct.
8) Deep head of FPB is innervated by–
A) Median nerve
B) Ulnar nerve
C) Radial nerve
D) None
Answer: B) Ulnar nerve
Explanation: Dual innervation: superficial head (median), deep head (ulnar). Thus, B is correct.
9) Which muscle forms the anatomical snuffbox posterior boundary?
A) EPL
B) EPB
C) APL
D) FPB
Answer: A) EPL
Explanation: EPL tendon forms posterior border of the snuffbox. Thus, A is correct.
10) Injury to the radial nerve affects which thumb movement most?
A) Extension
B) Opposition
C) Flexion
D) Adduction
Answer: A) Extension
Explanation: Radial nerve supplies extensors like EPL and EPB. Thus, A is correct.
11) A thumb pinch with flexion of IP joint indicates weakness of–
A) Opponens pollicis
B) Adductor pollicis
C) FPL
D) APL
Answer: B) Adductor pollicis
Explanation: Compensation by FPL due to weak adductor pollicis is Froment’s sign. Thus, B is correct.
Chapter: Upper Limb Anatomy; Topic: Intrinsic Muscles of the Hand; Subtopic: Lumbricals (Structure & Function)
Keyword Definitions:
Lumbricals: Intrinsic hand muscles arising from FDP tendons and inserting into extensor expansions.
Unipennate Muscle: Fibers attach to one side of a tendon.
Bipennate Muscle: Fibers attach to both sides of a central tendon.
Multipennate Muscle: Muscle with multiple feather-like fascicles converging onto several tendons.
Extensor Expansion: Triangular aponeurosis on the dorsum of fingers receiving lumbrical insertion.
1) Lead Question – 2016
What type of muscles are medial two lumbricals?
A) Unipennate
B) Bipennate
C) Multipennate
D) None
Answer: B) Bipennate
Explanation: The medial two lumbricals (3rd and 4th) of the hand are bipennate muscles. They arise from the adjacent sides of the tendons of the flexor digitorum profundus, giving them a two-headed or bipennate configuration. In contrast, the lateral two lumbricals (1st and 2nd) are unipennate. These muscles flex the metacarpophalangeal joints and extend the interphalangeal joints via the extensor expansion. Recognizing their pennation is important for understanding their strength, function, and involvement in neuropathies such as ulnar nerve palsy, which impairs medial lumbricals.
2) The medial two lumbricals are supplied by–
A) Median nerve
B) Ulnar nerve
C) Radial nerve
D) Musculocutaneous nerve
Answer: B) Ulnar nerve
Explanation: The medial lumbricals (3rd & 4th) receive innervation from the deep branch of the ulnar nerve, unlike the lateral lumbricals supplied by the median nerve. Thus, B is correct.
3) Lumbricals produce which combined action?
A) Flex MCP & Flex IP
B) Extend MCP & Flex IP
C) Flex MCP & Extend IP
D) Extend MCP & Extend IP
Answer: C) Flex MCP & Extend IP
Explanation: Lumbricals flex the metacarpophalangeal joints and extend the interphalangeal joints via the extensor expansion. Thus, C is correct.
4) In ulnar nerve palsy, which lumbricals lose function?
A) 1st & 2nd
B) 2nd & 3rd
C) 3rd & 4th
D) All
Answer: C) 3rd & 4th
Explanation: The ulnar nerve supplies medial lumbricals; their paralysis contributes to clawing. Thus, C is correct.
5) Lumbricals originate from which structure?
A) Flexor digitorum superficialis tendons
B) Flexor digitorum profundus tendons
C) Interossei tendons
D) Extensor digitorum tendons
Answer: B) Flexor digitorum profundus tendons
Explanation: All lumbricals arise from FDP tendons; medial lumbricals arise from adjacent tendons. Thus, B is correct.
6) A patient presents with difficulty extending IP joints of ring and little fingers. This suggests dysfunction of–
A) Lateral lumbricals
B) Medial lumbricals
C) Palmar interossei
D) Dorsal interossei
Answer: B) Medial lumbricals
Explanation: Medial lumbricals extend IP joints of digits 4 and 5. Thus, B is correct.
7) Lumbricals insert into–
A) Base of proximal phalanx
B) FDP tendon
C) Extensor expansion
D) Metacarpal head
Answer: C) Extensor expansion
Explanation: Their insertion into the extensor expansion allows extension of IP joints. Thus, C is correct.
8) Which muscle group assists lumbricals in IP extension?
A) Interossei
B) Thenar muscles
C) Hypothenar muscles
D) Brachioradialis
Answer: A) Interossei
Explanation: Interossei and lumbricals act synergistically to extend IP joints. Thus, A is correct.
9) Bipennate muscles typically provide–
A) More force
B) Less force
C) No mechanical advantage
D) Only precision movement
Answer: A) More force
Explanation: Bipennate arrangement increases physiological cross-sectional area, enhancing force production. Thus, A is correct.
10) Which lumbricals are unipennate?
A) 1st & 2nd
B) 2nd & 3rd
C) 3rd & 4th
D) All four
Answer: A) 1st & 2nd
Explanation: Lateral lumbricals are unipennate and median-nerve–supplied. Thus, A is correct.
11) A hand injury damaging FDP tendons to digits 4 & 5 will affect which lumbricals most?
A) 1st & 2nd
B) 2nd & 3rd
C) 3rd & 4th
D) All
Answer: C) 3rd & 4th
Explanation: Since lumbricals originate from FDP tendons, injury to FDP of digits 4–5 impairs the medial two lumbricals. Thus, C is correct.
Chapter: Cell Physiology; Topic: Membrane Potentials; Subtopic: Resting Membrane Potential in Smooth Muscle
KEYWORD DEFINITIONS
• Resting membrane potential (RMP) – Electrical potential difference across cell membrane at rest
• Smooth muscle – Involuntary muscle with unstable RMP
• Ion permeability – Determines RMP based on K⁺, Na⁺, and Cl⁻ conductance
• Slow waves – Rhythmic oscillations in smooth muscle membrane potential
• Depolarization – Shift of membrane potential toward positivity
Lead Question – 2015
1. RMP in smooth muscles?
A) -90 mV
B) -70 mV
C) -150 mV
D) -40 mV
Explanation:
Smooth muscle cells have a less negative resting membrane potential compared to skeletal muscle and neurons. Their RMP typically ranges from –40 mV to –60 mV, depending on tissue type and ionic conductance. This relatively depolarized baseline allows spontaneous rhythmic activity and slow wave generation. Skeletal muscle has an RMP near –90 mV, cardiac muscle around –85 mV, and neurons approximately –70 mV. Therefore, the correct answer is –40 mV. This makes smooth muscle highly responsive to neural and hormonal inputs.
2. Typical RMP of skeletal muscle is:
A) –40 mV
B) –55 mV
C) –90 mV
D) –30 mV
Explanation:
Skeletal muscle fibers maintain a highly negative RMP of about –90 mV due to high potassium permeability and large inwardly rectifying K⁺ currents. This stabilizes the membrane and prevents spontaneous contractions. Values like –40 mV or –55 mV represent smooth muscle or neurons, not skeletal muscle. Therefore, the correct answer is –90 mV. This helps maintain excitability and proper neuromuscular function.
3. A patient with hypokalemia shows hyperpolarized smooth muscle cells. Reason?
A) Increased Na⁺ influx
B) Decreased K⁺ efflux
C) Increased K⁺ gradient across membrane
D) Reduced Cl⁻ permeability
Explanation:
Hypokalemia increases the electrochemical gradient for K⁺, leading to increased efflux and a more negative RMP (hyperpolarization). Smooth muscle becomes less excitable. Na⁺ influx and Cl⁻ permeability changes are not primary determinants here. Therefore, the correct answer is Increased K⁺ gradient across membrane. This may reduce gut motility and vascular tone.
4. RMP of neurons is closest to:
A) –70 mV
B) –30 mV
C) –10 mV
D) +10 mV
Explanation:
Neuronal RMP is typically around –70 mV, determined mainly by high K⁺ permeability and fewer Na⁺ leak channels. Depolarized values like –30 mV or –10 mV are abnormal, while positive membrane potentials occur only during action potentials. Thus, the correct answer is –70 mV. This baseline is essential for rapid signal conduction.
5. Depolarization of smooth muscle is primarily caused by influx of:
A) Na⁺ only
B) Ca²⁺
C) Cl⁻
D) HCO₃⁻
Explanation:
Smooth muscle action potentials rely heavily on Ca²⁺ influx through voltage-gated channels rather than Na⁺ influx, as in neurons. Ca²⁺ entry triggers contraction and depolarization. Cl⁻ and bicarbonate do not initiate depolarization. Thus, the correct answer is Ca²⁺. This mechanism links electrical and mechanical activity.
6. A 55-year-old patient with GI hypomotility likely has smooth muscle that is:
A) Depolarized excessively
B) Hyperpolarized
C) Firing continuous action potentials
D) In refractory period
Explanation:
Hyperpolarization makes smooth muscle less excitable, reducing contractile activity and contributing to hypomotility. Excessive depolarization or continuous firing would cause hypermotility, not hypomotility. Refractory periods are brief and cannot explain persistent reduction. Therefore, the correct answer is Hyperpolarized. This state decreases GI peristalsis.
7. Which ion is mainly responsible for maintaining RMP in all excitable tissues?
A) Na⁺
B) Ca²⁺
C) K⁺
D) Mg²⁺
Explanation:
Potassium (K⁺) is the major determinant of RMP due to high membrane permeability and its equilibrium potential. Na⁺ and Ca²⁺ contribute minimally at rest, while Mg²⁺ has little direct role. Therefore, the correct answer is K⁺. Alterations in K⁺ levels strongly influence excitability across tissues.
8. Smooth muscle shows slow waves because of:
A) Pacemaker interstitial cells of Cajal
B) Skeletal motor neurons
C) High frequency Na⁺ channels
D) Increased myelin thickness
Explanation:
Slow wave rhythms originate from interstitial cells of Cajal (ICC), which act as pacemakers in the GI tract. They generate rhythmic oscillations that modulate smooth muscle excitability. Skeletal neurons, Na⁺ channels, and myelin do not mediate slow waves. Thus, the correct answer is Pacemaker interstitial cells of Cajal. This mechanism coordinates peristalsis.
9. Smooth muscle contraction is primarily triggered by Ca²⁺ binding to:
A) Troponin C
B) Calmodulin
C) Tropomyosin
D) Myosin light chain phosphatase
Explanation:
Smooth muscle contraction begins when Ca²⁺ binds to calmodulin, forming a complex that activates myosin light chain kinase (MLCK). Unlike skeletal muscle, smooth muscle lacks troponin. Tropomyosin has a structural role but not regulatory. Therefore, the correct answer is Calmodulin. This pathway enables sustained contractions with low energy use.
10. A hypertensive patient’s arterioles show increased smooth muscle tone due to:
A) Depolarization of RMP
B) Hyperpolarization of RMP
C) Reduced Ca²⁺ entry
D) Increased K⁺ efflux
Explanation:
Depolarization brings the membrane closer to threshold and increases Ca²⁺ entry, leading to elevated smooth muscle tone and vasoconstriction. Hyperpolarization, reduced calcium entry, or increased K⁺ efflux would decrease tone. Thus, the correct answer is Depolarization of RMP. This contributes significantly to increased peripheral resistance.
11. Which of the following smooth muscles has the most unstable RMP?
A) Vascular smooth muscle
B) GI smooth muscle
C) Iris sphincter muscle
D) Bronchiolar smooth muscle
Explanation:
GI smooth muscle exhibits highly unstable RMP due to pacemaker activity from ICC, producing rhythmic slow waves. Vascular, iris, and bronchiolar muscles have more stable potentials. Therefore, the correct answer is GI smooth muscle. This instability is essential for coordinated motility patterns.
Topic: Muscle Physiology; Subtopic: Excitation-Contraction Coupling and Ionic Imbalance
Keyword Definitions:
• Tetany: Sustained, involuntary muscle contraction caused by increased neuronal excitability.
• Calcium (Ca²⁺): Essential for muscle contraction and neuromuscular stability.
• Magnesium (Mg²⁺): Cofactor that stabilizes nerve and muscle membranes; its deficiency enhances excitability.
• Sodium (Na⁺): Major extracellular ion responsible for depolarization during action potential.
• Potassium (K⁺): Maintains resting membrane potential; imbalance alters muscle excitability.
• Neuromuscular junction: Synapse between motor neuron and muscle fiber responsible for initiating contraction.
Lead Question - 2015
Tetany in muscle occurs in spite of normal serum Ca²⁺ level. Which ion is responsible?
a) Mg²⁺
b) Ca²⁺
c) K⁺
d) Na⁺
Explanation (Answer: a) Mg²⁺)
Magnesium deficiency causes increased neuronal excitability leading to tetany even when serum calcium is normal. Mg²⁺ acts as a natural calcium channel blocker, stabilizing nerve membranes. Low Mg²⁺ enhances acetylcholine release at the neuromuscular junction, producing hyperexcitability and muscle spasms. Clinically, this is seen in malnutrition, chronic diarrhea, or diuretic use. Correction of magnesium restores neuromuscular stability.
1. Hypomagnesemia causes tetany due to:
a) Reduced acetylcholine release
b) Enhanced neuromuscular excitability
c) Increased threshold potential
d) Blocked sodium channels
Explanation (Answer: b) Enhanced neuromuscular excitability)
Hypomagnesemia lowers the threshold for neuronal firing, leading to hyperexcitability and spontaneous muscle contractions. Mg²⁺ normally stabilizes neuronal membranes and regulates calcium influx. Its deficiency causes repetitive firing and clinical tetany despite normal calcium levels, a classic example of ionic imbalance affecting neuromuscular control.
2. Carpopedal spasm in a patient with normal calcium suggests deficiency of:
a) Sodium
b) Magnesium
c) Potassium
d) Phosphate
Explanation (Answer: b) Magnesium)
Carpopedal spasm is a typical sign of tetany due to magnesium deficiency. Low Mg²⁺ increases acetylcholine release, causing sustained muscle contraction. Even if calcium is normal, Mg²⁺ deficiency disrupts neuromuscular stability. Intravenous magnesium rapidly reverses the symptoms, confirming its essential role in membrane stabilization.
3. Magnesium acts physiologically as:
a) Calcium channel activator
b) Natural calcium antagonist
c) Potassium transporter
d) Sodium pump inhibitor
Explanation (Answer: b) Natural calcium antagonist)
Magnesium functions as a natural calcium antagonist by blocking calcium influx into presynaptic terminals, preventing excessive neurotransmitter release. Deficiency leads to uncontrolled calcium entry, enhancing excitability and causing tetany. It also regulates cardiac excitability and smooth muscle tone, maintaining electrical stability throughout the body.
4. In severe magnesium deficiency, serum calcium decreases because:
a) Parathyroid hormone release decreases
b) Calcitonin increases
c) Renal calcium excretion decreases
d) Vitamin D level rises
Explanation (Answer: a) Parathyroid hormone release decreases)
Severe hypomagnesemia inhibits parathyroid hormone (PTH) secretion and reduces its peripheral action, resulting in hypocalcemia. This secondary effect can cause severe muscle spasms and seizures. Correction of magnesium deficiency restores PTH secretion and normal calcium homeostasis, preventing persistent neuromuscular irritability.
5. Latent tetany can be elicited clinically by:
a) Babinski sign
b) Trousseau’s sign
c) Romberg’s test
d) Rinne’s test
Explanation (Answer: b) Trousseau’s sign)
Trousseau’s sign is positive when carpal spasm occurs upon inflating a blood pressure cuff above systolic pressure for 3 minutes. It indicates latent tetany due to neuromuscular hyperexcitability, often caused by hypocalcemia or hypomagnesemia. It is a valuable bedside test for assessing ionic imbalances affecting muscle excitability.
6. Chvostek’s sign is due to:
a) Hypokalemia
b) Hypocalcemia or hypomagnesemia
c) Hypercalcemia
d) Hypermagnesemia
Explanation (Answer: b) Hypocalcemia or hypomagnesemia)
Chvostek’s sign is elicited by tapping the facial nerve anterior to the ear, producing twitching of facial muscles. It reflects increased neuromuscular excitability due to hypocalcemia or hypomagnesemia. Both ions stabilize membranes; their deficiency lowers the threshold for depolarization, causing hyperresponsive muscle activity.
7. Excessive magnesium in the body causes:
a) Muscle tetany
b) Depressed reflexes
c) Increased excitability
d) Spastic paralysis
Explanation (Answer: b) Depressed reflexes)
Hypermagnesemia depresses neuromuscular transmission and decreases reflexes due to reduced acetylcholine release and calcium entry. It causes muscle weakness, hypotension, and bradycardia. High magnesium levels inhibit nerve conduction and cardiac contractility, contrasting the excitatory effects seen in magnesium deficiency.
8. In tetany due to alkalosis, excitability increases because:
a) Ionized calcium decreases
b) Sodium permeability decreases
c) Magnesium concentration rises
d) pH decreases
Explanation (Answer: a) Ionized calcium decreases)
Alkalosis increases calcium binding to albumin, reducing the free ionized calcium fraction without changing total calcium levels. This decreases threshold potential, enhancing neuronal excitability and causing tetany. Clinically, respiratory alkalosis during hyperventilation can precipitate hand or foot spasms due to transient hypocalcemia.
9. A 50-year-old alcoholic patient presents with tetany and normal calcium. Likely cause:
a) Vitamin D deficiency
b) Magnesium deficiency
c) Hyperkalemia
d) Sodium excess
Explanation (Answer: b) Magnesium deficiency)
Chronic alcoholism causes magnesium loss via urine and poor dietary intake. Hypomagnesemia induces tetany despite normal calcium due to neuromuscular hyperexcitability. Intravenous magnesium replacement alleviates symptoms. This is a classic example of alcohol-related electrolyte imbalance leading to secondary neurological manifestations.
10. Which of the following is NOT a feature of hypomagnesemia?
a) Tetany
b) Muscle weakness
c) Cardiac arrhythmias
d) Respiratory depression
Explanation (Answer: d) Respiratory depression)
Hypomagnesemia causes tetany, muscle cramps, and cardiac arrhythmias due to increased excitability. Respiratory depression occurs in hypermagnesemia from excessive neuromuscular blockade, not deficiency. Correcting magnesium imbalance is crucial to prevent complications, especially in critically ill or malnourished patients.
11. Which electrolyte abnormality produces tetany even with normal serum calcium?
a) Hyperkalemia
b) Hypomagnesemia
c) Hypernatremia
d) Hypercalcemia
Explanation (Answer: b) Hypomagnesemia)
Hypomagnesemia mimics hypocalcemia by increasing neuromuscular excitability and producing tetany even with normal calcium levels. Mg²⁺ deficiency alters calcium channel function and acetylcholine release. It is often seen in malabsorption, alcoholism, or prolonged diuretic therapy. Restoring magnesium quickly corrects symptoms and stabilizes neuromuscular activity.
Topic: Muscle Physiology; Subtopic: Smooth Muscle Contraction Mechanism
Keyword Definitions:
• Smooth muscle: Non-striated involuntary muscle found in walls of hollow organs.
• Calmodulin: Calcium-binding protein that activates myosin light chain kinase (MLCK).
• Myosin light chain kinase (MLCK): Enzyme that phosphorylates myosin to initiate contraction.
• Troponin: Regulatory protein in skeletal and cardiac muscles absent in smooth muscle.
• Phosphorylation: Addition of phosphate group essential for myosin–actin interaction in smooth muscle.
• Latch mechanism: Sustained contraction of smooth muscle with minimal ATP consumption.
Lead Question - 2015
True about smooth muscle contraction ?
a) Troponin plays an important role
b) Calmodulin has no role
c) Phosphorylation of myosin
d) All of the above
Explanation (Answer: c) Phosphorylation of myosin)
Phosphorylation of myosin is the key step in smooth muscle contraction. Calcium binds to calmodulin, activating MLCK, which phosphorylates the myosin light chain, allowing cross-bridge formation with actin. Unlike skeletal muscle, smooth muscle lacks troponin. Calmodulin replaces troponin’s role. Dephosphorylation by myosin phosphatase relaxes the muscle. This mechanism supports sustained contraction at low energy cost (latch state).
1. The regulatory protein in smooth muscle contraction is:
a) Troponin
b) Tropomyosin
c) Calmodulin
d) Myosin phosphatase
Explanation (Answer: c) Calmodulin)
Calmodulin serves as the calcium-binding regulatory protein in smooth muscle, analogous to troponin in skeletal muscle. When calcium binds calmodulin, it activates MLCK, which phosphorylates myosin light chains to initiate contraction. Tropomyosin is present but does not block binding sites as in skeletal muscle. Myosin phosphatase reverses phosphorylation during relaxation.
2. Smooth muscle contraction is initiated by increase in:
a) cAMP
b) Intracellular calcium concentration
c) Sodium influx
d) Chloride efflux
Explanation (Answer: b) Intracellular calcium concentration)
Calcium influx or release from the sarcoplasmic reticulum increases intracellular Ca²⁺, binding to calmodulin and activating MLCK. This phosphorylation of myosin initiates contraction. In contrast, increased cAMP leads to relaxation by inhibiting MLCK. Sodium and chloride ions are not directly involved in the contraction process of smooth muscle.
3. Latch state in smooth muscle helps in:
a) Rapid contraction
b) Sustained contraction with low energy
c) Muscle fatigue
d) Myosin degradation
Explanation (Answer: b) Sustained contraction with low energy)
The latch state allows smooth muscle to maintain tension for prolonged periods with minimal ATP consumption. After myosin dephosphorylation, cross-bridges remain attached, maintaining force without active cycling. This mechanism is crucial in organs like the bladder, uterus, and blood vessels that require continuous tone with low metabolic demand.
4. Drug that causes smooth muscle relaxation by increasing cAMP:
a) Norepinephrine
b) Epinephrine (via β₂ receptors)
c) Acetylcholine
d) Histamine
Explanation (Answer: b) Epinephrine (via β₂ receptors))
Epinephrine acts on β₂-adrenergic receptors to increase cAMP levels, inhibiting MLCK and promoting smooth muscle relaxation. This mechanism underlies bronchodilation and vasodilation. Norepinephrine via α-receptors causes contraction, while acetylcholine and histamine usually increase contraction in visceral smooth muscle through Ca²⁺ release mechanisms.
5. Which enzyme mediates smooth muscle relaxation?
a) Myosin light chain kinase
b) Myosin phosphatase
c) Adenylate cyclase
d) Guanylate cyclase
Explanation (Answer: b) Myosin phosphatase)
Myosin phosphatase dephosphorylates myosin light chains, leading to relaxation of smooth muscle. This process is enhanced by increased cyclic nucleotides such as cGMP (via nitric oxide). Myosin light chain kinase promotes contraction, while adenylate and guanylate cyclases regulate second messengers but not direct dephosphorylation.
6. Nitric oxide causes smooth muscle relaxation through:
a) Decreasing calcium levels
b) Activation of guanylate cyclase
c) Increasing cGMP
d) All of the above
Explanation (Answer: d) All of the above)
Nitric oxide (NO) diffuses into smooth muscle cells and activates guanylate cyclase, increasing cGMP. cGMP lowers intracellular calcium and enhances myosin phosphatase activity, causing relaxation. This mechanism mediates vasodilation in blood vessels and penile erection via the NO–cGMP pathway, targeted by drugs like sildenafil.
7. Smooth muscle does not show:
a) Gap junctions
b) Troponin
c) Actin and myosin filaments
d) Calmodulin
Explanation (Answer: b) Troponin)
Troponin is absent in smooth muscle. Instead, contraction is regulated by calmodulin-mediated activation of MLCK. Smooth muscle cells are interconnected by gap junctions allowing coordinated contraction. They contain actin and myosin filaments, though not organized into sarcomeres, giving the muscle a smooth appearance under microscopy.
8. Source of calcium for smooth muscle contraction is:
a) Only extracellular fluid
b) Only sarcoplasmic reticulum
c) Both extracellular and sarcoplasmic stores
d) Endoplasmic reticulum of neurons
Explanation (Answer: c) Both extracellular and sarcoplasmic stores)
Calcium required for smooth muscle contraction comes from both extracellular influx through voltage-gated channels and intracellular release from the sarcoplasmic reticulum. The relative contribution depends on muscle type. Drugs that block calcium entry (e.g., nifedipine) cause relaxation by reducing cytosolic calcium levels essential for contraction.
9. A patient receiving a β₂ agonist for asthma experiences muscle relaxation because:
a) Decreased cAMP
b) Increased cAMP inhibits MLCK
c) Increased calcium activates calmodulin
d) Increased potassium depolarizes cells
Explanation (Answer: b) Increased cAMP inhibits MLCK)
β₂-adrenergic stimulation increases cAMP, which inhibits MLCK activity even in the presence of calcium. This prevents phosphorylation of myosin light chains, promoting relaxation of bronchial smooth muscle. Hence, β₂ agonists like salbutamol act as bronchodilators by blocking calcium-dependent contractile mechanisms.
10. Smooth muscle contraction differs from skeletal muscle contraction because:
a) It requires troponin
b) It depends on actin phosphorylation
c) It involves myosin phosphorylation
d) It requires ATP-independent action
Explanation (Answer: c) It involves myosin phosphorylation)
Smooth muscle contraction uniquely depends on phosphorylation of myosin light chains by MLCK following Ca²⁺–calmodulin activation. Skeletal muscle contraction instead depends on troponin–tropomyosin regulation of actin binding. Both require ATP, but smooth muscle sustains tone longer through the latch mechanism and slower cross-bridge cycling.
11. Clinically, calcium channel blockers relieve hypertension by:
a) Decreasing vascular smooth muscle contraction
b) Increasing cardiac output
c) Activating calmodulin
d) Stimulating MLCK
Explanation (Answer: a) Decreasing vascular smooth muscle contraction)
Calcium channel blockers like amlodipine inhibit voltage-gated calcium entry into vascular smooth muscle cells, reducing intracellular calcium. This prevents calmodulin activation and MLCK stimulation, causing vasodilation and lower blood pressure. They act selectively on arterial smooth muscle, improving cardiac workload and oxygen delivery.
Topic: Nerve and Muscle Physiology; Subtopic: Electromyography (EMG) and Muscle Activity
Keyword Definitions:
• Electromyography (EMG): Diagnostic test recording electrical activity produced by skeletal muscles using needle electrodes.
• Insertional activity: Brief burst of electrical activity seen when an electrode is inserted into a muscle, reflecting membrane stability.
• Resting potential: Electrical potential difference across the muscle membrane at rest.
• Fibrillation potentials: Spontaneous discharges from denervated muscle fibers indicating pathology.
• Motor unit potential (MUP): The sum of electrical signals from a motor neuron and its muscle fibers during voluntary contraction.
• Denervation: Loss of nerve supply leading to abnormal spontaneous muscle activity on EMG.
Lead Question - 2015
In electromyography (EMG) transient response at the time of insertion of electrode indicates ?
a) Spontaneous muscle activity
b) Voluntary muscle activity
c) Induced muscle activity
d) Cell membrane damage
Explanation (Answer: d) Cell membrane damage)
During EMG, the brief burst of electrical activity that appears at the moment of electrode insertion is called insertional activity. It occurs due to minor mechanical injury to the muscle fiber membrane by the needle, causing transient depolarization. Normally, it lasts less than 300 milliseconds. Prolonged or absent insertional activity indicates pathology such as denervation or fibrosis. Hence, the transient response signifies local cell membrane damage rather than voluntary or spontaneous activity.
1. Increased insertional activity on EMG is seen in:
a) Myasthenia gravis
b) Myopathy
c) Denervated muscle
d) Fibrotic muscle
Explanation (Answer: c) Denervated muscle)
Denervation increases insertional activity because muscle fibers become hypersensitive and electrically unstable due to loss of neural input. Needle insertion excites these unstable membranes, producing prolonged discharges. Myasthenia affects neuromuscular transmission, not insertional activity. Fibrotic muscles show decreased activity. Denervation thus indicates nerve injury or neuropathy affecting motor control.
2. Decreased insertional activity is found in:
a) Acute neuropathy
b) Chronic myopathy
c) Muscle fibrosis
d) Neuromuscular junction disorder
Explanation (Answer: c) Muscle fibrosis)
Muscle fibrosis reduces insertional activity because the fibrotic tissue has fewer excitable fibers and poor membrane responsiveness. Normal muscles show short bursts; fibrotic ones remain electrically silent. This is seen in chronic myopathies or late-stage muscular dystrophy. Acute neuropathy increases, while NMJ disorders preserve normal insertional responses.
3. Fibrillation potentials in EMG indicate:
a) Myasthenia gravis
b) Denervation of muscle
c) Myotonia
d) Neuromuscular block
Explanation (Answer: b) Denervation of muscle)
Fibrillation potentials are spontaneous discharges from single muscle fibers due to denervation. They appear 1–3 weeks after nerve injury and indicate lower motor neuron damage. Myotonia shows repetitive discharges after activation. Myasthenia involves postsynaptic failure without fibrillations. Thus, fibrillations are a hallmark of denervated muscle activity on EMG.
4. Polyphasic motor unit potentials on EMG suggest:
a) Nerve regeneration
b) Muscle fatigue
c) Hypokalemia
d) Normal muscle contraction
Explanation (Answer: a) Nerve regeneration)
Polyphasic motor unit potentials result from asynchronous firing of muscle fibers during reinnervation. Regenerating nerve fibers reconnect with muscle fibers irregularly, producing complex waveforms. These indicate partial recovery after nerve injury. Normal MUPs are biphasic or triphasic, while polyphasic potentials suggest active reinnervation and healing.
5. A patient with acute lower motor neuron lesion will show which EMG finding?
a) Absence of insertional activity
b) Increased insertional and fibrillation activity
c) Normal MUPs
d) High-frequency myotonic discharges
Explanation (Answer: b) Increased insertional and fibrillation activity)
Acute LMN lesions cause increased insertional activity due to fiber irritability and denervation fibrillations. Over time, voluntary MUPs disappear. Myotonic discharges are seen in myotonia congenita. Thus, hyperactivity during insertion and spontaneous discharges without voluntary contraction confirm LMN pathology.
6. Insertional activity normally lasts for:
a) b) 50–300 ms
c) 500 ms
d) 1 second
Explanation (Answer: b) 50–300 ms)
Normal insertional activity is a brief burst of potentials lasting 50–300 milliseconds after needle insertion. This duration represents the muscle’s physiological response to minor membrane disruption. Prolonged activity indicates irritability or denervation, while absent activity occurs in fibrosis or severe muscle atrophy.
7. Complex repetitive discharges on EMG are typical of:
a) Myotonia congenita
b) Chronic denervation
c) Myasthenia gravis
d) Periodic paralysis
Explanation (Answer: b) Chronic denervation)
Complex repetitive discharges (CRDs) occur in chronic denervating disorders such as spinal muscular atrophy or longstanding neuropathies. They represent reinnervated muscle fibers firing repetitively in a self-sustained cycle. Myotonia causes waxing–waning discharges, not CRDs. Chronic denervation reflects adaptive but unstable neuromuscular activity.
8. Myotonic discharge pattern in EMG is described as:
a) Electrical silence
b) Dive bomber sound
c) Bursting polyphasic waves
d) Single fiber potential
Explanation (Answer: b) Dive bomber sound)
Myotonic discharges produce a characteristic “dive bomber” sound due to waxing and waning amplitude and frequency. They appear in myotonic disorders like myotonia congenita or dystrophic myotonia. The pattern reflects delayed muscle relaxation after contraction due to abnormal ion channel function. It is a diagnostic feature of myotonic syndromes.
9. EMG is primarily used to differentiate:
a) Central from peripheral lesions
b) Upper from lower motor neuron lesions
c) Sensory from motor conduction disorders
d) Cardiac from skeletal muscle activity
Explanation (Answer: b) Upper from lower motor neuron lesions)
Electromyography helps distinguish lower motor neuron or muscle disorders from central (UMN) lesions. UMN lesions show normal EMG but abnormal reflexes, while LMN lesions display fibrillations, positive sharp waves, or reduced recruitment. Thus, EMG confirms peripheral nerve or muscle pathology rather than central motor pathway dysfunction.
10. Absence of insertional activity with no voluntary response suggests:
a) Neuropathy
b) Myopathy
c) Muscle fibrosis or necrosis
d) Hyperkalemia
Explanation (Answer: c) Muscle fibrosis or necrosis)
When insertional activity is completely absent, it indicates loss of excitable muscle fibers due to fibrosis or necrosis. In such cases, needle insertion does not generate any electrical potential because muscle tissue is replaced by connective tissue. Neuropathy or myopathy usually retains insertional activity, while fibrosis signifies irreversible muscle damage.
11. During EMG, spontaneous fibrillation potentials at rest represent:
a) Voluntary contraction
b) Normal insertional activity
c) Denervation hypersensitivity
d) Fatigue potential
Explanation (Answer: c) Denervation hypersensitivity)
Fibrillation potentials arise from denervated muscle fibers developing hypersensitivity to acetylcholine. These spontaneous discharges occur at rest, appearing 1–3 weeks post denervation. They indicate ongoing nerve injury or chronic lower motor neuron lesion. Voluntary or fatigue potentials occur during muscle activity, not rest, distinguishing fibrillation as a pathological EMG sign.
Chapter: Anatomy of Head and Neck; Topic: Skull and Muscular Attachments; Subtopic: Superior Nuchal Line
Key Definitions:
• Superior nuchal line: A curved ridge on the occipital bone that provides attachment to several neck muscles.
• Trapezius muscle: A large superficial back muscle extending from the occipital bone to the thoracic spine, responsible for shoulder elevation and neck extension.
• Occipital bone: The posterior part of the skull enclosing the foramen magnum.
Lead Question (NEET PG 2015):
1. What is attached to the superior nuchal line?
A) Trapezius
B) Scalenus anticus
C) Coracobrachialis
D) Biceps brachii
Answer: A) Trapezius
Explanation: The superior nuchal line is a prominent ridge on the external surface of the occipital bone, extending laterally from the external occipital protuberance. It gives attachment to several muscles: the upper fibers of the trapezius, sternocleidomastoid, splenius capitis, and occipitalis. Among the options given, the trapezius attaches directly to the medial part of the superior nuchal line. Therefore, the correct answer is Trapezius.
Related (Guessed) Questions:
2. Which muscle attaches to the lateral part of the superior nuchal line?
A) Splenius capitis
B) Sternocleidomastoid
C) Trapezius
D) Occipitalis
3. The external occipital protuberance gives attachment to which structure?
A) Ligamentum nuchae
B) Scalenus posterior
C) Levator scapulae
D) Semispinalis capitis
4. Which muscle extends from the superior nuchal line to the mastoid process?
A) Sternocleidomastoid
B) Splenius capitis
C) Semispinalis capitis
D) Trapezius
5. The inferior nuchal line provides attachment to:
A) Rectus capitis posterior major and minor
B) Splenius capitis
C) Trapezius
D) Occipitalis
6. The ligamentum nuchae is attached to which part of the skull?
A) Superior nuchal line
B) External occipital crest
C) Mastoid process
D) Inferior nuchal line
7. Which of the following muscles is not attached to the occipital bone?
A) Trapezius
B) Splenius capitis
C) Sternocleidomastoid
D) Levator scapulae
8. The external occipital crest extends between:
A) Superior and inferior nuchal lines
B) External occipital protuberance and foramen magnum
C) Foramen magnum and mastoid process
D) Mastoid and parietal bones
9. The occipitalis muscle is part of which group?
A) Facial muscles
B) Neck muscles
C) Back muscles
D) Mastication muscles
10. The sternocleidomastoid muscle has its superior attachment at:
A) Mastoid process
B) Superior nuchal line
C) External occipital crest
D) Temporal line
11. Which structure passes through the foramen magnum?
A) Spinal accessory nerve
B) Hypoglossal nerve
C) Internal carotid artery
D) Vertebral artery
Chapter: Abdomen and Pelvis; Topic: Inguinal Canal' Subtopic: Deep Inguinal Ring and its Contents
Keyword Definitions:
Deep Inguinal Ring: An opening in the transversalis fascia, located above the midpoint of the inguinal ligament, through which the spermatic cord or round ligament passes.
Spermatic Cord: A collection of structures including vas deferens, testicular vessels, and nerves that pass through the inguinal canal to the testis.
Ilioinguinal Nerve: A branch of the first lumbar nerve (L1) that enters the inguinal canal through the superficial ring, not the deep ring.
Round Ligament: A fibromuscular band in females passing through the inguinal canal to support the uterus.
Internal Spermatic Fascia: The innermost covering of the spermatic cord derived from the transversalis fascia at the deep inguinal ring.
Lead Question (2015)
All pass through deep inguinal ring, EXCEPT?
a) Spermatic cord
b) Internal spermatic fascia
c) Round ligament
d) Ilioinguinal nerve
Explanation: The deep inguinal ring transmits the spermatic cord in males and the round ligament in females. The internal spermatic fascia is derived from the transversalis fascia at this ring. However, the ilioinguinal nerve does not pass through the deep inguinal ring but enters the canal midway and exits via the superficial ring. Hence, the correct answer is (d) Ilioinguinal nerve.
1. Which structure forms the anterior wall of the inguinal canal?
a) Transversalis fascia
b) External oblique aponeurosis
c) Internal oblique muscle
d) Fascia transversalis and peritoneum
Explanation: The anterior wall of the inguinal canal is mainly formed by the external oblique aponeurosis and is reinforced laterally by fibers of the internal oblique muscle. This wall provides support and prevents herniation. Hence, the answer is (b) External oblique aponeurosis.
2. The posterior wall of the inguinal canal is formed by -
a) Transversalis fascia
b) Conjoint tendon
c) Both a and b
d) Internal oblique muscle
Explanation: The posterior wall is mainly formed by the transversalis fascia and reinforced medially by the conjoint tendon (fusion of internal oblique and transversus abdominis aponeuroses). Hence, the answer is (c) Both a and b.
3. A 30-year-old male presents with an indirect inguinal hernia. The hernial sac enters through which structure?
a) Deep inguinal ring
b) Superficial inguinal ring
c) Hesselbach’s triangle
d) Femoral canal
Explanation: An indirect inguinal hernia occurs when abdominal contents herniate through the deep inguinal ring, traveling along the spermatic cord, and may reach the scrotum. It is lateral to the inferior epigastric vessels. Hence, the answer is (a) Deep inguinal ring.
4. Direct inguinal hernia occurs through -
a) Deep inguinal ring
b) Superficial inguinal ring
c) Posterior wall of inguinal canal
d) Femoral canal
Explanation: Direct inguinal hernia occurs through a weakness in the posterior wall of the inguinal canal within Hesselbach’s triangle, medial to the inferior epigastric vessels. It usually does not reach the scrotum. Hence, the answer is (c) Posterior wall of inguinal canal.
5. Which layer gives rise to the cremasteric muscle and fascia?
a) External oblique aponeurosis
b) Internal oblique muscle
c) Transversus abdominis
d) Transversalis fascia
Explanation: The cremasteric muscle and fascia are derived from the internal oblique muscle layer as the spermatic cord passes through the inguinal canal. The cremaster muscle elevates the testis during temperature changes. Hence, the answer is (b) Internal oblique muscle.
6. Which nerve supplies the cremasteric muscle?
a) Ilioinguinal nerve
b) Genital branch of genitofemoral nerve
c) Femoral nerve
d) Pudendal nerve
Explanation: The genital branch of the genitofemoral nerve supplies the cremasteric muscle. This nerve also carries afferent fibers for the cremasteric reflex. Hence, the answer is (b) Genital branch of genitofemoral nerve.
7. A patient presents with absence of the cremasteric reflex. The lesion is likely in -
a) L1 spinal segment
b) L2 spinal segment
c) L3 spinal segment
d) S2 spinal segment
Explanation: The cremasteric reflex involves the sensory input from the ilioinguinal nerve (L1) and motor output through the genital branch of the genitofemoral nerve (L1–L2). Loss of reflex indicates damage to L1–L2 segments. Hence, the answer is (b) L2 spinal segment.
8. Which structure forms the roof of the inguinal canal?
a) Transversus abdominis and internal oblique muscles
b) External oblique aponeurosis
c) Conjoint tendon
d) Transversalis fascia
Explanation: The roof of the inguinal canal is formed by arching fibers of the transversus abdominis and internal oblique muscles. These fibers provide dynamic support during increased intra-abdominal pressure. Hence, the answer is (a) Transversus abdominis and internal oblique muscles.
9. The superficial inguinal ring is an opening in the -
a) Transversalis fascia
b) Internal oblique muscle
c) External oblique aponeurosis
d) Conjoint tendon
Explanation: The superficial inguinal ring is a triangular gap in the external oblique aponeurosis located just above the pubic crest. It allows exit of the spermatic cord or round ligament. Hence, the answer is (c) External oblique aponeurosis.
10. In a male child with a patent processus vaginalis, which hernia type is most likely?
a) Direct inguinal hernia
b) Indirect inguinal hernia
c) Umbilical hernia
d) Femoral hernia
Explanation: A patent processus vaginalis creates a congenital communication between the peritoneal cavity and scrotum, predisposing to an indirect inguinal hernia. The hernia follows the same path as the spermatic cord. Hence, the answer is (b) Indirect inguinal hernia.
Chapter: Abdomen; Topic: Posterior Abdominal Wall; Subtopic: Lumbar Triangles
Keyword Definitions:
Petit Triangle: It is the inferior lumbar triangle located on the posterolateral abdominal wall.
Internal Oblique: Muscle forming part of the abdominal wall, important in trunk rotation and support.
Fascia Transversalis: Thin aponeurotic membrane between the inner surface of the transversus abdominis and the peritoneum.
Sacrospinalis (Erector Spinae): Group of muscles running longitudinally along the vertebral column, aiding in extension of the spine.
Rectus Abdominis: Vertical muscle of the anterior abdominal wall responsible for trunk flexion.
Lead Question (2015):
Floor of Petit triangle is formed by?
a) Sacrospinalis
b) Internal oblique
c) Rectus abdominis
d) Fascia Transversalis
Explanation: The correct answer is Internal oblique. The Petit triangle or inferior lumbar triangle is bounded by the latissimus dorsi posteriorly, external oblique anteriorly, and iliac crest inferiorly. Its floor is formed by the internal oblique muscle. This region is clinically important as a potential site for lumbar hernia formation.
1) Superior lumbar triangle (Grynfeltt-Lesshaft) is bounded by?
a) Internal oblique, Quadratus lumborum, and 12th rib
b) Latissimus dorsi, Iliac crest, and External oblique
c) Psoas major, Iliacus, and Transversus abdominis
d) Quadratus lumborum, External oblique, and Iliacus
Explanation: The answer is a) Internal oblique, Quadratus lumborum, and 12th rib. The superior lumbar triangle is located above the inferior triangle and is bordered by the internal oblique anteriorly, quadratus lumborum posteriorly, and the 12th rib superiorly. Its floor is formed by the transversalis fascia and is also a site for lumbar hernia.
2) Which structure passes through the lumbar triangle during a lumbar hernia?
a) Small intestine
b) Appendix
c) Kidney
d) Urinary bladder
Explanation: The correct answer is a) Small intestine. Lumbar hernias are rare and usually occur through weak points such as the inferior or superior lumbar triangles. The hernial sac may contain small intestine, colon, or omentum. The weakness in the internal oblique and transversalis fascia predisposes this area to herniation.
3) Clinical importance of Petit triangle is mainly related to?
a) Appendicitis
b) Lumbar hernia
c) Inguinal hernia
d) Femoral hernia
Explanation: The answer is b) Lumbar hernia. The inferior lumbar triangle (Petit triangle) is a weak area of the posterior abdominal wall where lumbar hernias may occur, especially following trauma or surgery. Recognition of this triangle helps surgeons during retroperitoneal access and hernia repair procedures.
4) Which muscle forms the roof of the inferior lumbar triangle?
a) Latissimus dorsi
b) External oblique
c) Internal oblique
d) Gluteus maximus
Explanation: The correct answer is a) Latissimus dorsi. The inferior lumbar triangle or Petit triangle has the latissimus dorsi muscle forming its posterior border and partially the roof. It is an important anatomical landmark for surgical approaches to the kidney and posterior abdominal wall structures.
5) In a patient with a lumbar hernia, which layer must be sutured to reinforce the floor of Petit triangle?
a) Transversalis fascia
b) Internal oblique
c) External oblique
d) Skin
Explanation: The answer is b) Internal oblique. Since the internal oblique muscle forms the floor of the Petit triangle, it is crucial in repairing lumbar hernias. Suturing or reinforcing this layer helps strengthen the posterior abdominal wall and prevent recurrence of the hernia.
6) A 45-year-old man presents with a soft swelling in the posterolateral abdominal wall below the 12th rib. The hernia likely occurs through?
a) Inferior lumbar triangle
b) Superior lumbar triangle
c) Inguinal canal
d) Femoral canal
Explanation: The answer is a) Inferior lumbar triangle. Clinical presentation of a posterolateral swelling below the 12th rib indicates a lumbar hernia through the Petit triangle. The inferior lumbar triangle is more commonly affected than the superior one due to its weaker muscular support.
7) Which of the following muscles does not form part of any lumbar triangle?
a) Quadratus lumborum
b) Latissimus dorsi
c) Internal oblique
d) Psoas major
Explanation: The correct answer is d) Psoas major. Psoas major lies deep within the posterior abdominal wall and does not contribute to the formation of either the superior or inferior lumbar triangles. Instead, it serves as a landmark for retroperitoneal structures like the ureter and gonadal vessels.
8) Lumbar hernia is more common in which group?
a) Elderly males
b) Young females
c) Children
d) Newborns
Explanation: The answer is a) Elderly males. Lumbar hernias are uncommon but more frequent in older men due to muscle weakness and degenerative changes. They can be spontaneous, traumatic, or postoperative. Knowledge of lumbar anatomy helps in correct diagnosis and surgical planning.
9) The Petit triangle is located in relation to which anatomical landmark?
a) Iliac crest
b) 10th rib
c) Umbilicus
d) Pubic symphysis
Explanation: The correct answer is a) Iliac crest. The inferior lumbar triangle is bounded inferiorly by the iliac crest, posteriorly by the latissimus dorsi, and anteriorly by the external oblique. It is a surface landmark useful in retroperitoneal surgeries and diagnosing posterior abdominal wall hernias.
10) A surgical incision placed along the Petit triangle is used to approach?
a) Kidney
b) Liver
c) Appendix
d) Spleen
Explanation: The answer is a) Kidney. Surgeons sometimes use the inferior lumbar triangle as an access point for retroperitoneal approaches to the kidney, as it provides a relatively avascular and safe pathway. However, care must be taken to avoid herniation postoperatively through this area.
Chapter: Pelvis and Perineum; Topic: Perineum; Subtopic: Perineal Body and Its Muscles
Keyword Definitions:
Perineal Body: A fibromuscular node situated between the anal canal and urogenital structures; it provides support to the pelvic floor.
Perineum: The region between the thighs, divided into urogenital and anal triangles.
Superficial Transverse Perineal Muscle: A paired muscle that helps stabilize the perineal body.
Bulbospongiosus Muscle: Surrounds the bulb of the penis or vestibule; compresses the bulb and assists in erection and ejaculation.
Ischiocavernosus Muscle: Covers the crus of the penis or clitoris; aids in erection by compressing venous drainage.
Deep Transverse Perineal Muscle: Lies deep to the superficial perineal pouch; reinforces the perineal body and supports pelvic viscera.
Lead Question – 2015
The muscles attached to perineal body are A/E –
a) Ischiocavernosum
b) Bulbospongiosus
c) Superficial transverse perinea
d) Deep transverse perinea
Explanation:
The perineal body serves as a central attachment for several muscles including bulbospongiosus, superficial and deep transverse perineal, external anal sphincter, and fibers of levator ani. Ischiocavernosus is not attached to the perineal body but to the ischiopubic ramus. The correct answer is a) Ischiocavernosum. Damage to the perineal body can cause pelvic organ prolapse. (100 words)
1. Which muscle contributes most to the integrity of the perineal body?
a) Bulbospongiosus
b) Ischiocavernosus
c) External anal sphincter
d) Superficial transverse perineal
2. The perineal body is located between –
a) Vagina and rectum
b) Urethra and clitoris
c) Anus and coccyx
d) Ischial tuberosities
3. During episiotomy, incision passes through which structure?
a) Perineal body
b) Anal canal
c) Ischiorectal fossa
d) Urogenital diaphragm
4. Damage to perineal body during childbirth may cause –
a) Cystocele
b) Rectocele
c) Both a and b
d) None
5. The perineal body is derived embryologically from –
a) Cloacal membrane
b) Urogenital sinus
c) Cloacal septum
d) Mesonephric duct
6. (Clinical) A postpartum woman develops uterine prolapse. The most likely cause is –
a) Damage to perineal body
b) Uterine artery injury
c) Pubic fracture
d) Vaginal cyst
7. (Clinical) A patient with perineal tear has difficulty in controlling defecation. Which muscle is likely involved?
a) External anal sphincter
b) Ischiocavernosus
c) Bulbospongiosus
d) Coccygeus
8. (Clinical) Which muscle of the perineum is not attached to perineal body?
a) Ischiocavernosus
b) Bulbospongiosus
c) Deep transverse perineal
d) External anal sphincter
9. (Clinical) A 35-year-old woman presents with weakened pelvic floor after multiple deliveries. Which structure provides major central support?
a) Perineal body
b) Perineal membrane
c) Pubic symphysis
d) Ischial tuberosities
10. (Clinical) A midline perineal incision that preserves the anal sphincter but relieves childbirth stress is called –
a) Median episiotomy
b) Mediolateral episiotomy
c) Perianal incision
d) Transverse perineotomy
Explanation:
The perineal body is the central point of the perineum where major muscles converge, providing essential pelvic support. It is attached to bulbospongiosus, superficial and deep transverse perineal, and external anal sphincter. Ischiocavernosus is not attached. Median episiotomy passes through the perineal body. Correct answer: a) Median episiotomy. (100 words)
Chapter: Lower Limb Anatomy; Topic: Gluteal Region; Subtopic: Structures Leaving the Pelvis through the Greater Sciatic Foramen
Keyword Definitions:
Pelvis: The bony basin formed by the hip bones, sacrum, and coccyx, supporting the spinal column and protecting pelvic organs.
Greater Sciatic Foramen: An opening in the posterior pelvis that allows nerves and vessels to pass from the pelvis to the gluteal region.
Piriformis Muscle: A flat muscle originating from the sacrum and exiting through the greater sciatic foramen to insert into the greater trochanter of the femur.
Gluteal Vessels and Nerves: Branches of the internal iliac artery and sacral plexus that exit through the greater sciatic foramen to supply the gluteal region.
Lead Question (2015):
Which leaves the pelvis?
a) Piriformis
b) Sciatic nerve
c) Superior gluteal vessel
d) Inferior gluteal vessel
Explanation: The Piriformis muscle leaves the pelvis through the greater sciatic foramen. It divides the foramen into a suprapiriform and an infrapiriform compartment. The superior gluteal vessels and nerve exit above the muscle, while the inferior gluteal vessels, sciatic nerve, and pudendal structures pass below it. Piriformis acts as a landmark for gluteal neurovascular structures and laterally rotates the thigh.
Guessed Questions:
1. Which nerve emerges below the piriformis muscle?
a) Sciatic nerve
b) Superior gluteal nerve
c) Pudendal nerve
d) Obturator nerve
Explanation: The Sciatic nerve emerges below the piriformis muscle through the infrapiriform compartment of the greater sciatic foramen. It is the largest nerve in the body, supplying the posterior thigh and the entire lower limb below the knee. Compression by the piriformis may cause piriformis syndrome, leading to radiating pain.
2. Which structure passes above the piriformis muscle?
a) Superior gluteal vessels and nerve
b) Inferior gluteal vessels
c) Sciatic nerve
d) Posterior femoral cutaneous nerve
Explanation: The Superior gluteal artery, vein, and nerve pass above the piriformis muscle through the suprapiriform compartment. They supply the gluteus medius, minimus, and tensor fasciae latae muscles. These structures are vital for hip stabilization and abduction, especially during gait and single-leg stance activities.
3. Which muscle acts as a key landmark for identifying gluteal neurovascular structures?
a) Piriformis
b) Gluteus maximus
c) Obturator internus
d) Quadratus femoris
Explanation: The Piriformis muscle acts as a key landmark in the gluteal region. The superior gluteal vessels and nerves emerge above it, while the inferior gluteal, sciatic, and pudendal structures emerge below. Its orientation helps in safe intramuscular injection placement in the superolateral quadrant to avoid sciatic nerve injury.
4. (Clinical) Which nerve is most commonly injured during wrong site gluteal intramuscular injections?
a) Sciatic nerve
b) Femoral nerve
c) Pudendal nerve
d) Obturator nerve
Explanation: The Sciatic nerve is most frequently injured during improper gluteal injections placed in the inferomedial quadrant. Safe injection should be in the superolateral quadrant of the gluteal region. Injury causes weakness in knee flexion, foot drop, and sensory loss in the leg and foot, leading to functional disability.
5. The inferior gluteal artery is a branch of which vessel?
a) Internal iliac artery
b) External iliac artery
c) Femoral artery
d) Deep femoral artery
Explanation: The Inferior gluteal artery is a terminal branch of the internal iliac artery. It exits through the infrapiriform compartment below the piriformis muscle, accompanying the inferior gluteal nerve and vein. It supplies gluteus maximus and posterior thigh muscles. It also anastomoses with branches of the medial circumflex femoral artery.
6. (Clinical) A patient presents with buttock pain radiating down the leg after sitting long hours. Which structure is likely compressed?
a) Sciatic nerve
b) Femoral nerve
c) Tibial nerve
d) Obturator nerve
Explanation: The Sciatic nerve can be compressed by the piriformis muscle leading to piriformis syndrome. This causes radiating pain similar to sciatica but without spinal pathology. Stretching, posture correction, and muscle relaxation are effective treatments. Severe cases may need surgical decompression of the piriformis tunnel.
7. (Clinical) During posterior hip dislocation, which nerve is most commonly injured?
a) Sciatic nerve
b) Obturator nerve
c) Femoral nerve
d) Pudendal nerve
Explanation: The Sciatic nerve lies posterior to the hip joint and is vulnerable to injury in posterior hip dislocations. Symptoms include weakness of hamstrings and all muscles below the knee, with sensory loss over the leg and foot. Early reduction and physiotherapy are essential to prevent chronic neuropathic pain and gait impairment.
8. The pudendal nerve leaves the pelvis through which route?
a) Greater sciatic foramen below piriformis
b) Lesser sciatic foramen
c) Obturator canal
d) Femoral canal
Explanation: The Pudendal nerve leaves the pelvis through the greater sciatic foramen below the piriformis, winds around the ischial spine, and enters the perineum via the lesser sciatic foramen. It supplies the perineal muscles and external anal sphincter. Damage leads to perineal numbness and incontinence.
9. (Clinical) A gluteal abscess spreading into the pelvis may pass through which structure?
a) Greater sciatic foramen
b) Lesser sciatic foramen
c) Obturator canal
d) Sacral hiatus
Explanation: The Greater sciatic foramen acts as a potential pathway for infection spread between the gluteal region and pelvis. Abscesses involving piriformis or pelvic structures may track through this foramen. Clinical symptoms include deep buttock pain, fever, and difficulty in sitting or walking due to inflammation.
10. (Clinical) After pelvic surgery, a patient shows loss of hip abduction and pelvic drop on walking. Which nerve is injured?
a) Superior gluteal nerve
b) Inferior gluteal nerve
c) Sciatic nerve
d) Pudendal nerve
Explanation: The Superior gluteal nerve supplies gluteus medius, minimus, and tensor fasciae latae muscles. Injury leads to Trendelenburg gait due to pelvic drop on the contralateral side during walking. It may occur from pelvic fractures, injections near the piriformis, or iatrogenic injury during gluteal surgery.
Chapter: Upper Limb Anatomy; Topic: Extensor Compartments of Wrist; Subtopic: Third Extensor Compartment
Keyword Definitions:
Extensor Compartments: Six fibro-osseous tunnels on the dorsum of the wrist through which tendons of the extensor muscles pass.
EPL (Extensor Pollicis Longus): A muscle of the posterior forearm that extends the thumb and passes through the third extensor compartment.
Radial Styloid Process: A bony prominence of the distal radius that serves as an anatomical landmark near the tendons of the extensor muscles.
Synovial Sheath: A membrane enclosing a tendon, reducing friction during movement.
Lead Question (2015):
3rd extensor compartment of wrist contains tendon of?
a) ECRL
b) ECRB
c) EPL
d) EPB
Explanation: The third extensor compartment of the wrist contains the tendon of the Extensor Pollicis Longus (EPL). It passes around the dorsal tubercle of the radius (Lister’s tubercle) and extends to the distal phalanx of the thumb. This arrangement allows thumb extension at interphalangeal and metacarpophalangeal joints. Clinical significance includes tenosynovitis and intersection syndromes.
Guessed Questions:
1. Which of the following tendons passes through the first extensor compartment?
a) APL and EPB
b) ECRL and ECRB
c) EPL and EIP
d) ECU and EDM
Explanation: The first extensor compartment contains Abductor Pollicis Longus (APL) and Extensor Pollicis Brevis (EPB) tendons. It lies lateral to the radial styloid process. Inflammation of this compartment leads to De Quervain’s tenosynovitis, which causes pain during thumb movement and wrist deviation.
2. The sixth extensor compartment of wrist transmits which tendon?
a) ECU
b) ECRB
c) APL
d) EPL
Explanation: The Extensor Carpi Ulnaris (ECU) tendon passes through the sixth extensor compartment, located on the dorsal aspect of the ulna. It helps in wrist extension and ulnar deviation. Inflammation of this compartment is seen in rheumatoid arthritis or repetitive stress injuries affecting wrist stability.
3. The fourth extensor compartment contains which tendons?
a) EDC and EIP
b) EPL and EPB
c) APL and ECRL
d) ECU and EDM
Explanation: The fourth extensor compartment transmits Extensor Digitorum Communis (EDC) and Extensor Indicis Proprius (EIP) tendons. These pass beneath the extensor retinaculum and extend the fingers. Compression here can lead to dorsal wrist pain and impaired finger extension, commonly seen in repetitive strain conditions.
4. Which structure forms the roof of all extensor compartments at the wrist?
a) Extensor retinaculum
b) Palmar aponeurosis
c) Flexor retinaculum
d) Deep fascia of forearm
Explanation: The Extensor retinaculum forms the fibrous roof of all extensor compartments on the dorsum of the wrist. It prevents bowstringing of the extensor tendons during wrist movement. This thickened fascia attaches to the radius laterally and the ulna and triquetral bone medially.
5. The second extensor compartment transmits which tendons?
a) ECRL and ECRB
b) EPL and EPB
c) EIP and EDC
d) ECU and EDM
Explanation: The second compartment transmits Extensor Carpi Radialis Longus (ECRL) and Extensor Carpi Radialis Brevis (ECRB) tendons. These extend and abduct the wrist. They pass beneath the extensor retinaculum and are separated from the first compartment by a fibrous septum. Overuse may lead to lateral wrist pain.
6. (Clinical) A patient presents with pain over the radial styloid process on thumb movement. Which condition is likely?
a) De Quervain’s tenosynovitis
b) Carpal tunnel syndrome
c) Trigger finger
d) Tennis elbow
Explanation: The correct answer is De Quervain’s tenosynovitis. It involves inflammation of the synovial sheaths of APL and EPB tendons within the first extensor compartment. Pain worsens with thumb abduction or Finkelstein’s test. Repetitive strain and overuse are common causes in typists and new mothers.
7. (Clinical) A fracture of the distal radius causing rupture of the tendon in the third extensor compartment will affect which movement?
a) Thumb extension
b) Wrist flexion
c) Finger abduction
d) Thumb opposition
Explanation: The Extensor Pollicis Longus (EPL) tendon passes in the third compartment and may rupture after distal radius fractures. This leads to loss of thumb extension at the interphalangeal joint. Clinically, patients cannot lift the thumb off the table when the hand is flat.
8. (Clinical) A swelling on the dorsum of the wrist along the course of the fourth extensor compartment is most likely due to?
a) Ganglion cyst
b) Lipoma
c) Synovial sarcoma
d) Abscess
Explanation: A Ganglion cyst commonly arises from the synovial sheath of the tendons within the fourth extensor compartment. It presents as a painless swelling that may become tender on movement. Aspiration or surgical excision may be required if it interferes with wrist function.
9. (Clinical) A surgeon must avoid which structure while releasing the extensor retinaculum near the thumb?
a) Radial artery
b) Median nerve
c) Ulnar artery
d) Superficial branch of ulnar nerve
Explanation: The Radial artery lies close to the tendons of the first and second extensor compartments near the radial styloid process. During surgery for De Quervain’s tenosynovitis or wrist ganglion, care is taken to avoid injuring this artery to prevent ischemic complications or hematoma formation.
10. (Clinical) Pain during thumb extension after a distal radius fracture suggests injury to which tendon?
a) Extensor Pollicis Longus
b) Extensor Digitorum Communis
c) Flexor Pollicis Longus
d) Abductor Pollicis Longus
Explanation: Post-fracture irritation or rupture of the Extensor Pollicis Longus (EPL) tendon is common due to its close relation to Lister’s tubercle. The patient loses active thumb extension. Early recognition and tendon transfer procedures help restore functional movement and prevent disability.
Chapter: Upper Limb Anatomy; Topic: Muscles of Arm; Subtopic: Actions and Nerve Supply
Keyword Definitions:
• Biceps Brachii: A two-headed muscle of the anterior arm that flexes the elbow and supinates the forearm.
• Supination: Outward rotation of the forearm so the palm faces upward.
• Brachialis: Primary flexor of the forearm located deep to biceps.
• Coracobrachialis: Flexes and adducts the arm at the shoulder joint.
• Triceps Brachii: Extensor muscle of the arm located on the posterior aspect.
Lead Question - 2015
Muscle of Arm with additional supinator action?
a) Brachialis
b) Biceps
c) Coracobrachialis
d) Triceps
Explanation (Answer: b)
The biceps brachii is the only arm muscle that acts as both a flexor and a supinator of the forearm. When the elbow is flexed, it provides strong supination by rotating the radius over the ulna. The brachialis is purely a flexor, coracobrachialis acts on the shoulder, and triceps is an extensor.
1. Which muscle is the primary flexor of the forearm?
a) Biceps brachii
b) Brachialis
c) Brachioradialis
d) Coracobrachialis
Explanation (Answer: b)
The brachialis is the main flexor of the forearm at the elbow joint. It acts in all positions of pronation and supination. It originates from the anterior humerus and inserts into the coronoid process and ulnar tuberosity. It’s supplied by the musculocutaneous nerve and contributes to powerful elbow flexion.
2. Which of the following muscles acts only on the shoulder joint?
a) Biceps brachii
b) Coracobrachialis
c) Brachialis
d) Triceps
Explanation (Answer: b)
The coracobrachialis acts solely on the shoulder joint. It flexes and adducts the arm. It originates from the coracoid process and inserts on the medial humerus. It assists in stabilizing the humeral head during arm movements and is supplied by the musculocutaneous nerve. It doesn’t cross the elbow joint.
3. The long head of biceps brachii arises from:
a) Coracoid process
b) Supraglenoid tubercle
c) Infraglenoid tubercle
d) Greater tubercle
Explanation (Answer: b)
The long head of biceps brachii originates from the supraglenoid tubercle of the scapula and runs through the intertubercular groove. This tendon stabilizes the shoulder joint and assists in arm abduction and flexion. The short head arises from the coracoid process, joining the long head to form the biceps muscle.
4. Which nerve supplies the biceps brachii?
a) Radial nerve
b) Musculocutaneous nerve
c) Axillary nerve
d) Median nerve
Explanation (Answer: b)
The musculocutaneous nerve (C5–C7) innervates the biceps brachii, brachialis, and coracobrachialis muscles. It pierces the coracobrachialis and descends between the biceps and brachialis. It continues as the lateral cutaneous nerve of the forearm, supplying skin sensation. Damage to this nerve leads to weakened elbow flexion and supination.
5. The main action of the triceps brachii is:
a) Flexion of the elbow
b) Extension of the elbow
c) Supination of the forearm
d) Pronation of the forearm
Explanation (Answer: b)
The triceps brachii is the chief extensor of the elbow joint. Its long head arises from the infraglenoid tubercle, while lateral and medial heads originate from the posterior humerus. It inserts on the olecranon process and is innervated by the radial nerve. It stabilizes the elbow during extension and pushing actions.
6. A 25-year-old male presents with difficulty in supinating his forearm. Which muscle is likely injured?
a) Pronator teres
b) Biceps brachii
c) Brachialis
d) Brachioradialis
Explanation (Answer: b)
Injury to the biceps brachii leads to weakness in both forearm flexion and supination, especially when the elbow is flexed. The biceps acts as a powerful supinator in this position. The pronator teres pronates, not supinates. Brachialis flexes the forearm but doesn’t participate in rotation.
7. The bicipital aponeurosis provides protection to which structure during venipuncture?
a) Median nerve
b) Brachial artery
c) Radial artery
d) Ulnar nerve
Explanation (Answer: b)
The bicipital aponeurosis lies over the brachial artery and median nerve in the cubital fossa. It protects these structures during venipuncture in the median cubital vein. This broad aponeurosis extends medially from the biceps tendon and blends with the forearm fascia, reinforcing the fossa’s roof and ensuring vascular safety.
8. A patient with musculocutaneous nerve injury will have difficulty performing:
a) Extension at the elbow
b) Flexion and supination of the forearm
c) Pronation of the forearm
d) Wrist extension
Explanation (Answer: b)
The musculocutaneous nerve supplies the anterior compartment of the arm (biceps, brachialis, coracobrachialis). Injury impairs forearm flexion and supination. The biceps loses its function as both a flexor and supinator. Triceps and wrist extensors remain unaffected as they are supplied by the radial nerve.
9. Which of the following best describes the function of the brachioradialis muscle?
a) Supinates the forearm
b) Flexes the elbow in mid-pronation
c) Extends the forearm
d) Pronates the forearm
Explanation (Answer: b)
The brachioradialis flexes the forearm most effectively when the forearm is in a mid-prone (neutral) position. It acts as a stabilizer during rapid flexion movements and is supplied by the radial nerve. It doesn’t participate in pronation or supination. Clinically, its reflex tests the C6 spinal segment.
10. A rupture of the long head of the biceps tendon results in:
a) Popeye deformity
b) Winged scapula
c) Claw hand
d) Drop wrist
Explanation (Answer: a)
A rupture of the long head of biceps brachii causes a bulge in the lower arm called the “Popeye deformity.” This occurs due to retraction of the muscle belly. It leads to weakness in flexion and supination. The condition is commonly seen in athletes and older adults with tendon degeneration.
Chapter: Upper Limb Anatomy; Topic: Cubital Fossa; Subtopic: Boundaries and Contents
Keyword Definitions:
• Cubital Fossa: A triangular depression on the anterior aspect of the elbow.
• Brachioradialis: A forearm muscle forming the lateral boundary of the cubital fossa.
• Pronator Teres: A forearm muscle forming the medial boundary of the cubital fossa.
• Brachialis: Lies deep to the biceps and forms the floor of the cubital fossa.
• Biceps Brachii: Lies superficially and forms part of the roof over the cubital fossa.
Lead Question - 2015
Lateral boundary of cubital fossa is formed by:
a) Brachioradialis
b) Pronator teres
c) Brachialis
d) Biceps
Explanation (Answer: a)
The lateral boundary of the cubital fossa is formed by the brachioradialis muscle, while the medial boundary is formed by pronator teres. The floor is formed by brachialis and supinator, and the roof by skin, fascia, and bicipital aponeurosis. It contains the median nerve, brachial artery, and tendon of biceps brachii.
1. The medial boundary of the cubital fossa is formed by:
a) Brachioradialis
b) Pronator teres
c) Biceps brachii
d) Brachialis
Explanation (Answer: b)
The pronator teres forms the medial boundary of the cubital fossa. It originates from the medial epicondyle of the humerus and coronoid process of the ulna. The muscle assists in pronation and flexion of the forearm. Its relationship with the fossa helps guide clinical venipuncture and nerve block procedures.
2. The cubital fossa contains all of the following except:
a) Median nerve
b) Radial nerve
c) Brachial artery
d) Ulnar nerve
Explanation (Answer: d)
The ulnar nerve does not pass through the cubital fossa. It runs posterior to the medial epicondyle, in the groove between it and the olecranon. The cubital fossa contains the median nerve, brachial artery (dividing into radial and ulnar arteries), biceps tendon, and radial nerve (laterally under brachioradialis).
3. The floor of the cubital fossa is formed by:
a) Pronator teres and brachioradialis
b) Brachialis and supinator
c) Biceps and brachialis
d) Flexor carpi radialis and supinator
Explanation (Answer: b)
The floor of the cubital fossa is formed medially by brachialis and laterally by supinator. Brachialis originates from the anterior surface of the humerus, while supinator wraps around the proximal radius. These muscles support the deeper neurovascular structures during elbow flexion and provide protection during venipuncture.
4. Roof of the cubital fossa includes:
a) Skin and fascia only
b) Skin, fascia, and bicipital aponeurosis
c) Fascia and brachialis
d) Skin and radial nerve
Explanation (Answer: b)
The roof of the cubital fossa comprises the skin, superficial fascia, deep fascia reinforced by the bicipital aponeurosis, and the median cubital vein crossing superficially. This structure is clinically important for intravenous injections and blood sampling. The aponeurosis protects the underlying brachial artery and median nerve from accidental injury.
5. Which structure passes most superficially in the cubital fossa?
a) Brachial artery
b) Median nerve
c) Median cubital vein
d) Biceps tendon
Explanation (Answer: c)
The median cubital vein passes superficially in the roof of the cubital fossa, making it the preferred site for venipuncture. It connects the cephalic and basilic veins. Beneath it lie the bicipital aponeurosis and deeper structures like the brachial artery and median nerve, protected during superficial procedures.
6. A patient presents with a supracondylar fracture of the humerus. Which cubital fossa structure is most at risk?
a) Brachial artery
b) Ulnar nerve
c) Radial nerve
d) Median cubital vein
Explanation (Answer: a)
The brachial artery is most at risk in supracondylar fractures of the humerus due to its deep location within the cubital fossa. Injury may lead to ischemic contracture of the forearm muscles (Volkmann’s ischemic contracture). Prompt vascular assessment is essential to prevent irreversible damage and functional impairment.
7. Which nerve divides into superficial and deep branches within the cubital fossa?
a) Median nerve
b) Ulnar nerve
c) Radial nerve
d) Musculocutaneous nerve
Explanation (Answer: c)
The radial nerve divides into superficial and deep branches at the level of the cubital fossa. The superficial branch continues distally under brachioradialis for sensory supply, while the deep branch (posterior interosseous nerve) pierces the supinator muscle to supply extensor muscles of the forearm.
8. Which of the following muscles does NOT form part of the cubital fossa boundary?
a) Brachialis
b) Brachioradialis
c) Pronator teres
d) Supinator
Explanation (Answer: d)
Supinator forms part of the floor, not the boundary, of the cubital fossa. The lateral boundary is formed by brachioradialis, the medial by pronator teres, the floor by brachialis and supinator, and the roof by bicipital aponeurosis and skin. Supinator’s role is mainly in forearm supination, not boundary formation.
9. During venipuncture in the cubital fossa, which structure lies deep to the median cubital vein and must be protected?
a) Brachial artery
b) Radial artery
c) Ulnar nerve
d) Cephalic vein
Explanation (Answer: a)
The brachial artery lies deep to the median cubital vein and bicipital aponeurosis in the cubital fossa. The aponeurosis acts as a protective layer, preventing accidental arterial puncture during venipuncture. Awareness of this relationship is essential for safe medical procedures and avoiding hematoma formation or vascular injury.
10. Which artery divides into radial and ulnar branches in the cubital fossa?
a) Axillary artery
b) Brachial artery
c) Subclavian artery
d) Deep brachial artery
Explanation (Answer: b)
The brachial artery divides into the radial and ulnar arteries at the level of the neck of the radius within the cubital fossa. This division marks the transition from arm to forearm circulation. Clinically, palpation of the brachial artery pulse in the fossa aids in assessing upper limb blood flow.
Chapter: Upper Limb Anatomy; Topic: Muscles of Hand; Subtopic: Lumbricals
Keyword Definitions:
• Lumbricals: Small intrinsic muscles of the hand arising from tendons of flexor digitorum profundus and inserting into extensor expansion.
• MCP Joints: Metacarpophalangeal joints, where the fingers meet the hand.
• IP Joints: Interphalangeal joints of the fingers, responsible for flexion and extension movements.
• Ulnar Nerve: Supplies intrinsic muscles of hand except the thenar and first two lumbricals.
• Median Nerve: Supplies first two lumbricals and most forearm flexors.
Lead Question - 2015
True about lumbricals is
a) Flex IP joints and extends MCP joint
b) 1st and 2nd supplied by radial nerve
c) 3 and 4 supplied by superficial branch of ulnar
d) Origin from tendons of flexor digitorum profundus
Explanation (Answer: d)
Lumbricals originate from tendons of flexor digitorum profundus and insert into the extensor expansion. They flex the metacarpophalangeal (MCP) joints and extend interphalangeal (IP) joints. The first and second lumbricals are supplied by the median nerve, while the third and fourth are supplied by the deep branch of the ulnar nerve.
1. Which nerve supplies the 3rd and 4th lumbricals?
a) Median nerve
b) Radial nerve
c) Ulnar nerve
d) Musculocutaneous nerve
Explanation (Answer: c)
The third and fourth lumbricals are supplied by the deep branch of the ulnar nerve. This branch originates in the palm after passing through Guyon’s canal. The ulnar nerve provides motor supply to most intrinsic hand muscles, contributing to finger flexion at MCP and extension at IP joints.
2. Which of the following actions best describes lumbrical function?
a) Flex MCP and extend IP joints
b) Extend MCP and flex IP joints
c) Flex both MCP and IP joints
d) Extend both MCP and IP joints
Explanation (Answer: a)
Lumbricals act simultaneously to flex the MCP joints and extend the IP joints by inserting into the extensor expansion. This unique dual function enables precise hand movements such as writing and typing. Their coordination is essential for normal finger dexterity and smooth control of digital motion.
3. Lumbricals arise from which tendon?
a) Flexor digitorum profundus
b) Flexor digitorum superficialis
c) Extensor digitorum
d) Palmaris longus
Explanation (Answer: a)
All four lumbricals arise from the tendons of flexor digitorum profundus. The first and second originate from the lateral two tendons (unipennate), while the third and fourth are bipennate. They pass dorsally around the fingers to insert into extensor expansions, enabling coordinated flexion and extension.
4. Which of the following lumbricals are unipennate?
a) 1st and 2nd
b) 3rd and 4th
c) All four
d) None
Explanation (Answer: a)
The first and second lumbricals are unipennate, originating from one tendon each of flexor digitorum profundus. The third and fourth are bipennate, arising from adjacent tendons. This structural difference correlates with their nerve supply: the median nerve innervates unipennate, and the ulnar nerve supplies bipennate lumbricals.
5. Which lumbrical receives dual nerve supply occasionally?
a) First
b) Second
c) Third
d) Fourth
Explanation (Answer: c)
In some individuals, the third lumbrical receives dual innervation from both median and ulnar nerves, a variation that ensures preserved hand function if one nerve is compromised. Such anatomical variations hold clinical significance during nerve injury assessments and reconstructive surgeries of the palm and fingers.
6. A patient with carpal tunnel syndrome shows weakness in which lumbricals?
a) 1st and 2nd
b) 3rd and 4th
c) All lumbricals
d) None
Explanation (Answer: a)
Carpal tunnel syndrome compresses the median nerve beneath the flexor retinaculum, affecting its motor branches to the first and second lumbricals. This results in weakened flexion at MCP and impaired extension at IP joints of the index and middle fingers, characteristic of median nerve palsy.
7. Injury to the ulnar nerve at the wrist will affect which lumbricals?
a) 1st and 2nd
b) 3rd and 4th
c) Only 4th
d) None
Explanation (Answer: b)
Ulnar nerve injury at the wrist affects the deep branch, leading to paralysis of the third and fourth lumbricals. This produces clawing of the ring and little fingers due to unopposed extensor and flexor activity, a key diagnostic feature in ulnar nerve palsy at the hand level.
8. Which statement about lumbrical muscle insertion is true?
a) Inserts into extensor expansion
b) Inserts into flexor retinaculum
c) Inserts into carpal bones
d) Inserts into flexor digitorum superficialis
Explanation (Answer: a)
Lumbricals insert into the lateral side of the extensor expansion of corresponding digits. This strategic insertion allows them to coordinate flexion at MCP and extension at IP joints, facilitating delicate movements such as gripping and writing. Their action is vital for balanced finger coordination and dexterity.
9. A patient with ulnar nerve injury presents with claw hand. Which lumbricals are affected?
a) 3rd and 4th
b) 1st and 2nd
c) All four
d) None
Explanation (Answer: a)
Claw hand results from paralysis of the 3rd and 4th lumbricals supplied by the ulnar nerve. The MCP joints are hyperextended, and IP joints are flexed due to loss of lumbrical action. Early physiotherapy helps restore balance between flexor and extensor forces, minimizing deformity progression.
10. Which of the following statements regarding lumbricals is false?
a) Flex MCP and extend IP joints
b) Supplied by both median and ulnar nerves
c) Originate from FDP tendons
d) Supplied by radial nerve
Explanation (Answer: d)
Lumbricals are not supplied by the radial nerve. The first two receive innervation from the median nerve, and the last two from the ulnar nerve. The radial nerve supplies only extensor muscles in the forearm, not intrinsic hand muscles. Thus, option (d) is false and incorrect.
Chapter: Upper Limb Anatomy; Topic: Muscles of Scapula; Subtopic: Attachments on Medial Border of Scapula
Keyword Definitions:
• Scapula: Flat triangular bone forming the posterior part of the shoulder girdle.
• Medial Border: The edge of the scapula closest to the vertebral column, serving as an attachment for several muscles.
• Serratus Anterior: Muscle aiding in protraction of the scapula.
• Levator Scapulae: Elevates the scapula.
• Rhomboid Major: Retracts and stabilizes the scapula.
• Teres Major: A muscle attached to the inferior angle and lateral border of the scapula, not the medial border.
Lead Question – 2015
Not attached on medial border of scapula?
a) Serratus anterior
b) Levator scapulae
c) Rhomboideus major
d) Teres major
Explanation: The Teres major muscle arises from the dorsal surface of the inferior angle and the lower part of the lateral border of the scapula, not the medial border. The medial border gives attachment to the levator scapulae, rhomboid major, rhomboid minor, and serratus anterior. Hence, the correct answer is d) Teres major.
Guessed Questions:
1. Which muscle inserts into the medial border of the scapula?
a) Teres major
b) Rhomboid minor
c) Latissimus dorsi
d) Deltoid
Explanation: The Rhomboid minor inserts into the medial border of the scapula at the level of the spine. It retracts the scapula and stabilizes it. The other muscles attach to different parts of the humerus or scapula. Hence, the correct answer is b) Rhomboid minor.
2. Which nerve supplies the serratus anterior muscle?
a) Dorsal scapular nerve
b) Long thoracic nerve
c) Axillary nerve
d) Suprascapular nerve
Explanation: The Long thoracic nerve supplies the serratus anterior muscle, enabling scapular protraction and upward rotation. Damage to this nerve leads to a winged scapula. The other nerves supply different muscles of the shoulder girdle. Hence, the correct answer is b) Long thoracic nerve.
3. Paralysis of which muscle causes winging of the scapula?
a) Rhomboid major
b) Levator scapulae
c) Serratus anterior
d) Teres major
Explanation: Winging of the scapula occurs due to paralysis of the serratus anterior muscle from long thoracic nerve injury. The scapula protrudes posteriorly when pushing against resistance. This condition commonly occurs after axillary surgery or trauma. Hence, the correct answer is c) Serratus anterior.
4. Which muscle elevates the scapula?
a) Levator scapulae
b) Serratus anterior
c) Pectoralis minor
d) Teres minor
Explanation: The Levator scapulae elevates the medial border of the scapula and assists in downward rotation. It is innervated by the dorsal scapular nerve. The other muscles perform protraction or depression functions. Hence, the correct answer is a) Levator scapulae.
5. The dorsal scapular nerve supplies all except?
a) Rhomboid minor
b) Levator scapulae
c) Rhomboid major
d) Serratus anterior
Explanation: The Serratus anterior is supplied by the long thoracic nerve, not the dorsal scapular nerve. The dorsal scapular nerve innervates the rhomboid major, rhomboid minor, and levator scapulae muscles. Hence, the correct answer is d) Serratus anterior.
6. A patient with a fractured inferior angle of the scapula may have difficulty in?
a) Arm elevation
b) Scapular protraction
c) Scapular depression
d) Arm abduction beyond 90°
Explanation: The Teres major originates from the inferior angle of the scapula, and its function includes medial rotation and adduction of the arm. Fracture at this site limits these actions. Therefore, arm adduction and medial rotation are primarily affected. Hence, the correct answer is a) Arm elevation.
7. The scapular notch transmits which structure?
a) Suprascapular artery
b) Suprascapular nerve
c) Axillary nerve
d) Circumflex scapular artery
Explanation: The Suprascapular nerve passes through the scapular notch beneath the superior transverse scapular ligament. The artery passes above the ligament. This region is clinically significant due to entrapment neuropathies. Hence, the correct answer is b) Suprascapular nerve.
8. Which muscle helps in downward rotation of the scapula?
a) Serratus anterior
b) Pectoralis minor
c) Trapezius (upper fibers)
d) Deltoid
Explanation: The Pectoralis minor assists in the downward rotation of the scapula by pulling the coracoid process inferiorly and medially. It also aids in scapular depression during forced inspiration. Hence, the correct answer is b) Pectoralis minor.
9. A patient presents with difficulty retracting the scapula. Which muscle is likely affected?
a) Rhomboid major
b) Levator scapulae
c) Serratus anterior
d) Pectoralis minor
Explanation: The Rhomboid major muscle retracts and stabilizes the scapula. Damage to its nerve supply (dorsal scapular nerve) leads to weakened retraction. Hence, the correct answer is a) Rhomboid major.
10. The inferior angle of the scapula is an important landmark for identifying which level?
a) T4 vertebra
b) T7 vertebra
c) T10 vertebra
d) T12 vertebra
Explanation: The Inferior angle of the scapula lies opposite the T7 vertebral spine when the arm is by the side. It serves as an important surface landmark during clinical examinations and imaging. Hence, the correct answer is b) T7 vertebra.
Chapter: Upper Limb Anatomy;Topic: Muscles of the Arm; Subtopic: Origin and Insertion of Muscles around Scapula
Keyword Definitions:
Infraglenoid Tubercle: A small prominence located below the glenoid cavity of the scapula, serving as the point of origin for the long head of the triceps brachii muscle.
Triceps Brachii: A large muscle at the back of the upper arm with three heads (long, lateral, and medial) responsible for forearm extension at the elbow.
Biceps Brachii: A two-headed muscle on the anterior aspect of the arm involved in flexion and supination of the forearm.
Scapula: The flat triangular bone of the shoulder that provides attachment sites for multiple muscles.
Lead Question – 2015
Which of the following arises from infraglenoid tubercle –
a) Long head of biceps
b) Long head of triceps
c) Short head of biceps
d) Coracobrachialis
Explanation: The long head of the triceps brachii originates from the infraglenoid tubercle of the scapula. It descends between the teres minor and teres major, joining other heads to form a common tendon inserting into the olecranon process of the ulna. It extends the elbow and aids shoulder extension. Answer: (b) Long head of triceps.
1. The long head of biceps arises from:
a) Supraglenoid tubercle
b) Infraglenoid tubercle
c) Coracoid process
d) Glenoid cavity
Explanation: The long head of the biceps brachii originates from the supraglenoid tubercle of the scapula. Its tendon passes through the shoulder joint cavity and the intertubercular groove of the humerus, stabilized by the transverse humeral ligament. Answer: (a) Supraglenoid tubercle.
2. The short head of the biceps brachii arises from:
a) Coracoid process
b) Acromion
c) Clavicle
d) Scapular spine
Explanation: The short head of the biceps brachii originates from the tip of the coracoid process along with the coracobrachialis. It joins the long head to insert into the radial tuberosity, enabling flexion of the elbow and supination of the forearm. Answer: (a) Coracoid process.
3. Which muscle of the arm is supplied by the radial nerve?
a) Triceps brachii
b) Biceps brachii
c) Coracobrachialis
d) Brachialis
Explanation: The radial nerve supplies the triceps brachii, anconeus, and posterior compartment muscles of the arm and forearm. It facilitates elbow extension and wrist movement. Damage results in wrist drop due to paralysis of extensors. Answer: (a) Triceps brachii.
4. Clinical: A patient with posterior cord injury presents with inability to extend the elbow. Which nerve is affected?
a) Musculocutaneous nerve
b) Radial nerve
c) Axillary nerve
d) Median nerve
Explanation: The radial nerve, derived from the posterior cord of the brachial plexus, innervates the triceps brachii. Injury leads to loss of elbow extension and wrist drop. Sensory loss occurs over the posterior arm and forearm. Answer: (b) Radial nerve.
5. Which of the following muscles extends the forearm at the elbow joint?
a) Biceps brachii
b) Brachialis
c) Triceps brachii
d) Coracobrachialis
Explanation: The triceps brachii, particularly its long and lateral heads, are the primary extensors of the elbow joint. The medial head provides sustained contraction during extension. The triceps also aids shoulder extension. Answer: (c) Triceps brachii.
6. Clinical: A person receives a penetrating injury near the spiral groove. Which movement is most affected?
a) Shoulder flexion
b) Elbow extension
c) Wrist extension
d) Elbow flexion
Explanation: Radial nerve injury in the spiral groove spares the long head of triceps but paralyzes extensors of the wrist and fingers, resulting in wrist drop. Shoulder and elbow functions are relatively preserved. Answer: (c) Wrist extension.
7. Which head of the triceps is the deepest?
a) Long head
b) Lateral head
c) Medial head
d) None of the above
Explanation: The medial head of the triceps is the deepest and lies beneath the other two heads. It originates from the posterior surface of the humerus below the radial groove and provides stability during prolonged contraction. Answer: (c) Medial head.
8. Which artery supplies the triceps brachii?
a) Brachial artery
b) Deep brachial artery
c) Axillary artery
d) Subclavian artery
Explanation: The deep brachial artery (profunda brachii), a branch of the brachial artery, supplies the triceps brachii. It accompanies the radial nerve in the spiral groove and provides collateral circulation around the elbow. Answer: (b) Deep brachial artery.
9. Clinical: A patient is unable to adduct and extend the shoulder after injury to the axilla. Which muscle is likely involved?
a) Teres minor
b) Triceps long head
c) Biceps short head
d) Supraspinatus
Explanation: The long head of the triceps brachii assists in shoulder adduction and extension. Damage to the infraglenoid region or radial nerve impairs these movements. Answer: (b) Triceps long head.
10. The triceps tendon inserts into which bony landmark?
a) Olecranon process
b) Coronoid process
c) Radial tuberosity
d) Ulnar tuberosity
Explanation: All three heads of the triceps brachii converge to insert on the olecranon process of the ulna. This insertion allows efficient extension at the elbow joint during pushing or throwing actions. Answer: (a) Olecranon process.
Chapter: Upper Limb Anatomy; Topic: Scapular Muscles; Subtopic: Levator Scapulae Muscle
Keyword Definitions:
Levator Scapulae: A muscle originating from the transverse processes of C1–C4 vertebrae that elevates the scapula.
Scapula: A flat triangular bone connecting the humerus with the clavicle, forming the shoulder blade.
Insertion: The point where a muscle attaches to a bone, usually the movable part during contraction.
Medial Border: The edge of the scapula nearest the vertebral column.
Lead Question – 2015
Insertion of levator scapulae is ?
a) Lateral border of scapula
b) Supralateral part of scapula
c) Superior part of medial scapula border
d) Inferior angle of scapula
Explanation: Levator scapulae inserts into the superior part of the medial border of the scapula, between the superior angle and the root of the spine of scapula. It acts to elevate the scapula and assist in downward rotation. The muscle is innervated by the dorsal scapular nerve (C5) and cervical nerves (C3, C4).
1. Which nerve supplies the levator scapulae muscle?
a) Long thoracic nerve
b) Dorsal scapular nerve
c) Axillary nerve
d) Spinal accessory nerve
Explanation: Levator scapulae is primarily supplied by the dorsal scapular nerve (C5) and cervical nerves (C3, C4). This innervation enables elevation and rotation of the scapula during shoulder movements. Injury to this nerve weakens scapular elevation, but trapezius partially compensates due to its similar action.
2. Which of the following muscles attaches to the superior angle of the scapula?
a) Rhomboid major
b) Rhomboid minor
c) Levator scapulae
d) Serratus anterior
Explanation: Levator scapulae inserts at the superior angle and upper medial border of the scapula, elevating it and assisting in rotation. Rhomboids attach to the medial border below this point. Serratus anterior inserts along the anterior surface of the medial border.
3. Clinical sign of dorsal scapular nerve injury includes:
a) Winged scapula
b) Drooping of shoulder
c) Difficulty elevating scapula
d) Flattened deltoid
Explanation: Dorsal scapular nerve injury leads to weakness in levator scapulae and rhomboids, resulting in difficulty elevating and retracting the scapula. However, winged scapula occurs due to long thoracic nerve injury affecting serratus anterior, distinguishing it from this condition.
4. Levator scapulae arises from which vertebrae?
a) C1–C4 transverse processes
b) C5–C7 spinous processes
c) C2–C6 spinous processes
d) C1–C3 anterior tubercles
Explanation: The levator scapulae originates from the transverse processes of the first four cervical vertebrae (C1–C4). Its fibers descend to insert into the medial border of the scapula, elevating the bone and rotating it downward during shoulder movements.
5. In a patient with levator scapulae paralysis, which movement is most affected?
a) Shoulder abduction
b) Scapular elevation
c) Shoulder flexion
d) Arm medial rotation
Explanation: Levator scapulae is responsible for elevation and downward rotation of the scapula. Paralysis leads to difficulty in lifting the shoulder and reduced scapular stability. Other shoulder movements, like abduction and flexion, remain largely unaffected due to intact deltoid and trapezius action.
6. Which muscle acts synergistically with levator scapulae during scapular elevation?
a) Trapezius
b) Serratus anterior
c) Pectoralis minor
d) Latissimus dorsi
Explanation: The upper fibers of the trapezius muscle assist the levator scapulae in elevating the scapula. Both muscles coordinate for shrugging or lifting the shoulders. Serratus anterior acts oppositely by protracting and rotating the scapula upward.
7. Which artery accompanies the dorsal scapular nerve near the levator scapulae?
a) Subscapular artery
b) Dorsal scapular artery
c) Thoracodorsal artery
d) Transverse cervical artery
Explanation: The dorsal scapular artery runs alongside the dorsal scapular nerve, supplying the levator scapulae, rhomboids, and part of the trapezius. It provides vascular support to muscles stabilizing the scapula, ensuring their function during shoulder and neck movements.
8. A patient has pain along the upper neck and shoulder due to levator scapulae spasm. Which posture worsens pain?
a) Neck flexion
b) Shoulder elevation
c) Prolonged head rotation
d) Arm abduction
Explanation: Prolonged head rotation or looking down while sitting strains levator scapulae, leading to myofascial pain. This is common in people working long hours at computers. Stretching and posture correction relieve symptoms. Shoulder elevation briefly relaxes the muscle but chronic tension aggravates it.
9. Levator scapulae assists which other muscle in downward rotation of the scapula?
a) Pectoralis major
b) Rhomboid minor
c) Latissimus dorsi
d) Trapezius lower fibers
Explanation: Levator scapulae works with rhomboids to rotate the scapula downward. Both pull the medial border upward and inward. This movement is vital for returning the elevated scapula to rest after shoulder abduction. Dysfunction causes imbalance in scapular rotation and posture.
10. During neck dissection surgery, injury to the dorsal scapular nerve may affect:
a) Trapezius and sternocleidomastoid
b) Rhomboids and levator scapulae
c) Deltoid and teres minor
d) Serratus anterior and pectoralis minor
Explanation: Dorsal scapular nerve injury during neck dissection weakens levator scapulae and rhomboids, leading to scapular droop and impaired retraction. Trapezius remains intact as it is supplied by the spinal accessory nerve, differentiating the pattern of muscle weakness clinically.
11. Levator scapulae stabilizes the scapula during which movement?
a) Climbing
b) Throwing
c) Lifting heavy weight
d) Writing
Explanation: During climbing or lifting, levator scapulae stabilizes and elevates the scapula, preventing downward displacement. It maintains scapular position, allowing efficient upper limb force transmission. Dysfunction reduces lifting power and neck stability, emphasizing its postural importance in upper limb mechanics.
Topic: Muscle Physiology; Subtopic: Muscle Filament Structure and Function
Keyword Definitions:
Actin: Thin filament protein forming the backbone of the I-band and interacting with myosin for muscle contraction.
Myosin: Thick filament protein responsible for generating contractile force in muscles.
Titin: Giant elastic protein anchoring myosin to the Z-line, maintaining sarcomere alignment and elasticity.
Actinin: Protein that anchors actin filaments to the Z-line in the sarcomere.
Tropomyosin: Regulatory protein covering myosin-binding sites on actin at rest.
Troponin: Calcium-binding protein complex regulating interaction between actin and myosin.
Sarcomere: Basic contractile unit of a muscle fiber, defined between two Z-lines.
Lead Question – 2014
Myosin and actin filaments are kept in place by
a) Tropomyosin
b) Troponin
c) Actinin
d) Titin
Explanation: The correct answer is Titin. Titin is a large elastic protein that connects the Z-line to the M-line, anchoring thick myosin filaments and maintaining sarcomere stability. It provides passive elasticity to muscle and helps restore resting length after contraction. Without titin, myosin alignment and muscle elasticity are severely disrupted.
1) Which protein anchors thin actin filaments to the Z-line?
a) Titin
b) Actinin
c) Troponin
d) Myomesin
Explanation: The correct answer is Actinin. α-Actinin is a structural protein found at the Z-line of sarcomeres that cross-links actin filaments, keeping them in alignment and stabilizing the Z-disc. It ensures organized contraction and uniform tension distribution during muscle shortening.
2) Which of the following proteins prevents myosin from binding to actin in a resting muscle?
a) Troponin
b) Tropomyosin
c) Titin
d) Nebulin
Explanation: The correct answer is Tropomyosin. Tropomyosin is a regulatory protein that lies along the actin filament, covering the binding sites for myosin heads. When calcium levels rise, troponin moves tropomyosin aside, allowing cross-bridge formation and contraction to occur.
3) A mutation in the titin gene primarily affects which muscle property?
a) Excitability
b) Elasticity
c) Contractility
d) Conductivity
Explanation: The correct answer is Elasticity. Titin acts as a molecular spring that contributes to passive elasticity of muscles. Mutations cause muscle weakness, cardiomyopathies, and impaired sarcomere alignment, reducing the muscle’s ability to return to its original length after stretch.
4) Which of the following is responsible for the calcium-mediated regulation of actin-myosin interaction?
a) Titin
b) Troponin
c) Actinin
d) Myomesin
Explanation: The correct answer is Troponin. The troponin complex, composed of TnI, TnT, and TnC, regulates actin-myosin binding. When calcium binds to TnC, a conformational change removes tropomyosin from actin’s binding site, enabling cross-bridge formation and muscle contraction.
5) Which part of the sarcomere shortens during muscle contraction?
a) A-band
b) I-band
c) M-line
d) Z-line
Explanation: The correct answer is I-band. During contraction, thin actin filaments slide over thick myosin filaments, reducing the length of the I-band and H-zone. The A-band remains constant, as it corresponds to the length of myosin filaments, which do not shorten.
6) A patient with a titin deficiency presents with muscle weakness and cardiac dysfunction. Which disease is likely?
a) Duchenne muscular dystrophy
b) Limb-girdle muscular dystrophy
c) Dilated cardiomyopathy
d) Myasthenia gravis
Explanation: The correct answer is Dilated cardiomyopathy. Titin mutations disrupt sarcomere integrity in cardiac muscle, leading to weakened contraction and chamber dilation. This causes reduced ejection fraction and systolic heart failure, seen in familial titinopathies.
7) Which of the following proteins determines the precise length of actin filaments in skeletal muscle?
a) Nebulin
b) Titin
c) Troponin
d) Myomesin
Explanation: The correct answer is Nebulin. Nebulin acts as a molecular ruler for actin filaments, maintaining uniform thin filament length. It stabilizes actin and contributes to efficient sarcomere contraction and alignment across muscle fibers.
8) A biopsy of skeletal muscle reveals disorganized sarcomeres with detached myosin filaments. Which protein is defective?
a) Troponin
b) Actinin
c) Titin
d) Tropomyosin
Explanation: The correct answer is Titin. Titin maintains myosin filament alignment within the sarcomere. Its deficiency causes instability of thick filaments, loss of elasticity, and disorganization, leading to myopathies characterized by weakness and structural degeneration.
9) Which ion binds to troponin to initiate muscle contraction?
a) Sodium
b) Potassium
c) Calcium
d) Magnesium
Explanation: The correct answer is Calcium. Calcium ions released from the sarcoplasmic reticulum bind to troponin C, causing tropomyosin to shift from actin-binding sites. This allows myosin heads to attach to actin, initiating cross-bridge cycling and muscle contraction.
10) During relaxation of muscle fibers, which mechanism helps restore calcium levels in the sarcoplasmic reticulum?
a) SERCA pump
b) Na⁺/K⁺ ATPase
c) Voltage-gated channels
d) Ryanodine receptor
Explanation: The correct answer is SERCA pump. The sarcoplasmic/endoplasmic reticulum Ca²⁺-ATPase (SERCA) actively pumps calcium back into the SR after contraction, reducing cytoplasmic calcium levels, detaching cross-bridges, and promoting muscle relaxation.
Chapter: Head and Neck Anatomy; Topic: Muscles of the Tongue; Subtopic: Embryological Origin of Tongue Muscles
Keyword Definitions:
• Occipital Myotomes: Embryonic muscle segments that give rise to intrinsic and most extrinsic muscles of the tongue.
• Palatoglossus: Muscle of the tongue derived from the 4th pharyngeal arch and supplied by the vagus nerve (cranial part of accessory nerve).
• Hypoglossal Nerve: Motor nerve that supplies all tongue muscles except palatoglossus.
• Pharyngeal Arch: Embryonic structures that form muscles and nerves of the face and neck.
Lead Question – 2014
Tongue muscle which is not developed from occipital myotome?
a) Styloglossus
b) Hyoglossus
c) Genioglossus
d) Palatoglossus
Explanation:
All muscles of the tongue, except the palatoglossus, develop from occipital myotomes and are supplied by the hypoglossal nerve. Palatoglossus arises from the fourth pharyngeal arch and is innervated by the pharyngeal branch of the vagus nerve. Hence, palatoglossus is the only tongue muscle not derived from occipital myotome. Correct answer: (d).
1) Which nerve supplies all intrinsic and extrinsic muscles of the tongue except one?
a) Glossopharyngeal nerve
b) Hypoglossal nerve
c) Vagus nerve
d) Trigeminal nerve
Explanation:
The hypoglossal nerve (cranial nerve XII) supplies all intrinsic and extrinsic tongue muscles except the palatoglossus, which is supplied by the vagus nerve via the pharyngeal plexus. This distinction is important in neurological testing of tongue deviation. Hence, (b) is correct.
2) Which tongue muscle is supplied by the vagus nerve?
a) Genioglossus
b) Styloglossus
c) Hyoglossus
d) Palatoglossus
Explanation:
The palatoglossus muscle is the only tongue muscle supplied by the vagus nerve through the pharyngeal plexus. It functions to elevate the posterior part of the tongue during swallowing. All other tongue muscles receive innervation from the hypoglossal nerve. Thus, (d) is correct.
3) The hypoglossal nerve is derived from which embryonic structure?
a) First pharyngeal arch
b) Occipital myotomes
c) Neural crest cells
d) Mandibular prominence
Explanation:
The hypoglossal nerve develops from occipital myotomes, which migrate anteriorly to form the intrinsic and extrinsic muscles of the tongue. This embryological link explains its motor supply to the tongue. Therefore, (b) is correct.
4) Which muscle forms the bulk of the tongue and is responsible for protrusion?
a) Genioglossus
b) Hyoglossus
c) Styloglossus
d) Palatoglossus
Explanation:
The genioglossus is the largest and most powerful tongue muscle, responsible for protrusion and depression of the tongue. It arises from the superior mental spine of the mandible. Injury to its nerve supply causes tongue deviation. Hence, (a) is correct.
5) Clinical Case: A patient with tongue deviation to the right has a lesion of which nerve?
a) Left hypoglossal nerve
b) Right hypoglossal nerve
c) Glossopharyngeal nerve
d) Vagus nerve
Explanation:
Unilateral hypoglossal nerve paralysis causes deviation of the tongue toward the affected side upon protrusion due to unopposed action of the opposite genioglossus. Hence, deviation to the right indicates right hypoglossal nerve damage. Answer: (b).
6) Clinical Case: During swallowing, the posterior part of the tongue is elevated by which muscle?
a) Genioglossus
b) Palatoglossus
c) Hyoglossus
d) Styloglossus
Explanation:
Palatoglossus elevates the posterior tongue to aid swallowing, closing the oropharyngeal isthmus. It’s supplied by the vagus nerve via the pharyngeal plexus. This function distinguishes it from other extrinsic muscles. Thus, (b) is correct.
7) Clinical Case: In a patient with lesion of the vagus nerve, which tongue function is most affected?
a) Protrusion
b) Retraction
c) Elevation of posterior tongue
d) Depression
Explanation:
The vagus nerve supplies the palatoglossus muscle, which elevates the posterior tongue during swallowing. Vagus nerve damage leads to dysphagia and poor closure of the oropharyngeal isthmus. Therefore, (c) is correct.
8) The styloglossus muscle acts to:
a) Protrude tongue
b) Retract and elevate tongue
c) Depress tongue
d) Flatten tongue
Explanation:
The styloglossus retracts and elevates the tongue. It originates from the styloid process and inserts into the lateral tongue. This muscle assists in swallowing and repositioning the tongue after protrusion. Hence, (b) is correct.
9) The hyoglossus muscle arises from:
a) Hyoid bone
b) Styloid process
c) Mandible
d) Palatine aponeurosis
Explanation:
The hyoglossus originates from the greater horn and body of the hyoid bone. It depresses the sides of the tongue and assists in retracting it. It is innervated by the hypoglossal nerve. Hence, (a) is correct.
10) Clinical Case: During thyroid surgery, accidental damage to the hypoglossal nerve would affect which function?
a) Vocal cord movement
b) Tongue movement
c) Swallowing reflex only
d) Soft palate elevation
Explanation:
The hypoglossal nerve controls intrinsic and extrinsic tongue muscles (except palatoglossus). Damage leads to deviation, atrophy, and difficulty in speech and swallowing. Vocal cord movements remain unaffected as they depend on recurrent laryngeal nerve. Hence, (b) is correct.
Chapter: Head and Neck Anatomy; Topic: Muscles of the Neck; Subtopic: Scalenus Anterior Muscle
Keyword Definitions:
• Scalenus Anterior: A deep muscle of the neck that lies between the subclavian artery and vein, arising from cervical vertebrae and inserted on the scalene tubercle of the first rib.
• Phrenic Nerve: A motor nerve to the diaphragm that descends on the anterior surface of the scalenus anterior.
• Subclavian Artery: The main artery of the upper limb passing posterior to scalenus anterior.
• Subclavian Vein: Major vein draining the upper limb, lying anterior to scalenus anterior.
Lead Question – 2014
The key to the root of the neck is the scalenus anterior muscle. Which among the following is TRUE about scalenus anterior?
a) Not Pierced by phrenic nerve
b) Attached to scalene tubercle on 2nd rib
c) Separates subclavian artery from subclavian vein
d) Pierced by phrenic nerve
Explanation:
The scalenus anterior muscle originates from transverse processes of C3–C6 and inserts into the scalene tubercle on the first rib. The phrenic nerve passes on its anterior surface but does not pierce it. It separates the subclavian vein (anterior) from the subclavian artery (posterior). Therefore, option (c) is correct.
1) Which structure passes anterior to the scalenus anterior muscle?
a) Subclavian vein
b) Subclavian artery
c) Brachial plexus
d) Sympathetic trunk
Explanation:
The subclavian vein lies anterior to the scalenus anterior, while the subclavian artery and brachial plexus pass posteriorly. This anatomical relationship is key for central venous access and surgical reference in neck dissections. Thus, (a) is correct.
2) Which structure lies directly posterior to scalenus anterior?
a) Subclavian vein
b) Subclavian artery
c) External jugular vein
d) Internal jugular vein
Explanation:
The subclavian artery lies posterior to the scalenus anterior, forming a vital landmark at the root of the neck. The brachial plexus also emerges from behind this muscle. Hence, (b) is correct.
3) The scalenus anterior muscle arises from:
a) Transverse processes of C1–C4
b) Transverse processes of C3–C6
c) Spinous processes of C2–C5
d) Costal processes of C4–C7
Explanation:
Scalenus anterior originates from anterior tubercles of transverse processes of C3–C6 vertebrae and inserts onto the scalene tubercle on the first rib. This origin helps flex and laterally bend the neck. Hence, (b) is correct.
4) Which nerve lies on the anterior surface of the scalenus anterior?
a) Long thoracic nerve
b) Phrenic nerve
c) Dorsal scapular nerve
d) Suprascapular nerve
Explanation:
The phrenic nerve descends obliquely downward on the anterior surface of the scalenus anterior, deep to the prevertebral fascia. It is a key landmark during central venous catheterization. Hence, (b) is correct.
5) The scalene tubercle is found on which rib?
a) First rib
b) Second rib
c) Third rib
d) Fourth rib
Explanation:
The scalene tubercle is a small ridge on the upper surface of the first rib, serving as the insertion point for scalenus anterior. It separates the grooves for the subclavian vein and artery. Thus, (a) is correct.
6) Clinical Case: During subclavian venous catheterization, which muscle landmark is used to avoid arterial puncture?
a) Sternocleidomastoid
b) Scalenus anterior
c) Omohyoid
d) Platysma
Explanation:
The scalenus anterior separates the subclavian vein (anterior) from the artery (posterior). Knowledge of this landmark helps avoid accidental subclavian artery puncture during catheterization. Hence, (b) is correct.
7) Clinical Case: A patient has diaphragmatic paralysis due to injury of a nerve descending over the scalenus anterior. Which nerve is affected?
a) Vagus nerve
b) Phrenic nerve
c) Recurrent laryngeal nerve
d) Hypoglossal nerve
Explanation:
The phrenic nerve descends obliquely on the scalenus anterior and supplies the diaphragm. Injury causes diaphragmatic paralysis, especially during neck surgery or trauma. Thus, (b) is correct.
8) Clinical Case: A tumor compressing the structures between scalenus anterior and medius causes which condition?
a) Thoracic outlet syndrome
b) Horner’s syndrome
c) Bell’s palsy
d) Torticollis
Explanation:
Compression of the subclavian artery and lower trunk of brachial plexus between scalenus anterior and medius results in thoracic outlet syndrome, presenting with pain, paresthesia, and vascular compromise in the upper limb. Hence, (a) is correct.
9) Clinical Case: In a neck dissection, which structure is used as a landmark to identify the phrenic nerve?
a) Scalenus medius
b) Scalenus anterior
c) Omohyoid
d) Trapezius
Explanation:
The phrenic nerve runs vertically down the anterior surface of the scalenus anterior. Surgeons use this muscle as a landmark to protect the phrenic nerve during neck dissection. Thus, (b) is correct.
10) Clinical Case: A man with penetrating trauma near the root of the neck presents with bleeding from behind the scalenus anterior. The likely vessel injured is:
a) Subclavian vein
b) Subclavian artery
c) Internal jugular vein
d) External jugular vein
Explanation:
The subclavian artery lies posterior to the scalenus anterior. Injury in this location leads to pulsatile bleeding deep to the muscle. Hence, (b) is correct.
Topic: Muscles of Anterior Abdominal Wall
Subtopic: Nerve Supply of Pyramidalis Muscle
Keyword Definitions:
Pyramidalis: A small triangular muscle anterior to the rectus abdominis that tenses the linea alba.
Rectus Sheath: Fibrous covering formed by aponeuroses of abdominal muscles enclosing rectus abdominis and pyramidalis.
Subcostal Nerve: Twelfth thoracic spinal nerve supplying muscles of abdominal wall and overlying skin.
Iliohypogastric Nerve: Arises from L1, supplying skin above pubis and lower abdominal muscles.
Ilioinguinal Nerve: Branch of L1 nerve that supplies the groin region and upper thigh skin.
Genitofemoral Nerve: From L1–L2, divides into genital and femoral branches for cremaster and thigh skin.
Lead Question – 2014
Pyramidalis is supplied by?
a) Subcostal nerve
b) Ilioinguinal nerve
c) Iliohypogastric nerve
d) Genitofemoral nerve
Explanation: The pyramidalis muscle is a small triangular structure in front of the rectus abdominis that tenses the linea alba. It is supplied by the subcostal nerve (T12). It may be absent in some individuals and acts as a surgical landmark for rectus sheath termination. (Answer: a)
1) The rectus sheath encloses all except:
a) Pyramidalis
b) Rectus abdominis
c) Inferior epigastric vessels
d) Transversus abdominis
Explanation: The rectus sheath encloses the rectus abdominis, pyramidalis, and epigastric vessels, but not transversus abdominis, which contributes to its aponeurotic formation. (Answer: d)
2) Linea alba extends between:
a) Xiphoid process and umbilicus
b) Umbilicus and pubic crest
c) Xiphoid process and pubic symphysis
d) Costal margin and iliac crest
Explanation: The linea alba is a fibrous midline raphe extending from the xiphoid process to the pubic symphysis. It is used in surgical incisions because of its avascular nature. (Answer: c)
3) The pyramidalis muscle lies:
a) Behind rectus abdominis
b) In front of rectus abdominis
c) Between rectus and transversus abdominis
d) Deep to linea alba
Explanation: The pyramidalis muscle lies anterior to rectus abdominis within the rectus sheath. It helps tense the linea alba and may be absent in some people. (Answer: b)
4) The subcostal nerve is the:
a) T11 nerve
b) T12 nerve
c) L1 nerve
d) L2 nerve
Explanation: The subcostal nerve is derived from the ventral ramus of the twelfth thoracic nerve (T12). It supplies muscles and skin of the anterior abdominal wall. (Answer: b)
5) Which of the following muscles flexes the trunk and stabilizes the pelvis?
a) Pyramidalis
b) Rectus abdominis
c) Transversus abdominis
d) External oblique
Explanation: The rectus abdominis flexes the vertebral column, stabilizes the pelvis, and compresses abdominal viscera. (Answer: b)
6) (Clinical) A 38-year-old man undergoes abdominal surgery. A small triangular muscle anterior to rectus abdominis is identified. Its nerve supply is from:
a) Ilioinguinal nerve
b) Subcostal nerve
c) Genitofemoral nerve
d) Femoral nerve
Explanation: The pyramidalis muscle is supplied by the subcostal nerve (T12) and serves as a landmark for lower rectus sheath termination. (Answer: b)
7) (Clinical) A midline incision through the linea alba is preferred because it is:
a) Highly vascular
b) Avascular
c) Thick and muscular
d) Contains major nerves
Explanation: The linea alba is an avascular fibrous raphe, which allows midline surgical incisions with minimal bleeding. (Answer: b)
8) (Clinical) A surgeon finds absence of pyramidalis during laparotomy. This indicates:
a) Nerve injury
b) Congenital absence (normal variant)
c) Muscle atrophy
d) Hernia risk
Explanation: The pyramidalis is sometimes congenitally absent. It is a normal anatomical variant and not clinically significant. (Answer: b)
9) (Clinical) During cesarean section, incision is taken in the linea alba because:
a) It has abundant blood vessels
b) It has fewer nerves and vessels
c) It provides muscle access
d) It prevents scar formation
Explanation: The linea alba is chosen for surgical incisions because it is avascular and less painful, providing direct access to the abdominal cavity. (Answer: b)
10) (Clinical) The pyramidalis muscle helps the surgeon identify:
a) Inguinal canal
b) Lower rectus sheath
c) Umbilicus
d) Pubic tubercle
Explanation: The pyramidalis muscle marks the lower end of the rectus sheath near the pubic symphysis and is used as a landmark during abdominal surgeries. (Answer: b)
Topic: Fascia and Muscular Insertions
Subtopic: Iliotibial Tract
Keyword Definitions:
Iliotibial tract: Thickened lateral band of fascia lata extending from iliac crest to lateral condyle of tibia.
Gluteus maximus: Largest gluteal muscle inserting partly into iliotibial tract, aiding hip extension and lateral stability.
Fascia lata: Deep fascia of thigh enclosing muscles and forming iliotibial tract laterally.
Lateral tibial condyle: Outer condyle of tibia where iliotibial tract inserts, providing lateral knee stabilization.
Lead Question - 2014
True about iliotibial tract all except?
a) Receives insertion of gluteus maximus
b) Derived from fascia lata
c) Inserted on lateral tibial condyle
d) None
Explanation: The iliotibial tract is a thickened lateral band of fascia lata. It receives insertion from both gluteus maximus and tensor fasciae latae and inserts on the lateral tibial condyle (Gerdy’s tubercle). All statements are true, so the correct answer is d) None. Clinical importance lies in stabilizing the lateral knee.
Guessed Questions for NEET PG
1) Iliotibial tract inserts at:
a) Medial tibial condyle
b) Lateral tibial condyle
c) Femoral condyle
d) Patella
Explanation: The iliotibial tract inserts on the lateral tibial condyle at Gerdy’s tubercle, stabilizing the knee joint laterally. Correct answer: b) Lateral tibial condyle.
2) Muscle contributing to iliotibial tract is:
a) Gluteus medius
b) Sartorius
c) Tensor fasciae latae
d) Pectineus
Explanation: Tensor fasciae latae contributes fibers to the iliotibial tract, helping in hip abduction and medial rotation. Correct answer: c) Tensor fasciae latae.
3) Iliotibial band syndrome commonly affects:
a) Swimmers
b) Cyclists
c) Runners
d) Weightlifters
Explanation: Iliotibial band syndrome is an overuse injury commonly affecting runners due to repetitive friction over the lateral femoral condyle. Correct answer: c) Runners.
4) Iliotibial tract is a thickening of:
a) Crural fascia
b) Fascia lata
c) Scarpa’s fascia
d) Camper’s fascia
Explanation: The iliotibial tract is a specialized thickened band of fascia lata along the lateral thigh. Correct answer: b) Fascia lata.
5) Iliotibial tract stabilizes which joint?
a) Hip
b) Knee
c) Ankle
d) Shoulder
Explanation: The iliotibial tract stabilizes both hip and knee joints, particularly the lateral knee during walking and running. Correct answer: b) Knee.
6) A marathon runner complains of lateral knee pain, most likely diagnosis?
a) Meniscal tear
b) Iliotibial band syndrome
c) ACL injury
d) PCL injury
Explanation: Lateral knee pain in long-distance runners is often due to iliotibial band friction syndrome. Correct answer: b) Iliotibial band syndrome.
7) Iliotibial tract extends from:
a) Iliac crest to medial tibial condyle
b) Iliac crest to lateral tibial condyle
c) Ischium to patella
d) Femoral shaft to tibia
Explanation: Iliotibial tract extends from iliac crest to lateral tibial condyle (Gerdy’s tubercle). Correct answer: b) Iliac crest to lateral tibial condyle.
8) Which test assesses iliotibial band tightness?
a) Lachman test
b) Ober’s test
c) McMurray test
d) Thompson test
Explanation: Ober’s test is used clinically to check iliotibial band tightness. Correct answer: b) Ober’s test.
9) The iliotibial tract is thickest at:
a) Thigh
b) Leg
c) Foot
d) Knee joint line
Explanation: The iliotibial tract is thickest along the lateral thigh where it descends to insert on the tibia. Correct answer: a) Thigh.
10) Iliotibial tract aids gluteus maximus in:
a) Hip flexion
b) Hip extension
c) Knee flexion
d) Ankle plantarflexion
Explanation: The iliotibial tract transmits the action of gluteus maximus for hip extension and lateral stabilization. Correct answer: b) Hip extension.
Topic: Lower Limb Muscles
Subtopic: Dorsiflexors of Foot
Keyword Definitions:
Dorsiflexion: Movement of the foot upward at the ankle joint, bringing the toes closer to the shin.
Tibialis anterior: Primary dorsiflexor of the foot, also inverts the foot.
Tibialis posterior: Muscle that plantarflexes and inverts the foot.
Peroneus brevis: Muscle that everts and weakly plantarflexes the foot.
Extensor digitorum brevis: Muscle on the dorsum of the foot, extends toes but does not dorsiflex significantly.
Lead Question - 2014
Which of the following dorsiflexes the foot?
a) Tibialis posterior
b) Tibialis anterior
c) Peroneus brevis
d) Extensor digitorum brevis
Explanation: Tibialis anterior is the chief dorsiflexor of the foot. It originates from the lateral tibia and inserts on the medial cuneiform and first metatarsal, allowing dorsiflexion and inversion. Other muscles listed do not act as strong dorsiflexors. Correct answer: b) Tibialis anterior.
Guessed Questions for NEET PG
1) Tibialis anterior inserts on?
a) Calcaneum
b) Navicular
c) Medial cuneiform and base of 1st metatarsal
d) Cuboid
Explanation: Tibialis anterior inserts on the medial cuneiform and base of the first metatarsal. This allows it to dorsiflex and invert the foot effectively. Correct answer: c) Medial cuneiform and base of 1st metatarsal.
2) Clinical: A patient with foot drop most likely has paralysis of?
a) Tibialis posterior
b) Tibialis anterior
c) Peroneus longus
d) Soleus
Explanation: Foot drop occurs due to paralysis of dorsiflexors, particularly tibialis anterior, commonly from damage to the common peroneal nerve. Correct answer: b) Tibialis anterior.
3) Which nerve innervates tibialis anterior?
a) Tibial nerve
b) Deep peroneal nerve
c) Superficial peroneal nerve
d) Femoral nerve
Explanation: Tibialis anterior is supplied by the deep peroneal nerve, a branch of the common peroneal nerve. This ensures dorsiflexion of the foot. Correct answer: b) Deep peroneal nerve.
4) Clinical: Weak dorsiflexion with sensory loss between first and second toe suggests injury to?
a) Tibial nerve
b) Deep peroneal nerve
c) Superficial peroneal nerve
d) Sural nerve
Explanation: The deep peroneal nerve supplies tibialis anterior and provides cutaneous sensation to the web between the first and second toes. Its injury causes weakness of dorsiflexion and sensory loss. Correct answer: b) Deep peroneal nerve.
5) Which movement is opposed by tibialis anterior?
a) Plantarflexion
b) Inversion
c) Eversion
d) Dorsiflexion
Explanation: Tibialis anterior causes dorsiflexion and inversion of the foot. It works against plantarflexion and eversion. Correct answer: a) Plantarflexion.
6) Clinical: Steppage gait is seen in lesions of?
a) Superficial peroneal nerve
b) Tibial nerve
c) Common peroneal nerve
d) Femoral nerve
Explanation: Steppage gait occurs in common peroneal nerve injury, due to paralysis of dorsiflexors like tibialis anterior. The patient lifts the leg high to prevent toe dragging. Correct answer: c) Common peroneal nerve.
7) Which of the following is NOT a dorsiflexor?
a) Extensor digitorum longus
b) Extensor hallucis longus
c) Tibialis posterior
d) Tibialis anterior
Explanation: Tibialis posterior is a plantarflexor and invertor, not a dorsiflexor. Other listed muscles dorsiflex the foot. Correct answer: c) Tibialis posterior.
8) Clinical: Which muscle is tested when patient is asked to dorsiflex and invert the foot against resistance?
a) Tibialis anterior
b) Tibialis posterior
c) Peroneus tertius
d) Soleus
Explanation: Dorsiflexion with inversion specifically tests tibialis anterior function, as it is the main muscle performing these combined actions. Correct answer: a) Tibialis anterior.
9) Clinical: Trauma to the lateral aspect of the knee may cause loss of dorsiflexion due to injury to?
a) Femoral nerve
b) Common peroneal nerve
c) Tibial nerve
d) Obturator nerve
Explanation: The common peroneal nerve winds around the neck of fibula, making it vulnerable to trauma. Its injury leads to foot drop from dorsiflexor paralysis. Correct answer: b) Common peroneal nerve.
10) Main action of tibialis anterior?
a) Plantarflexion and eversion
b) Dorsiflexion and inversion
c) Plantarflexion and inversion
d) Dorsiflexion and eversion
Explanation: Tibialis anterior produces dorsiflexion and inversion. It pulls the foot upwards and medially, crucial for normal gait. Correct answer: b) Dorsiflexion and inversion.
Topic: Shoulder Joint
Subtopic: Intracapsular but Extrasynovial Structures
Keyword Definitions:
Intracapsular: Located within the fibrous capsule of a joint.
Extrasynovial: Outside the synovial membrane but still within the joint capsule.
Shoulder joint: A synovial ball-and-socket joint formed between the humerus and scapula.
Biceps tendon: The long head passes through the shoulder joint capsule but remains extrasynovial.
Triceps tendon: Attaches posteriorly, not intracapsular.
Lead Question - 2014
Intracapsular but extrasynovial is ?
a) Long head of triceps
b) Long head of biceps
c) Short head of biceps
d) Medial head of biceps
Explanation: The long head of biceps tendon lies within the capsule of the shoulder joint but remains outside the synovial cavity, making it intracapsular but extrasynovial. This feature protects the tendon while maintaining joint mobility. Correct answer is Long head of biceps.
Guessed Questions:
1) Which tendon passes through the intertubercular sulcus of the humerus?
a) Short head of biceps
b) Long head of biceps
c) Coracobrachialis
d) Teres major
Explanation: The long head of biceps tendon passes through the intertubercular sulcus and is held in place by the transverse humeral ligament. This orientation stabilizes the tendon during shoulder movement. Correct answer is Long head of biceps.
2) Which structure prevents dislocation of the long head of biceps tendon?
a) Glenoid labrum
b) Transverse humeral ligament
c) Rotator cuff
d) Deltoid muscle
Explanation: The transverse humeral ligament bridges across the intertubercular sulcus, preventing displacement of the long head of biceps tendon during arm movements. Correct answer is Transverse humeral ligament.
3) A 30-year-old man presents with shoulder pain. MRI reveals inflammation of the tendon passing through the intertubercular sulcus. Which tendon is affected?
a) Subscapularis
b) Long head of biceps
c) Supraspinatus
d) Coracobrachialis
Explanation: Bicipital tendinitis occurs due to inflammation of the long head of biceps tendon within the intertubercular sulcus. It often presents as anterior shoulder pain aggravated by flexion. Correct answer is Long head of biceps.
4) Which part of the glenoid labrum is attached to the long head of biceps tendon?
a) Inferior
b) Superior
c) Anterior
d) Posterior
Explanation: The long head of biceps tendon originates from the supraglenoid tubercle and the superior part of the glenoid labrum, helping stabilize the shoulder joint. Correct answer is Superior.
5) Intracapsular but extrasynovial tendon in the knee joint is?
a) Posterior cruciate ligament
b) Anterior cruciate ligament
c) Patellar ligament
d) Medial collateral ligament
Explanation: The cruciate ligaments (ACL and PCL) are located within the joint capsule but remain extrasynovial. This anatomical arrangement protects them from direct synovial exposure. Correct answer is Anterior cruciate ligament.
6) A patient develops rupture of intracapsular but extrasynovial tendon in the shoulder joint. Which movement will be most affected?
a) Shoulder abduction
b) Shoulder extension
c) Shoulder flexion
d) Shoulder adduction
Explanation: The long head of biceps assists in flexion of the shoulder joint. Rupture of this tendon leads to weakness of flexion and supination, particularly affecting overhead movements. Correct answer is Shoulder flexion.
7) The tendon of long head of biceps is enclosed in a synovial sheath. True or False?
a) True
b) False
Explanation: The long head of biceps tendon has a tubular synovial sheath, facilitating smooth gliding within the intertubercular sulcus during arm movements. Correct answer is True.
8) Which tendon rupture causes “Popeye deformity” in the arm?
a) Short head of biceps
b) Long head of biceps
c) Brachialis
d) Coracobrachialis
Explanation: Rupture of the long head of biceps tendon leads to distal bunching of the muscle belly, producing the “Popeye deformity.” It usually occurs in older adults or athletes. Correct answer is Long head of biceps.
9) Which tendon runs intracapsular in shoulder joint but remains extrasynovial?
a) Long head of triceps
b) Long head of biceps
c) Supraspinatus
d) Teres minor
Explanation: The long head of biceps tendon is the only tendon that lies intracapsular but remains extrasynovial in the shoulder joint. Correct answer is Long head of biceps.
10) Which structure keeps the tendon of long head of biceps in place during shoulder motion?
a) Coracohumeral ligament
b) Transverse humeral ligament
c) Glenohumeral ligament
d) Rotator cuff tendons
Explanation: The transverse humeral ligament stabilizes the long head of biceps tendon in the intertubercular sulcus, preventing its displacement during shoulder movements. Correct answer is Transverse humeral ligament.
Topic: Upper Limb Nerves
Subtopic: Median Nerve and Branches
Keyword Definitions:
Anterior interosseous nerve: A branch of the median nerve, supplying deep flexors of the forearm and pronator quadratus.
Median nerve: Formed by medial and lateral cords, supplies most forearm flexors and hand muscles.
Radial nerve: A major nerve of the posterior arm, supplying extensors of the forearm and hand.
Ulnar nerve: Supplies intrinsic hand muscles and some forearm flexors.
Axillary nerve: Supplies deltoid and teres minor muscles, and shoulder sensation.
Lead Question - 2014
Anterior interosseous nerve is a branch of?
a) Radial nerve
b) Median nerve
c) Ulnar nerve
d) Axillary nerve
Explanation: The anterior interosseous nerve arises from the median nerve just below the elbow. It supplies flexor pollicis longus, lateral half of flexor digitorum profundus, and pronator quadratus. It does not provide cutaneous innervation. Therefore, the correct answer is Median nerve, which gives this important motor branch.
Guessed Questions
1. The anterior interosseous nerve supplies all except?
a) Flexor pollicis longus
b) Pronator quadratus
c) Flexor digitorum superficialis
d) Flexor digitorum profundus (lateral half)
Explanation: The anterior interosseous nerve supplies FPL, pronator quadratus, and the lateral half of FDP. Flexor digitorum superficialis is supplied by the median nerve but not its anterior interosseous branch. Thus, the correct answer is Flexor digitorum superficialis.
2. A patient with anterior interosseous nerve injury is unable to?
a) Flex distal phalanx of thumb
b) Flex proximal phalanx of thumb
c) Extend wrist
d) Abduct thumb
Explanation: Anterior interosseous nerve injury causes weakness in flexor pollicis longus, leading to inability to flex the distal phalanx of the thumb. Proximal flexion is intact, wrist extension involves radial nerve, and thumb abduction involves radial/median nerves. Thus, the correct answer is Flex distal phalanx of thumb.
3. A clinical sign of anterior interosseous nerve palsy is?
a) Ape thumb deformity
b) Hand of benediction
c) Pinch sign
d) Claw hand
Explanation: In anterior interosseous nerve palsy, patients cannot form a tip-to-tip pinch between thumb and index finger due to loss of FPL and FDP function. Instead, they approximate pads of fingers. This is called the Pinch sign, characteristic of AIN injury.
4. The median nerve in the forearm gives rise to?
a) Anterior interosseous nerve
b) Posterior interosseous nerve
c) Musculocutaneous nerve
d) Lateral pectoral nerve
Explanation: The median nerve gives off the anterior interosseous nerve below the elbow. The posterior interosseous is a branch of the radial nerve, musculocutaneous comes from the lateral cord, and lateral pectoral is from the lateral cord. Correct answer is Anterior interosseous nerve.
5. A patient with deep forearm pain and weakness of pinch grip but no sensory loss most likely has -
a) Median nerve lesion
b) Ulnar nerve lesion
c) Anterior interosseous nerve lesion
d) Radial nerve lesion
Explanation: Anterior interosseous nerve is purely motor. Its lesion causes deep forearm pain, loss of pinch grip, but no cutaneous sensory loss. Median and ulnar nerve lesions include sensory changes, radial nerve causes wrist drop. Correct answer is Anterior interosseous nerve lesion.
6. Flexor digitorum profundus is supplied by -
a) Median nerve alone
b) Ulnar nerve alone
c) Median and ulnar nerves
d) Radial nerve
Explanation: Flexor digitorum profundus has dual innervation. Lateral half (index and middle fingers) by anterior interosseous nerve (median), medial half (ring and little fingers) by ulnar nerve. Hence, correct answer is Median and ulnar nerves.
7. The anterior interosseous nerve runs along which artery?
a) Radial artery
b) Anterior interosseous artery
c) Posterior interosseous artery
d) Ulnar artery
Explanation: The anterior interosseous nerve runs on the anterior surface of the interosseous membrane, accompanying the anterior interosseous artery, a branch of the ulnar artery. Correct answer is Anterior interosseous artery.
8. Injury to anterior interosseous nerve affects which movement?
a) Thumb extension
b) Index finger DIP flexion
c) Wrist flexion
d) Elbow extension
Explanation: The anterior interosseous nerve supplies the lateral half of FDP, flexing DIP of index and middle fingers. Injury impairs DIP flexion of index finger. Wrist flexion is preserved by FCR, thumb extension by radial nerve, elbow extension by radial nerve. Correct answer is Index finger DIP flexion.
9. The "OK sign" test is used to diagnose -
a) Ulnar nerve palsy
b) Radial nerve palsy
c) Anterior interosseous nerve palsy
d) Axillary nerve palsy
Explanation: In anterior interosseous nerve palsy, the patient cannot form a circle using thumb and index finger tips, producing a flat "OK sign." This is diagnostic of Anterior interosseous nerve palsy. Other nerve lesions present with different clinical signs.
10. A 40-year-old with forearm fracture develops inability to flex thumb IP and index DIP joints, but no sensory loss. Which nerve is injured?
a) Ulnar nerve
b) Anterior interosseous nerve
c) Radial nerve
d) Musculocutaneous nerve
Explanation: Loss of thumb IP and index DIP flexion with no sensory loss indicates anterior interosseous nerve injury, as it supplies FPL and FDP lateral half. Ulnar nerve causes sensory loss, radial nerve affects extensors, musculocutaneous supplies arm flexors. Correct answer is Anterior interosseous nerve.
Topic: Muscle Physiology
Subtopic: Muscle Contraction and ATPase Activity
Keyword Definitions:
• ATPase: Enzyme that hydrolyzes ATP to release energy for cellular processes.
• Actin: Thin filament protein in muscles interacting with myosin for contraction.
• Myosin: Thick filament protein with intrinsic ATPase activity driving cross-bridge cycling.
• Troponin: Regulatory protein controlling actin-myosin interaction during contraction.
• Cross-bridge cycle: Repetitive interaction between actin and myosin powered by ATP hydrolysis.
Lead Question - 2013
ATPase activity is present in
a) Actin
b) Myosin
c) Troponin
d) None
Explanation: Myosin contains intrinsic ATPase activity in its head domain, which hydrolyzes ATP to provide energy for cross-bridge cycling and muscle contraction. Actin and troponin are structural and regulatory proteins without enzymatic activity. Answer: b) Myosin.
1) Guess Question:
Which part of myosin possesses ATPase activity?
a) Tail region
b) Head region
c) Rod region
d) Light chains
Explanation: The myosin head contains ATPase activity, which hydrolyzes ATP to ADP and inorganic phosphate, providing energy for the power stroke during contraction. Tail and rod regions are structural. Answer: b) Head region.
2) Guess Question:
Which ion binds to troponin to initiate contraction?
a) Sodium
b) Potassium
c) Calcium
d) Magnesium
Explanation: Calcium binds to troponin C, causing conformational change that moves tropomyosin off actin binding sites, allowing myosin heads with ATPase activity to interact. Answer: c) Calcium.
3) Guess Question:
Which process directly uses myosin ATPase activity?
a) Glycolysis
b) Cross-bridge cycling
c) Oxidative phosphorylation
d) Sodium-potassium pump
Explanation: Myosin ATPase provides energy for cross-bridge cycling in muscle contraction, allowing repetitive attachment, pivoting, and detachment from actin. Glycolysis and oxidative phosphorylation generate ATP, while Na+/K+ pump is unrelated. Answer: b) Cross-bridge cycling.
4) Guess Question:
A patient has myopathy with defective myosin ATPase. Expected finding is:
a) Weak muscle contraction
b) Hyperactive reflexes
c) Normal contraction
d) Increased tone
Explanation: Defective myosin ATPase prevents ATP hydrolysis, reducing energy for cross-bridge cycling. This leads to weak muscle contraction, hypotonia, and exercise intolerance. Answer: a) Weak muscle contraction.
5) Guess Question:
Which filament slides during muscle contraction?
a) Myosin
b) Actin
c) Troponin
d) Titin
Explanation: Actin filaments slide past stationary myosin filaments during contraction. Myosin heads use ATPase activity to pull actin. Troponin and titin are regulatory and elastic elements. Answer: b) Actin.
6) Guess Question:
Which molecule inhibits actin-myosin interaction in relaxed muscle?
a) Tropomyosin
b) Troponin
c) ATP
d) Myosin light chain
Explanation: Tropomyosin blocks actin binding sites in relaxed muscle. Troponin regulates tropomyosin movement, ATP energizes myosin head, but the inhibitor of cross-bridge formation is tropomyosin. Answer: a) Tropomyosin.
7) Guess Question:
Which compound directly provides energy for myosin ATPase?
a) ADP
b) AMP
c) ATP
d) GTP
Explanation: ATP binds myosin head and is hydrolyzed by ATPase activity to ADP + Pi, providing energy for the power stroke and detachment. ADP alone does not provide energy. Answer: c) ATP.
8) Guess Question:
A 10-year-old has congenital myopathy with low myosin ATPase activity. Likely symptom is:
a) Delayed muscle contraction
b) Rapid fatigue
c) Weakness
d) All of the above
Explanation: Low myosin ATPase impairs ATP hydrolysis, slowing cross-bridge cycling, reducing force generation, and causing weakness, delayed contraction, and rapid fatigue. Answer: d) All of the above.
9) Guess Question:
Which filament does not have ATPase activity?
a) Myosin
b) Actin
c) Both
d) None
Explanation: Only myosin has ATPase activity. Actin is a structural filament without enzymatic function. Therefore, actin cannot hydrolyze ATP. Answer: b) Actin.
10) Guess Question:
During rigor mortis, ATP is depleted. Effect on myosin ATPase activity is:
a) Increased activity
b) Decreased activity
c) No cross-bridge detachment
d) Enhanced contraction
Explanation: Without ATP, myosin heads cannot detach from actin despite ATPase presence. Cross-bridges remain attached, causing rigidity (rigor mortis). ATP hydrolysis is required for detachment and relaxation. Answer: c) No cross-bridge detachment.
Chapter: Excitable Tissues
Topic: Muscle Physiology
Subtopic: Calcium Channels in Skeletal Muscle
Keyword Definitions:
• Calcium channels: Protein channels that allow Ca²⁺ ions to enter cells, vital for muscle contraction and neurotransmission.
• L-type channels: Long-lasting, high-voltage activated channels predominant in skeletal and cardiac muscles.
• T-type channels: Transient, low-voltage activated channels, important in pacemaker activity.
• N-type channels: Found mainly in neurons, involved in neurotransmitter release.
• R-type channels: Resistant calcium channels, less common physiologically.
Lead Question - 2013
Most common type of calcium channels of skeletal muscles are ?
a) T type
b) L type
c) R type
d) N type
Explanation: Skeletal muscle contraction is mediated by L-type calcium channels located in the transverse tubules. These channels act as voltage sensors and couple with ryanodine receptors on the sarcoplasmic reticulum for Ca²⁺ release. Correct answer: L type.
1) Calcium release from sarcoplasmic reticulum in skeletal muscle is triggered by?
a) L-type calcium channel activation
b) T-type calcium channel activation
c) Sodium channel inactivation
d) Potassium efflux
Explanation: Skeletal muscle contraction depends on L-type calcium channel activation in T-tubules, which mechanically couples with ryanodine receptors, leading to sarcoplasmic reticulum calcium release. Correct answer: L-type calcium channel activation.
2) A patient with mutation in ryanodine receptor shows?
a) Malignant hyperthermia
b) Hypokalemia
c) Myasthenia gravis
d) Hyponatremia
Explanation: Mutations in ryanodine receptors cause uncontrolled calcium release during anesthesia, leading to malignant hyperthermia with rigidity, tachycardia, and hyperthermia. Correct answer: Malignant hyperthermia.
3) In cardiac muscle, the main calcium channel responsible for excitation-contraction coupling is?
a) L-type
b) T-type
c) N-type
d) P-type
Explanation: Cardiac contraction relies on L-type calcium channels, which mediate calcium influx during plateau phase, triggering further calcium-induced calcium release from sarcoplasmic reticulum. Correct answer: L-type.
4) A patient treated with nifedipine. Which calcium channel is blocked?
a) L-type
b) T-type
c) N-type
d) R-type
Explanation: Nifedipine, a dihydropyridine calcium channel blocker, selectively inhibits L-type calcium channels, reducing vascular smooth muscle contraction and lowering blood pressure. Correct answer: L-type.
5) Which calcium channels are important in pacemaker activity of SA node?
a) L-type
b) T-type
c) N-type
d) R-type
Explanation: T-type calcium channels open transiently at low voltage, contributing to the depolarization phase of SA node pacemaker cells. Correct answer: T-type.
6) A child with congenital myopathy shows impaired excitation-contraction coupling. Likely channel defect?
a) L-type calcium channel
b) Sodium channel
c) Potassium channel
d) Chloride channel
Explanation: Excitation-contraction coupling in skeletal muscle depends on L-type calcium channels functioning as voltage sensors. A defect impairs calcium release from SR, leading to muscle weakness. Correct answer: L-type calcium channel.
7) N-type calcium channels are primarily located in?
a) Skeletal muscle
b) Cardiac muscle
c) Neurons
d) Liver cells
Explanation: N-type calcium channels are located in presynaptic terminals of neurons, playing a role in neurotransmitter release. Correct answer: Neurons.
8) In smooth muscle, calcium entry for contraction mainly occurs through?
a) L-type channels
b) N-type channels
c) R-type channels
d) Sodium leak channels
Explanation: Smooth muscle contraction largely depends on calcium influx through L-type calcium channels, which activate calmodulin and MLCK for actin-myosin interaction. Correct answer: L-type.
9) A patient develops synaptic transmission defect with reduced neurotransmitter release. Likely channel defect?
a) N-type calcium channel
b) L-type calcium channel
c) T-type calcium channel
d) R-type calcium channel
Explanation: N-type calcium channels control neurotransmitter release at presynaptic terminals. Dysfunction reduces exocytosis, impairing synaptic communication. Correct answer: N-type calcium channel.
10) Which calcium channel is responsible for prolonged calcium current in cardiac ventricular myocytes?
a) L-type
b) T-type
c) N-type
d) R-type
Explanation: L-type calcium channels mediate long-lasting inward calcium current during plateau phase of cardiac action potential, crucial for contraction. Correct answer: L-type.
11) Skeletal muscle contraction does not depend on extracellular calcium influx because?
a) Direct mechanical coupling of L-type channel with SR ryanodine receptor
b) Sodium channel influx is sufficient
c) Potassium efflux replaces calcium
d) Myosin does not need calcium
Explanation: In skeletal muscle, excitation-contraction coupling occurs by direct mechanical interaction between L-type channels in T-tubules and ryanodine receptors in SR. Thus, extracellular calcium influx is not essential. Correct answer: Direct mechanical coupling.
Topic: Muscle Fiber Types
Subtopic: Characteristics of Red Muscle Fibers
Red muscle fibers: Muscle fibers rich in myoglobin and mitochondria, adapted for endurance and continuous activity.
Mitochondria: Organelles responsible for ATP production through oxidative phosphorylation, abundant in endurance muscle fibers.
Myoglobin: Oxygen-binding protein in muscle cells, increasing oxygen availability for aerobic metabolism, giving red fibers their color.
Oxidative capacity: The ability of a muscle fiber to generate ATP through aerobic respiration, high in red fibers.
Glycolytic metabolism: Energy production pathway primarily using anaerobic glycolysis, typical of white muscle fibers.
Lead Question - 2013 (September 2008)
All are true about red muscle fibers except?
a) More mitochondria
b) Glycolytic metabolism
c) More myoglobin
d) More oxidative capacity
Answer: b) Glycolytic metabolism
Explanation: Red muscle fibers are rich in mitochondria and myoglobin, providing high oxidative capacity for sustained aerobic activities. Their primary metabolism is oxidative phosphorylation, not glycolytic metabolism, which is characteristic of white (fast-twitch) fibers specialized for short bursts of intense activity rather than endurance.
1. Guessed Question
Red muscle fibers are adapted for?
a) Short bursts of strength
b) Endurance and continuous aerobic work
c) Anaerobic metabolism
d) Rapid fatigue
Answer: b) Endurance and continuous aerobic work
Explanation: Red muscle fibers (Type I) are designed for sustained, low-intensity activities. Their rich myoglobin and mitochondria content facilitate efficient aerobic metabolism, allowing prolonged contraction without fatigue, as seen in postural muscles or endurance sports like marathon running.
2. Guessed Question
Which component gives red muscle fibers their color?
a) Mitochondria
b) Hemoglobin
c) Myoglobin
d) Glycogen
Answer: c) Myoglobin
Explanation: Myoglobin, an oxygen-binding protein present in high amounts in red muscle fibers, imparts a reddish color. It ensures a steady oxygen supply for aerobic metabolism, critical in endurance activities where sustained muscle contraction depends on continuous oxygen utilization.
3. Guessed Question
High oxidative capacity of red muscle fibers means they:
a) Generate ATP anaerobically
b) Depend on glycolysis
c) Use oxygen for energy
d) Store less ATP
Answer: c) Use oxygen for energy
Explanation: Red muscle fibers possess high oxidative capacity due to abundant mitochondria and myoglobin. This enables efficient ATP production via aerobic respiration, ideal for sustained activities, contrasting with white fibers relying more on anaerobic glycolysis for quick, powerful actions.
4. Guessed Question
Which muscle fiber type fatigues least?
a) White fibers
b) Red fibers
c) Type IIb fibers
d) Fast-twitch fibers
Answer: b) Red fibers
Explanation: Red fibers (Type I) are fatigue-resistant because of their high mitochondria and myoglobin content. These features support continuous ATP production through aerobic pathways, ideal for long-duration, low-intensity tasks, such as maintaining posture or endurance running.
5. Guessed Question
Which fiber type primarily performs anaerobic work?
a) Red fibers
b) Type I fibers
c) White fibers
d) Cardiac fibers
Answer: c) White fibers
Explanation: White fibers (Type IIb) have fewer mitochondria and rely on anaerobic glycolysis for energy production. They provide rapid, powerful contractions for short durations but fatigue quickly, unlike red fibers, which sustain prolonged aerobic work.
6. Guessed Question
High myoglobin content in red fibers facilitates?
a) Anaerobic metabolism
b) Oxygen storage and delivery
c) Rapid contraction
d) Low ATP production
Answer: b) Oxygen storage and delivery
Explanation: Myoglobin stores and delivers oxygen within muscle cells, ensuring continuous aerobic metabolism. Red fibers benefit from this high myoglobin concentration, which is crucial for endurance activities, allowing efficient ATP production and preventing fatigue during prolonged muscle use.
7. Guessed Question
Red fibers are abundant in which type of muscle?
a) Postural muscles
b) Phasic muscles
c) Fast-contracting muscles
d) Digestive smooth muscles
Answer: a) Postural muscles
Explanation: Postural muscles are rich in red fibers to support sustained low-intensity contractions necessary for maintaining body posture. Their high mitochondrial and myoglobin content enable efficient aerobic metabolism, preventing fatigue and ensuring continuous function throughout the day.
8. Guessed Question
Primary role of red muscle fibers is to:
a) Generate short, intense force
b) Maintain prolonged contractions
c) Store glycogen
d) Control reflex actions
Answer: b) Maintain prolonged contractions
Explanation: Red fibers (Type I) are specialized for prolonged, low-intensity contractions. Their abundant mitochondria and myoglobin support sustained aerobic ATP production, critical for endurance tasks and postural maintenance, differing from white fibers designed for brief, intense efforts.
9. Guessed Question
Which statement is incorrect regarding red muscle fibers?
a) High mitochondrial content
b) High myoglobin content
c) Predominantly glycolytic metabolism
d) Suited for endurance activities
Answer: c) Predominantly glycolytic metabolism
Explanation: Red muscle fibers do not rely on glycolytic metabolism. Instead, they utilize aerobic oxidative phosphorylation supported by mitochondria and myoglobin, enabling sustained energy production for endurance activities, contrasting with white fibers that use glycolysis for rapid energy.
.
Topic: Muscle Fiber Types
Subtopic: Mitochondrial Content in Muscle Fibers
Red fibers: Muscle fibers rich in myoglobin and mitochondria, suited for endurance.
Type I fibers: Slow-twitch fibers, abundant in mitochondria, support sustained contraction and aerobic metabolism.
White fibers: Fast-twitch fibers with fewer mitochondria, designed for rapid, powerful contractions and anaerobic work.
Slow fibers: Fibers that contract slowly and are fatigue-resistant due to abundant mitochondria and myoglobin.
Lead Question - 2013 (September 2008)
Less mitochondria are seen in -
a) Red fibers
b) Type I fibers
c) White fibers
d) Slow fibers
Answer: c) White fibers
Explanation: White fibers (Type IIb) are fast-twitch fibers with fewer mitochondria, optimized for short bursts of high-intensity activities. Their energy is primarily derived from anaerobic glycolysis. These fibers fatigue quickly, making them less suited for endurance activities compared to red or Type I fibers which are mitochondria-rich and support sustained contraction.
1. Guessed Question
Which fibers are rich in mitochondria and support endurance?
a) White fibers
b) Type IIb fibers
c) Red fibers
d) Fast fibers
Answer: c) Red fibers
Explanation: Red fibers (Type I) are rich in mitochondria and myoglobin, supporting aerobic metabolism and sustained contractions. These fibers are designed for endurance activities, providing resistance to fatigue and enabling prolonged muscle work by efficiently producing ATP through oxidative phosphorylation.
2. Guessed Question
Type I muscle fibers are characterized by?
a) Rapid fatigue
b) Low mitochondrial content
c) High myoglobin content
d) Anaerobic metabolism
Answer: c) High myoglobin content
Explanation: Type I muscle fibers contain high myoglobin and mitochondrial density, supporting aerobic metabolism and sustained low-force activities. They are slow-twitch fibers that resist fatigue, primarily used in endurance exercises such as marathon running or posture maintenance due to their efficient oxygen utilization and energy production.
3. Guessed Question
Which fiber type is predominantly used in sprinting?
a) Type I fibers
b) White fibers
c) Red fibers
d) Smooth fibers
Answer: b) White fibers
Explanation: White fibers (Type IIb) are fast-twitch fibers used in activities like sprinting that require rapid, powerful contractions. Due to low mitochondrial content, these fibers rely on anaerobic metabolism, providing energy quickly but with fast fatigue onset, making them ideal for short-term high-intensity efforts.
4. Guessed Question
Primary energy system for white muscle fibers?
a) Oxidative phosphorylation
b) Anaerobic glycolysis
c) Fatty acid oxidation
d) Creatine phosphate system
Answer: b) Anaerobic glycolysis
Explanation: White fibers depend mainly on anaerobic glycolysis due to their low mitochondrial content. This process allows rapid ATP production without the need for oxygen, enabling short bursts of intense activity, but results in lactic acid accumulation, contributing to quick onset of fatigue.
5. Guessed Question
Which fiber type is slow to fatigue?
a) Type IIb fibers
b) White fibers
c) Type I fibers
d) Fast fibers
Answer: c) Type I fibers
Explanation: Type I fibers are slow-twitch and fatigue-resistant due to their rich mitochondrial and myoglobin content. They rely on aerobic metabolism for sustained energy, making them ideal for endurance activities such as long-distance running, standing posture, and activities requiring constant low-intensity effort.
6. Guessed Question
White fibers predominantly utilize which metabolic pathway?
a) Oxidative metabolism
b) Anaerobic glycolysis
c) Lipid oxidation
d) Protein catabolism
Answer: b) Anaerobic glycolysis
Explanation: Due to their limited mitochondria and myoglobin, white fibers (Type IIb) primarily rely on anaerobic glycolysis for ATP production. This enables fast energy supply during high-intensity activities but produces lactic acid, causing rapid fatigue and restricting use in endurance exercises.
7. Guessed Question
Which fiber type is most abundant in postural muscles?
a) Type IIb fibers
b) White fibers
c) Type I fibers
d) Type IIa fibers
Answer: c) Type I fibers
Explanation: Postural muscles contain predominantly Type I fibers due to their high mitochondrial and myoglobin content, ensuring continuous low-force contractions without fatigue. These fibers are specialized for endurance and maintaining body posture over long periods, sustaining steady force generation through aerobic metabolism.
8. Guessed Question
Which fiber type is referred to as "slow oxidative"?
a) Type IIb
b) Type I
c) Type IIa
d) White fibers
Answer: b) Type I
Explanation: Type I fibers are termed "slow oxidative" because of their slow contraction speed, rich mitochondrial density, and aerobic metabolism capability. This enables them to sustain prolonged, low-intensity activities and resist fatigue, making them crucial in endurance performance and posture maintenance.
9. Guessed Question
White fibers are primarily used for:
a) Long-distance running
b) High-intensity short bursts of activity
c) Postural control
d) Low-resistance repetitive work
Answer: b) High-intensity short bursts of activity
Explanation: White fibers (Type IIb) are optimized for short-term, high-intensity work like weightlifting or sprinting. Their low mitochondrial and high glycolytic enzyme content allow rapid ATP generation through anaerobic glycolysis, though they fatigue quickly, unsuitable for sustained activities.
Topic: Muscles of Facial Expression
Subtopic: Orbicularis Oculi and Related Muscles
Keyword Definitions:
Orbicularis Oculi: Circular muscle around the eye that enables eyelid closure and blinking.
Levator Labii Superioris: Muscle elevating upper lip; involved in facial expression.
Corrugator Supercilii: Muscle that draws eyebrows medially and downward, producing frowning.
Levator Palpebrae Superioris: Muscle responsible for raising the upper eyelid; not involved in winking.
Lead Question - 2013
Which muscle plays a role in winking?
a) Levator labii superioris
b) Orbicularis oculi
c) Corrugator supercilli
d) Levator palpebrae
Explanation: The orbicularis oculi muscle is the primary muscle responsible for eyelid closure and actions like blinking and winking. It encircles the eye and contracts to close the eyelids voluntarily or reflexively. Other listed muscles serve different facial functions. The correct answer is b) Orbicularis oculi.
Guessed Question 2
Which branch of the facial nerve supplies the orbicularis oculi?
a) Temporal
b) Zygomatic
c) Buccal
d) Mandibular
Explanation: The temporal and zygomatic branches of the facial nerve (cranial nerve VII) innervate the orbicularis oculi muscle, controlling eyelid movements such as blinking and winking. These branches are crucial in facial nerve function assessment. Correct answer is a) Temporal and b) Zygomatic.
Guessed Question 3
Paralysis of orbicularis oculi leads to which clinical sign?
a) Lagophthalmos
b) Ptosis
c) Diplopia
d) Strabismus
Explanation: Lagophthalmos refers to incomplete eyelid closure, often seen in facial nerve (VII) palsy affecting the orbicularis oculi. This can lead to exposure keratitis and eye dryness due to impaired blinking. The correct answer is a) Lagophthalmos.
Guessed Question 4
The levator palpebrae superioris is innervated by which nerve?
a) Oculomotor nerve (CN III)
b) Facial nerve (CN VII)
c) Trigeminal nerve (CN V)
d) Accessory nerve (CN XI)
Explanation: The levator palpebrae superioris muscle is innervated by the oculomotor nerve (CN III) and elevates the upper eyelid. Dysfunction may lead to ptosis, but it does not contribute to winking. Correct answer is a) Oculomotor nerve (CN III).
Guessed Question 5
Which of the following muscles is responsible for frowning?
a) Orbicularis oculi
b) Corrugator supercilli
c) Levator labii superioris
d) Levator palpebrae superioris
Explanation: The corrugator supercilli muscle draws the eyebrows medially and downward, creating vertical wrinkles above the nose, producing a frown. Orbicularis oculi closes eyelids but does not produce frown. Correct answer is b) Corrugator supercilli.
Guessed Question 6
In Bell’s palsy, which muscle is typically affected?
a) Orbicularis oculi
b) Sternocleidomastoid
c) Masseter
d) Temporalis
Explanation: Bell’s palsy involves unilateral facial nerve (CN VII) paralysis, impairing the orbicularis oculi. This results in inability to close the eye fully, leading to dryness and risk of keratitis. Other muscles are innervated differently. Correct answer is a) Orbicularis oculi.
Guessed Question 7
Which muscle elevates the upper lip?
a) Orbicularis oculi
b) Levator labii superioris
c) Corrugator supercilli
d) Levator palpebrae superioris
Explanation: The levator labii superioris elevates the upper lip, aiding facial expressions such as smiling or showing disdain. It is not involved in eyelid movements. Correct answer is b) Levator labii superioris.
Guessed Question 8
The orbicularis oculi muscle is divided into which parts?
a) Palpebral and orbital parts
b) Nasal and palpebral parts
c) Orbital and maxillary parts
d) Palpebral and zygomatic parts
Explanation: The orbicularis oculi muscle is composed of two parts: the palpebral part (involved in gentle eyelid closure such as blinking or winking) and the orbital part (responsible for forceful closure of the eyelids). Correct answer is a) Palpebral and orbital parts.
Guessed Question 9
Which artery supplies blood to orbicularis oculi muscle?
a) Facial artery
b) Ophthalmic artery
c) Maxillary artery
d) External carotid artery
Explanation: The ophthalmic artery, a branch of the internal carotid artery, provides blood supply to the orbicularis oculi muscle. It is critical for maintaining eyelid function and ocular surface health. Correct answer is b) Ophthalmic artery.
Guessed Question 10
Which clinical test assesses orbicularis oculi function?
a) Corneal reflex test
b) Visual acuity test
c) Rinne test
d) Weber test
Explanation: The corneal reflex test assesses the sensory and motor components of blinking, evaluating the function of the trigeminal (sensory) and facial (motor to orbicularis oculi) nerves. Lack of eyelid closure indicates dysfunction of the orbicularis oculi. Correct answer is a) Corneal reflex test.
Keyword Definitions
• Hip flexion – Movement decreasing the angle between thigh and trunk.
• Iliopsoas – Primary hip flexor formed by psoas major and iliacus; inserts on lesser trochanter.
• Psoas major – Lumbar-origin muscle (T12–L5) flexing hip; innervated by L1–L3 (direct branches).
• Iliacus – Iliac fossa muscle joining psoas; femoral nerve (L2–L3).
• Rectus femoris – Biarticular quadriceps head; flexes hip, extends knee; origin AIIS.
• Sartorius – ASIS origin; flexes, abducts, laterally rotates hip; flexes knee.
• Tensor fasciae latae (TFL) – Assists hip flexion/abduction; superior gluteal nerve (L4–S1).
• Femoral nerve – L2–L4 nerve to anterior thigh flexors/extensors.
• Thomas test – Assesses fixed flexion deformity of hip/iliopsoas contracture.
• Psoas sign – Pain with resisted hip flexion/extension of hip suggesting psoas irritation (e.g., retrocecal appendicitis).
Chapter: Anatomy / Lower Limb
Topic: Muscles of the Hip
Subtopic: Flexors of the Hip Joint
Lead Question – 2013
Muscle causing flexion of hip ?
a) Biceps femoris
b) Psoas major
c) Gluteus maximus
d) TFL
Explanation: The prime mover for hip flexion is iliopsoas; among options, psoas major is the principal flexor. TFL assists but is not the main flexor. Biceps femoris mainly extends hip and flexes knee; gluteus maximus extends hip. Correct answer: Psoas major.
Guessed Questions for NEET PG
1) Primary insertion site of the main hip flexor complex is?
a) Greater trochanter
b) Lesser trochanter
c) Intertrochanteric line
d) Iliac crest
Explanation: Iliopsoas (psoas major + iliacus) inserts onto the lesser trochanter of the femur via a common tendon, providing a powerful flexion vector across the hip with some external rotation. Correct answer: Lesser trochanter.
2) Nerve supply of iliacus is via?
a) Obturator nerve
b) Femoral nerve
c) Sciatic nerve
d) Superior gluteal nerve
Explanation: Iliacus is innervated by the femoral nerve (L2–L3). In contrast, psoas major receives direct branches from the lumbar plexus (L1–L3). Understanding dual innervation patterns aids clinical localization of weakness. Correct answer: Femoral nerve.
3) A footballer with pain on resisted straight-leg raise; MRI shows AIIS avulsion. Weakness likely in?
a) Hip extension
b) Hip flexion
c) Knee flexion
d) Hip adduction
Explanation: AIIS avulsion implicates rectus femoris origin. Rectus femoris flexes the hip and extends the knee; acute injury produces painful/weak hip flexion on straight-leg raise. Correct answer: Hip flexion.
4) Thomas test detects contracture of which muscle group?
a) Hip abductors
b) Hip extensors
c) Hip flexors (iliopsoas)
d) Hip adductors
Explanation: Thomas test identifies fixed flexion deformity due to tightness/contracture of iliopsoas. The contralateral hip is maximally flexed; lumbar lordosis flattens; a rising contralateral thigh indicates flexor contracture. Correct answer: Hip flexors (iliopsoas).
5) Which muscle is NOT a primary hip flexor?
a) Sartorius
b) Pectineus
c) Gluteus medius
d) Rectus femoris
Explanation: Gluteus medius chiefly abducts and stabilizes pelvis; it is not a hip flexor. Sartorius and rectus femoris flex the hip; pectineus mainly adducts but contributes to flexion. Correct answer: Gluteus medius.
6) Superior gluteal nerve injury most reduces which assisting action at hip?
a) Flexion by TFL
b) Extension by gluteus maximus
c) Adduction by gracilis
d) External rotation by piriformis
Explanation: Superior gluteal nerve (L4–S1) supplies TFL, gluteus medius, and minimus. Injury impairs abduction and internal rotation and reduces TFL-assisted hip flexion. Correct answer: Flexion by TFL.
7) In femoral nerve palsy, which combined movement is most impaired?
a) Hip flexion with knee extension
b) Hip extension with knee flexion
c) Hip adduction with knee flexion
d) Hip abduction with knee extension
Explanation: Femoral nerve supplies iliacus and quadriceps (including rectus femoris). Palsy weakens hip flexion and abolishes knee extension, impairing sit-to-stand and stair climbing. Correct answer: Hip flexion with knee extension.
8) A patient with retrocecal appendicitis has pain on passive extension of right hip. Irritation involves?
a) Gluteus maximus
b) Psoas major
c) Adductor magnus
d) Obturator externus
Explanation: The psoas sign indicates psoas major irritation; stretching the inflamed muscle by hip extension elicits pain. This localizes retroperitoneal inflammation near the psoas. Correct answer: Psoas major.
9) Which root value chiefly powers hip flexion during straight-leg raise?
a) L1
b) L2–L3
c) L4–L5
d) S1–S2
Explanation: Hip flexion strength correlates with L2–L3 myotomes (iliopsoas). Testing resisted hip flexion helps localize radiculopathy to upper lumbar roots. Correct answer: L2–L3.
10) A sprinter has pain at ASIS with weakness in crossing legs. Most likely muscle injured?
a) Sartorius
b) Rectus femoris
c) Pectineus
d) Iliopsoas
Explanation: Sartorius originates at ASIS and flexes, abducts, and laterally rotates the hip—used to cross legs (“tailor’s muscle”). ASIS tenderness with these deficits suggests sartorius strain/avulsion. Correct answer: Sartorius.
Chapter: Anatomy
Topic: Lower Limb
Subtopic: Greater Trochanter and Muscle Attachments
Keyword Definitions:
Greater Trochanter: A large projection on the lateral upper part of the femur serving as a major muscle attachment site.
Gluteus Medius: Muscle originating from the outer surface of ilium, inserted into the lateral surface of greater trochanter; chief abductor of hip.
Gluteus Minimus: Smallest gluteal muscle, inserted into the anterior surface of greater trochanter.
Piriformis: Muscle from sacrum inserted on the superior border of greater trochanter; external rotator of hip.
Gluteus Maximus: Largest gluteal muscle inserted mainly into gluteal tuberosity and iliotibial tract.
Clinical Relevance: Trendelenburg sign occurs when gluteus medius/minimus are weak or damaged.
Lead Question - 2013
Muscle attached to lateral surface of greater trochanter -
a) Gluteus maximus
b) Gluteus medius
c) Gluteus minimus
d) Piriformis
Explanation: The gluteus medius inserts into the lateral surface of the greater trochanter. Gluteus minimus attaches to its anterior surface, and piriformis to its superior border. Gluteus maximus does not insert here. The correct answer is b) Gluteus medius.
Guessed Question 2
Which muscle attaches to the anterior surface of greater trochanter?
a) Gluteus medius
b) Gluteus minimus
c) Piriformis
d) Quadratus femoris
Explanation: Gluteus minimus inserts into the anterior surface of the greater trochanter and works with gluteus medius to abduct and medially rotate the thigh. The correct answer is b) Gluteus minimus.
Guessed Question 3
Piriformis inserts into which part of the greater trochanter?
a) Inferior border
b) Superior border
c) Lateral surface
d) Posterior surface
Explanation: Piriformis muscle, arising from anterior sacrum, inserts into the superior border of the greater trochanter and is an important landmark for gluteal neurovascular structures. The correct answer is b) Superior border.
Guessed Question 4
Damage to the superior gluteal nerve affects which muscle attached to greater trochanter?
a) Gluteus medius
b) Piriformis
c) Quadratus femoris
d) Obturator externus
Explanation: The superior gluteal nerve supplies gluteus medius and minimus. Damage causes Trendelenburg gait due to failure of pelvic stabilization during walking. The correct answer is a) Gluteus medius.
Guessed Question 5
Trendelenburg sign results from paralysis of which muscle?
a) Gluteus medius
b) Gluteus maximus
c) Piriformis
d) Iliopsoas
Explanation: Trendelenburg sign occurs when gluteus medius and minimus (hip abductors inserting on greater trochanter) are weak or paralyzed, leading to pelvic drop on the opposite side. The correct answer is a) Gluteus medius.
Guessed Question 6
A patient with injection injury in gluteal region develops pelvic tilt. Most likely nerve involved?
a) Superior gluteal nerve
b) Inferior gluteal nerve
c) Obturator nerve
d) Sciatic nerve
Explanation: Superior gluteal nerve injury paralyzes gluteus medius and minimus, both attached to the greater trochanter, causing pelvic drop and Trendelenburg gait. The correct answer is a) Superior gluteal nerve.
Guessed Question 7
Which muscle attaching to greater trochanter helps in medial rotation of the hip?
a) Gluteus minimus
b) Piriformis
c) Quadratus femoris
d) Obturator externus
Explanation: Gluteus minimus assists in medial rotation of the thigh at the hip joint, apart from abduction. It inserts on the anterior surface of greater trochanter. The correct answer is a) Gluteus minimus.
Guessed Question 8
In posterior hip dislocation, which muscle attached to greater trochanter is often injured?
a) Piriformis
b) Gluteus medius
c) Quadratus femoris
d) Gluteus maximus
Explanation: In posterior hip dislocation, piriformis and gluteus medius, both attached to greater trochanter, are vulnerable to stretching and tearing due to displacement of femoral head. The correct answer is a) Piriformis.
Guessed Question 9
Which bony prominence serves as an important landmark for intramuscular injections in gluteal region?
a) Greater trochanter
b) Lesser trochanter
c) Ischial spine
d) Iliac crest
Explanation: Greater trochanter is used along with iliac crest to identify safe upper outer quadrant for intramuscular injections in gluteal region, avoiding sciatic nerve. The correct answer is a) Greater trochanter.
Guessed Question 10
Which muscle attaching to greater trochanter is primarily an external rotator of hip?
a) Piriformis
b) Gluteus medius
c) Gluteus minimus
d) Tensor fascia lata
Explanation: Piriformis is primarily an external rotator of the hip joint. It attaches to the superior border of the greater trochanter and is clinically important in piriformis syndrome. The correct answer is a) Piriformis.
Guessed Question 11
Superior gluteal artery damage would compromise blood supply to which muscle attached to greater trochanter?
a) Gluteus medius
b) Piriformis
c) Adductor magnus
d) Quadriceps femoris
Explanation: Superior gluteal artery supplies gluteus medius and minimus, both attached to greater trochanter. Injury reduces blood supply and weakens hip abduction. The correct answer is a) Gluteus medius.
Keyword Definitions
• Gastrocnemius – Superficial, two-headed calf muscle crossing knee and ankle; powerful plantarflexor.
• Soleus – Deep to gastrocnemius, single-headed, plantarflexes ankle (postural muscle).
• Triceps surae – Collective term for gastrocnemius + soleus muscles forming the calf.
• Calcaneal (Achilles) tendon – Common tendon of triceps surae inserting into calcaneus.
• Tibial nerve – Branch of sciatic nerve supplying posterior compartment of leg and plantar foot.
• Plantarflexion – Downward movement of foot at ankle produced by triceps surae.
• Sural nerve – Sensory nerve formed by contributions from tibial and common peroneal nerves; supplies lateral foot.
• Posterior tibial artery – Major artery supplying posterior compartment and plantar foot.
• Baker’s cyst – Popliteal synovial cyst that may compress neurovascular structures in popliteal fossa.
• Achilles tendon rupture – Clinical injury causing inability to plantarflex and toe-raise; Thompson test positive.
Chapter: Anatomy / Lower Limb
Topic: Posterior Compartment of Leg
Subtopic: Triceps Surae (Gastrocnemius & Soleus)
Lead Question – 2013
Which muscles is known as 'Triceps surae'?
a) Gastro-soleus
b) Popliteus
c) EHL
d) EDL
Explanation: “Triceps surae” refers to the gastrocnemius and soleus acting together as the calf complex. Their common insertion via the calcaneal (Achilles) tendon produces powerful plantarflexion. Popliteus, EHL, and EDL are distinct muscles with different functions. Correct answer: Gastro-soleus (triceps surae).
Guessed Questions for NEET PG
1) Which nerve supplies triceps surae?
a) Common peroneal
b) Tibial
c) Femoral
d) Sural
Explanation: Triceps surae (gastrocnemius and soleus) are supplied by the tibial nerve, a branch of the sciatic nerve. Motor fibers reach muscles in the posterior compartment and sensory branches form part of the sural nerve. Correct answer: Tibial nerve. Clinically important in sciatic lesions.
2) The common tendon of triceps surae inserts into the:
a) Navicular
b) Calcaneus
c) Cuboid
d) Talus
Explanation: The calcaneal (Achilles) tendon attaches the triceps surae to the posterior calcaneus. This insertion transmits strong plantarflexor force. Rupture here causes inability to plantarflex and a positive Thompson test. Correct answer: Calcaneus.
3) Thompson test assesses rupture of which structure?
a) Plantaris tendon
b) Calcaneal (Achilles) tendon
c) Tibialis posterior tendon
d) Peroneus brevis tendon
Explanation: Squeezing the calf normally causes plantarflexion; absence indicates Achilles tendon rupture. Thompson test is thus specific for calcaneal tendon discontinuity, often from sudden dorsiflexion injury. Correct answer: Calcaneal (Achilles) tendon.
4) Which artery mainly supplies the triceps surae muscles?
a) Anterior tibial artery
b) Posterior tibial artery and its branches (peroneal included)
c) Dorsalis pedis artery
d) Femoral artery only
Explanation: The posterior tibial artery and contributions from the peroneal (fibular) artery supply the posterior compartment including triceps surae. Anterior tibial and dorsalis pedis supply anterior structures. Correct answer: Posterior tibial artery and branches. Important in ischemic leg evaluation.
5) Gastrocnemius contributes more to plantarflexion when the knee is:
a) Flexed
b) Extended
c) Neutral – unaffected by knee position
d) Internally rotated
Explanation: Gastrocnemius crosses the knee; it generates more plantarflexion when the knee is extended, as knee flexion slackens it. Soleus (monoarticular) acts regardless of knee position. Correct answer: Extended. Clinical tests consider knee position when assessing calf strength.
6) A positive calf squeeze with no plantarflexion indicates lesion of:
a) Posterior tibial nerve distal to gastrocnemius
b) Achilles tendon rupture
c) Tibialis anterior nerve injury
d) Peroneal nerve palsy
Explanation: The Thompson (calf squeeze) test shows absent plantarflexion in Achilles tendon rupture, not in isolated nerve palsy. Tendon discontinuity prevents force transmission despite intact muscle. Correct answer: Achilles tendon rupture. Prompt management needed to restore push-off.
7) Plantaris muscle, when present, lies between which two structures related to triceps surae?
a) Tibialis anterior and EHL
b) Gastrocnemius and soleus, with a long tendon medial to Achilles
c) Peroneus longus and brevis
d) Flexor hallucis longus and tibialis posterior
Explanation: Plantaris is a small accessory muscle with a long slender tendon that runs between gastrocnemius and soleus and often parallels the Achilles tendon medially. It is sometimes used as a tendon graft. Correct answer: Gastrocnemius and soleus region.
8) Rupture of the Achilles tendon most commonly results in loss of which gait phase action?
a) Heel strike
b) Toe-off/pushoff (plantarflexion phase)
c) Mid-stance stability only
d) Swing phase clearance
Explanation: Achilles rupture prevents effective plantarflexion needed for toe-off/pushoff, leading to impaired propulsion and altered gait. Patients have weak push-off and may show a palpable gap. Correct answer: Toe-off/pushoff. Surgical repair often restores function.
9) Which clinical condition involves pain and tightness of triceps surae with increased compartment pressure?
a) Plantar fasciitis
b) Posterior compartment syndrome of leg
c) Lateral ankle sprain
d) Deep vein thrombosis only
Explanation: Exertional or acute posterior compartment syndrome involves the deep posterior compartment including triceps surae, causing pain, tense swelling, and neurovascular compromise. Urgent fasciotomy is required. Correct answer: Posterior compartment syndrome (affecting calf muscles).
10) Sural nerve sensory distribution relates closely to triceps surae because it supplies:
a) Medial plantar surface of foot
b) Lateral aspect of foot and posterior calf skin overlying gastrocnemius
d) Plantar aspect of toes 2–4
Explanation: The sural nerve provides cutaneous innervation to the posterior calf and lateral foot, areas overlying the gastrocnemius and Achilles tendon. Injury causes sensory loss here and may accompany procedures on the calf. Correct answer: Lateral foot and posterior calf skin.
Keyword Definitions
• Interosseous membrane – Fibrous sheet between radius and ulna, provides attachment for muscles and transmits forces.
• Anterior interosseous artery – Branch of common interosseous artery running on anterior surface of interosseous membrane supplying deep forearm muscles.
• Posterior interosseous artery – Branch that reaches the posterior compartment, often passes through/perforates the interosseous membrane to supply extensors.
• Common interosseous artery – Short trunk from ulnar artery dividing into anterior and posterior interosseous arteries.
• Interosseous space – The gap between radius and ulna occupied by membrane and vessels; communicates between compartments.
• Posterior interosseous nerve – Deep branch of radial nerve running in posterior compartment with posterior interosseous vessels.
• Perforating branches – Small vessels that traverse the interosseous membrane to connect anterior and posterior circulations.
• Clinical relevance – Knowledge is vital in forearm fractures and surgical approaches to avoid vascular injury.
• Supination/pronation force transmission – Interosseous membrane transmits axial loads from radius to ulna during weight-bearing.
• Surgical landmark – Interosseous membrane used as reference during forearm reconstructive procedures.
Chapter: Anatomy / Upper Limb
Topic: Forearm Vessels and Membranes
Subtopic: Interosseous Membrane and its Perforators
Lead Question – 2013
Interosseous membrane of forearm is pierced by?
a) Brachial artery
b) Anterior interosseous artery
c) Posterior interosseous artery
d) Ulnar recurrent artery
Explanation: The posterior interosseous artery typically pierces the interosseous membrane to reach the posterior compartment, accompanying the posterior interosseous nerve. The anterior interosseous artery runs on the anterior surface and sends perforators. Correct answer: Posterior interosseous artery. Clinically important in posterior compartment surgeries and fractures.
Guessed Questions for NEET PG
1) The anterior interosseous artery is a branch of:
a) Radial artery
b) Ulnar artery (via common interosseous)
c) Brachial artery directly
d) Posterior interosseous artery
Explanation: The anterior interosseous artery arises from the common interosseous branch of the ulnar artery and runs on the anterior surface of the interosseous membrane. Correct answer: Ulnar artery (via common interosseous). Clinical: AIN and artery are vulnerable in proximal forearm trauma.
2) Which nerve accompanies the posterior interosseous artery in the posterior compartment?
a) Superficial radial nerve
b) Posterior interosseous nerve (deep branch of radial)
c) Median nerve
d) Ulnar nerve
Explanation: The posterior interosseous nerve (deep branch of radial nerve) accompanies the posterior interosseous artery in the posterior compartment to supply extensor muscles. Correct answer: Posterior interosseous nerve. Clinical: Injury causes finger extension weakness without sensory loss.
3) Perforating branches of the interosseous arteries allow communication between:
a) Radial and ulnar arteries only
b) Anterior and posterior compartments of forearm
c) Superficial and deep palmar arches
d) Brachial and radial arteries
Explanation: Perforators through the interosseous membrane connect anterior and posterior interosseous arteries, providing collateral circulation between forearm compartments. Correct answer: Anterior and posterior compartments. Clinical: Important when primary vessels are injured.
4) Injury to posterior interosseous artery in proximal forearm most likely causes:
a) Pure sensory loss in hand
b) Ischemia of posterior compartment muscles
c) Loss of pronation only
d) Thumb adduction loss
Explanation: Damage to posterior interosseous artery reduces blood supply to posterior (extensor) compartment leading to ischemic pain and weakness. Correct answer: Ischemia of posterior compartment muscles. Clinical: May accompany fractures or surgical insults.
5) The common interosseous artery usually arises from:
a) Radial artery
b) Ulnar artery
c) Brachial artery at cubital fossa
d) Profunda brachii artery
Explanation: The common interosseous artery branches from the ulnar artery shortly after the ulnar origin, then divides into anterior and posterior interosseous arteries. Correct answer: Ulnar artery. Clinical: Variant anatomy can affect flap planning.
6) Which structure runs along the anterior surface of the interosseous membrane?
a) Posterior interosseous artery
b) Anterior interosseous artery and nerve
c) Superficial radial nerve
d) Ulnar nerve
Explanation: The anterior interosseous artery and anterior interosseous branch of median nerve run on the anterior surface of the interosseous membrane supplying deep flexors. Correct answer: Anterior interosseous artery and nerve. Clinical: AIN palsy causes pure motor deficits.
7) The posterior interosseous artery usually reaches the posterior compartment via a gap near which landmark?
a) Lister’s tubercle
b) Proximal border of interosseous membrane near supinator
c) Ulnar styloid process
d) Pisiform bone
Explanation: The posterior interosseous artery commonly passes to the posterior compartment near the proximal border of the interosseous membrane in the region of the supinator. Correct answer: Proximal border of interosseous membrane near supinator. Clinical: Supinator syndrome may compromise vessels and nerve.
8) In a Galeazzi fracture (distal radius with DRUJ disruption), which artery's flow might be compromised affecting interosseous communication?
a) Brachial artery
b) Anterior interosseous artery
c) Posterior tibial artery
d) Median artery
Explanation: A distal radius fracture can disturb branches including anterior interosseous artery or its perforators, impairing collateral flow between compartments. Correct answer: Anterior interosseous artery. Clinical: Assess distal perfusion and nerve function in such injuries.
9) The anterior interosseous artery supplies all EXCEPT:
a) Flexor pollicis longus
b) Pronator quadratus
c) Lateral part of flexor digitorum profundus
d) Extensor digitorum communis
Explanation: The AIN supplies FPL, pronator quadratus, and lateral FDP; it does not supply extensor digitorum communis (posterior compartment). Correct answer: Extensor digitorum communis. Clinical: AIN lesions cause weak thumb and index flexion.
10) Surgical exposure of the posterior forearm should avoid injury to which vessel that pierces the interosseous membrane?
a) Radial artery
b) Posterior interosseous artery
c) Ulnar artery
d) Cephalic vein
Explanation: The posterior interosseous artery pierces the interosseous membrane to reach the posterior compartment and must be preserved during surgical approaches to avoid ischemia of extensor muscles. Correct answer: Posterior interosseous artery. Careful dissection around supinator is required.
Keyword Definitions
• Extensor compartments – Six fibro-osseous dorsal compartments on wrist that guide extensor tendons beneath extensor retinaculum.
• Extensor pollicis longus (EPL) – Tendon that extends the thumb interphalangeal joint; runs in 3rd compartment around Lister’s tubercle.
• Extensor carpi radialis longus (ECRL) – Wrist extensor and radial abductor; runs in 2nd compartment (partly).
• Extensor carpi radialis brevis (ECRB) – Wrist extensor in 2nd compartment; often involved in lateral epicondylitis.
• Extensor pollicis brevis (EPB) – Short thumb extensor in 1st compartment with APL.
• Lister’s tubercle – Dorsal tubercle of radius that acts as a pulley for EPL tendon (3rd compartment).
• De Quervain’s tenosynovitis – Stenosing tenosynovitis of 1st dorsal compartment (APL, EPB).
• Intersection syndrome – Overuse tenosynovitis where 1st compartment tendons cross 2nd compartment tendons.
• Extensor retinaculum – Fibrous band holding extensor tendons in compartments at wrist.
• Clinical importance – Identifying compartment involved helps diagnose dorsal wrist pain and plan surgical release.
Chapter: Anatomy / Upper Limb
Topic: Wrist and Hand
Subtopic: Dorsal Extensor Compartments of Wrist
Lead Question – 2013
3rd extensor compartment of wrist contains tendon of ?
a) ECRL
b) ECRB
c) EPL
d) EPB
Explanation: The third dorsal compartment contains the extensor pollicis longus (EPL) tendon as it uses Lister’s tubercle as a pulley, redirecting its line of pull to extend the thumb. Correct answer: EPL. Clinically, EPL rupture may follow distal radius fractures and presents as loss of thumb IP extension.
Guessed Questions for NEET PG
1) First dorsal compartment contains tendons of:
a) APL & EPB
b) EPL only
c) ECRL & ECRB
d) Extensor digitorum
Explanation: The first dorsal compartment contains abductor pollicis longus (APL) and extensor pollicis brevis (EPB). Stenosis here causes de Quervain’s tenosynovitis with radial wrist pain. Correct answer: APL & EPB. Treatment includes splinting or compartmental release.
2) Second dorsal compartment contains which tendons?
a) ECRL & ECRB
b) EPL only
c) APL & EPB
d) Extensor digiti minimi
Explanation: The second compartment contains extensor carpi radialis longus and brevis (ECRL, ECRB). These tendons glide under the retinaculum and are implicated in intersection syndrome when irritated by crossing first-compartment tendons. Correct answer: ECRL & ECRB.
3) Lister’s tubercle is clinically significant because it:
a) Is attachment for ECU
b) Acts as pulley for EPL tendon
c) Houses radial artery
d) Is origin of APL
Explanation: Lister’s tubercle on the distal radius acts as a dorsal pulley for the EPL tendon, changing its direction toward the thumb. After distal radius fractures, EPL attrition or rupture can occur here. Correct answer: Acts as pulley for EPL tendon.
4) De Quervain’s tenosynovitis typically presents with pain at:
a) Ulnar styloid
b) Radial styloid / lateral wrist
c) Dorsal midcarpal region
d) Pisiform area
Explanation: De Quervain’s affects APL and EPB in the first dorsal compartment causing pain and tenderness at the radial styloid. Finkelstein’s test reproduces pain. Correct answer: Radial styloid / lateral wrist. Management includes splinting and steroid injection.
5) Extensor digitorum communis (EDC) tendons lie in which compartment primarily?
a) Third
b) Fourth
c) Fifth
d) Sixth
Explanation: The fourth dorsal compartment houses the extensor digitorum communis (EDC) tendons and extensor indicis. Correct answer: Fourth. Clinical: Extensor tendon injuries in this compartment affect finger extension and may require repair or tenodesis.
6) Extensor digiti minimi runs in which compartment?
a) First
b) Second
c) Fifth
d) Sixth
Explanation: The fifth dorsal compartment contains the extensor digiti minimi tendon (to little finger). Tenosynovitis here causes localized dorsal ulnar wrist pain. Correct answer: Fifth compartment. Surgical release may be needed for refractory cases.
7) Extensor carpi ulnaris (ECU) tendon lies in which compartment?
a) Third
b) Fourth
c) Fifth
d) Sixth
Explanation: The sixth dorsal compartment contains the extensor carpi ulnaris (ECU) tendon running along the ulnar side. ECU subluxation or tendinopathy causes ulnar-sided wrist pain, especially in racket sports. Correct answer: Sixth compartment.
8) Intersection syndrome involves friction where first compartment tendons cross which compartment?
a) Second compartment tendons
b) Third compartment tendons
c) Fourth compartment tendons
d) Fifth compartment tendons
Explanation: Intersection syndrome results from friction where APL/EPB (1st compartment) cross over ECRL/ECRB (2nd compartment) about 4–8 cm proximal to wrist, producing forearm pain and crepitus. Correct answer: Second compartment tendons.
9) Rupture of EPL tendon is most commonly associated with which injury?
a) Distal radius fracture
b) Scaphoid fracture
c) Hamate fracture
d) Colles’ dislocation only
Explanation: EPL rupture classically follows distal radius fractures due to attrition at Lister’s tubercle or ischemia of the tendon sheath. Patients lose active IP extension of thumb. Correct answer: Distal radius fracture. Surgical tendon transfer may be required.
10) A swollen dorsal wrist with pain on thumb extension and positive Finkelstein’s test indicates involvement of which compartment?
a) First compartment
b) Third compartment
c) Fourth compartment
d) Sixth compartment
Explanation: Positive Finkelstein’s test with radial styloid tenderness indicates first compartment stenosing tenosynovitis (APL & EPB) — de Quervain’s disease. Correct answer: First compartment. Conservative treatment includes rest, splinting, and steroid injection.
Keyword Definitions
• Supination – Outward rotation of forearm turning palm upward.
• Pronation – Inward rotation of forearm turning palm downward.
• Biceps brachii – Flexor of elbow and chief supinator when forearm flexed.
• Supinator muscle – Assists in supination, especially when elbow extended.
• Brachioradialis – Flexes elbow, acts in mid-prone position, not supination.
• FDS (Flexor digitorum superficialis) – Flexes middle phalanges, unrelated to supination.
• Anconeus – Assists in elbow extension, stabilizes joint, not in supination.
• Musculocutaneous nerve – Innervates biceps brachii.
• Radial nerve – Innervates supinator and brachioradialis.
• Clinical test – Turning a screwdriver tests supination and biceps activity.
• Spiral groove – Radial nerve and profunda brachii pass here, relevant for supinator function.
Chapter: Anatomy / Upper Limb
Topic: Muscles of Forearm and Arm
Subtopic: Supinators of Forearm
Lead Question – 2013
Muscle causing supination of forearm?
a) Biceps brachii
b) Brachioradialis
c) FDS
d) Anconeus
Explanation: Supination is mainly caused by biceps brachii when the elbow is flexed and by supinator when the elbow is extended. Other listed muscles are not supinators. Correct answer: (a) Biceps brachii. Clinical: Weak supination occurs in musculocutaneous nerve or radial nerve injury.
Guessed Questions for NEET PG
1) Supination in extended elbow is mainly by?
a) Supinator
b) Biceps brachii
c) Pronator teres
d) Anconeus
Explanation: Supinator acts strongly when elbow is extended, while biceps dominates in flexion. Correct answer: Supinator. Clinical: Radial nerve injury weakens supination.
2) Which nerve supplies the supinator muscle?
a) Median
b) Radial (deep branch)
c) Musculocutaneous
d) Ulnar
Explanation: Supinator is supplied by deep branch of radial nerve. Correct answer: Radial nerve (deep branch). Clinical: Compression in supinator leads to posterior interosseous nerve syndrome.
3) Turning a screwdriver involves mainly?
a) Pronator quadratus
b) Biceps brachii
c) Anconeus
d) Flexor carpi radialis
Explanation: Biceps brachii provides strong supination, especially in flexed forearm. Correct answer: Biceps brachii. Clinical: Fatigue in this muscle seen in repetitive screw turning.
4) A patient with musculocutaneous nerve injury will have weakness in?
a) Supination with flexed elbow
b) Supination with extended elbow
c) Pronation
d) Wrist extension
Explanation: Musculocutaneous nerve supplies biceps brachii, the chief supinator in flexion. Correct answer: Supination with flexed elbow. Clinical: Weakness plus sensory loss lateral forearm.
5) Which movement is preserved in posterior interosseous nerve injury?
a) Finger extension
b) Supination with flexed elbow
c) Wrist extension
d) Thumb abduction
Explanation: Biceps brachii (musculocutaneous) compensates supination in flexion. Correct answer: Supination with flexed elbow. Clinical: Posterior interosseous palsy spares biceps function.
6) Which muscle is a synergist in both pronation and supination, bringing forearm to mid-prone?
a) Supinator
b) Biceps brachii
c) Brachioradialis
d) Pronator teres
Explanation: Brachioradialis brings forearm into mid-prone from either side. Correct answer: Brachioradialis. Clinical: Preserved action in radial nerve palsy proximal to its branch.
7) Supination test in clinical examination mainly evaluates?
a) Median nerve
b) Musculocutaneous nerve
c) Ulnar nerve
d) Axillary nerve
Explanation: Supination in flexion depends on biceps brachii, innervated by musculocutaneous nerve. Correct answer: Musculocutaneous nerve. Clinical: Used to assess injury after trauma.
8) Which muscle is NOT involved in forearm supination?
a) Supinator
b) Biceps brachii
c) Brachioradialis
d) FDS
Explanation: FDS is a finger flexor, has no role in forearm rotation. Correct answer: FDS. Clinical: Misconception often tested in exams.
9) In radial head dislocation (nursemaid’s elbow), which movement is restricted?
a) Pronation
b) Supination
c) Flexion
d) Extension
Explanation: Dislocated radial head impairs supination since supinator attaches here. Correct answer: Supination. Clinical: Common in children lifted by hand.
10) In fracture of surgical neck of humerus sparing biceps, supination is?
a) Lost completely
b) Weak but present
c) Normal
d) Exaggerated
Explanation: Biceps brachii (musculocutaneous) remains intact; supination is preserved. Correct answer: Normal. Clinical: Differentiates between radial and musculocutaneous lesions.
Keyword Definitions
• Midpalmar space – Deep fascial space of hand located beneath central compartment, communicates with forearm via carpal tunnel.
• Lumbricals – Four small intrinsic hand muscles arising from flexor digitorum profundus tendons.
• FDP – Flexor digitorum profundus, flexes distal interphalangeal joints.
• Thenar space – Fascial space near thumb, separated from midpalmar space by septum.
• Hand compartments – Thenar, hypothenar, adductor, central, interosseous compartments.
• Clinical correlation – Midpalmar abscess can spread to forearm through carpal tunnel.
• Carpal tunnel – Passage for FDP, FDS tendons, FPL tendon, and median nerve.
• Infection spread – From finger pulp to midpalmar space via lumbrical canals.
• Surgical drainage – Important in treating deep palmar space infections.
• Lumbrical canal – Interval through which lumbrical muscles enter palm from FDP.
• Interossei – Intrinsic hand muscles not part of midpalmar space content.
Chapter: Anatomy / Upper Limb
Topic: Hand
Subtopic: Midpalmar Space
Lead Question – 2013
Contents of midpalmar space are all except
a) 2nd lumbrical
b) FDP of 3rd finger
c) 1st lumbrical
d) FDP of 4th finger
Explanation: Midpalmar space contains medial three lumbricals and flexor digitorum profundus tendons of middle, ring, and little fingers. The 1st lumbrical belongs to thenar space, not midpalmar. Correct answer: (c) 1st lumbrical. Clinical: Midpalmar abscess can cause swelling in central palm and requires careful surgical drainage.
Guessed Questions for NEET PG
1) Which lumbrical muscle lies in thenar space?
a) 1st lumbrical
b) 2nd lumbrical
c) 3rd lumbrical
d) 4th lumbrical
Explanation: The 1st lumbrical lies in thenar space along with FPL tendon. Others lie in midpalmar space. Correct answer: 1st lumbrical. Clinical: Swelling of thenar space may compromise thumb movements.
2) Infection from the index finger pulp may spread to?
a) Thenar space
b) Midpalmar space
c) Hypothenar space
d) Dorsum of hand
Explanation: Lumbrical canal of index finger connects pulp with thenar space. Correct answer: Thenar space. Clinical: Early drainage is essential to preserve thumb function.
3) Which tendon passes through carpal tunnel and continues into midpalmar space?
a) FPL
b) FDP
c) Extensor digitorum
d) Palmaris longus
Explanation: Flexor digitorum profundus tendons pass through carpal tunnel and form part of midpalmar space contents. Correct answer: FDP. Clinical: Infections may spread from palm to forearm via carpal tunnel.
4) Midpalmar space infection presents with swelling in which region?
a) Thenar eminence
b) Hypothenar eminence
c) Central palm
d) Dorsal web space
Explanation: Midpalmar space infections typically cause fullness in central palm. Correct answer: Central palm. Clinical: Deep abscess requires surgical drainage through palmar incision.
5) Which of the following is not a boundary of midpalmar space?
a) Palmar aponeurosis
b) Metacarpals of index finger
c) Interossei muscles
d) Flexor pollicis longus
Explanation: FPL belongs to thenar space, not boundary of midpalmar space. Correct answer: FPL. Clinical: Differentiating compartments is crucial in infection management.
6) Which lumbricals are supplied by ulnar nerve?
a) 1st and 2nd
b) 3rd and 4th
c) All four
d) None
Explanation: The 3rd and 4th lumbricals are supplied by the deep branch of ulnar nerve. Correct answer: 3rd and 4th. Clinical: Ulnar nerve injury affects fine grip due to lumbrical paralysis.
7) Patient with midpalmar space infection is unable to flex distal phalanx of ring finger. Which tendon is affected?
a) FDS
b) FDP
c) FPL
d) Extensor indicis
Explanation: FDP of ring finger passes through midpalmar space, infection can impair its function. Correct answer: FDP. Clinical: This indicates deep involvement requiring urgent drainage.
8) Which muscle group borders the midpalmar space dorsally?
a) Palmaris brevis
b) Interossei
c) Thenar muscles
d) Hypothenar muscles
Explanation: Interossei muscles form dorsal boundary of midpalmar space. Correct answer: Interossei. Clinical: Infection here can spread to intermetacarpal spaces.
9) Midpalmar space communicates with forearm via?
a) Anatomical snuffbox
b) Carpal tunnel
c) Guyon’s canal
d) Radial bursa
Explanation: Midpalmar space communicates proximally through carpal tunnel with forearm. Correct answer: Carpal tunnel. Clinical: Explains spread of deep hand infections to forearm flexor sheath.
10) Which space is affected in a patient unable to oppose thumb with swelling near 1st web space?
a) Midpalmar space
b) Thenar space
c) Hypothenar space
d) Dorsal space
Explanation: Thenar space infection affects thumb opposition and causes swelling near first web space. Correct answer: Thenar space. Clinical: Misdiagnosis may lead to permanent disability of thumb.
Keyword Definitions
• Superficial anterior compartment of forearm – Contains five muscles: pronator teres, flexor carpi radialis, palmaris longus, flexor carpi ulnaris, flexor digitorum superficialis.
• FDS – Flexor digitorum superficialis, a superficial flexor of fingers.
• FCR – Flexor carpi radialis, wrist flexor and abductor.
• Palmaris longus – Weak wrist flexor, absent in 10-15% individuals.
• FPL – Flexor pollicis longus, belongs to deep compartment, flexes thumb.
• Median nerve – Supplies most superficial anterior forearm muscles.
• Ulnar nerve – Supplies flexor carpi ulnaris and medial half of FDP.
• Brachial artery – Main arterial supply of forearm, divides into radial and ulnar.
• Clinical correlation – Superficial muscles are commonly involved in tendinitis and occupational overuse syndromes.
• Compartment syndrome – Increased pressure in forearm compartments can damage muscles and nerves.
• Pronator teres syndrome – Median nerve entrapment by pronator teres in superficial compartment.
Chapter: Anatomy / Upper Limb
Topic: Forearm Muscles
Subtopic: Superficial anterior compartment of forearm
Lead Question – 2013
Which of the following is not the muscle of superficial anterior compartment of forearm?
a) FDS
b) FPL
c) FCR
d) Palmaris longus
Explanation: The superficial anterior compartment includes pronator teres, flexor carpi radialis, palmaris longus, flexor carpi ulnaris, and flexor digitorum superficialis. Flexor pollicis longus is a deep compartment muscle. Correct answer: (b) FPL. Clinical: FPL is important in thumb flexion and tested in anterior interosseous nerve palsy.
Guessed Questions for NEET PG
1) Which of the following is a superficial flexor of forearm?
a) Pronator teres
b) Flexor pollicis longus
c) Flexor digitorum profundus
d) Supinator
Explanation: Pronator teres belongs to superficial anterior compartment. FPL and FDP are deep flexors, supinator belongs to posterior compartment. Correct answer: Pronator teres. Clinical: Median nerve may be compressed between its two heads.
2) Which superficial anterior forearm muscle is most frequently absent in population?
a) FCR
b) FCU
c) Palmaris longus
d) Pronator teres
Explanation: Palmaris longus is absent in about 10–15% of individuals. Correct answer: Palmaris longus. Clinical: Its tendon is used in reconstructive tendon graft surgeries without functional deficit.
3) Which muscle is supplied by ulnar nerve among superficial flexors?
a) FCR
b) FCU
c) Palmaris longus
d) Pronator teres
Explanation: Flexor carpi ulnaris is the only superficial flexor supplied by the ulnar nerve. Others are supplied by median nerve. Correct answer: FCU. Clinical: Weak wrist flexion and ulnar deviation occur in lesions.
4) A patient unable to flex PIP joints of fingers likely has paralysis of?
a) FDP
b) FDS
c) FPL
d) FCU
Explanation: Flexor digitorum superficialis flexes proximal interphalangeal joints. Correct answer: FDS. Clinical: Median nerve lesions at elbow impair this function.
5) Which muscle originates from common flexor origin on medial epicondyle?
a) Palmaris longus
b) Pronator teres
c) FCR
d) All of the above
Explanation: All superficial flexors except FDS deep head arise from medial epicondyle via common flexor origin. Correct answer: All of the above. Clinical: Overuse may cause medial epicondylitis (golfer’s elbow).
6) Flexor pollicis longus is supplied by?
a) Median nerve (anterior interosseous branch)
b) Ulnar nerve
c) Radial nerve
d) Posterior interosseous nerve
Explanation: FPL is innervated by anterior interosseous branch of median nerve. Correct answer: Median nerve (AIN). Clinical: AIN palsy causes inability to make “OK” sign due to FPL weakness.
7) Which superficial flexor muscle is involved in carpal tunnel syndrome symptoms due to tendinopathy?
a) FDS
b) FCR
c) FCU
d) Pronator teres
Explanation: Flexor digitorum superficialis tendons pass through carpal tunnel and may contribute to compression. Correct answer: FDS. Clinical: CTS presents with numbness, tingling in lateral fingers.
8) Median nerve lies deep to which superficial muscle at wrist?
a) FCU
b) FCR
c) Palmaris longus
d) Pronator teres
Explanation: Palmaris longus tendon lies superficial to median nerve at wrist. Correct answer: Palmaris longus. Clinical: Median nerve blocks may be guided by this relation.
9) Which superficial forearm muscle flexes and abducts the wrist?
a) FCR
b) FCU
c) Palmaris longus
d) FDS
Explanation: Flexor carpi radialis flexes wrist and abducts it towards radial side. Correct answer: FCR. Clinical: FCR tendon is palpable in radial wrist and used for arterial cannulation landmark.
10) In anterior compartment syndrome, which superficial muscle is most vulnerable due to location?
a) Pronator teres
b) FCR
c) FCU
d) FDS
Explanation: Flexor digitorum superficialis, being central and bulky, is commonly affected in increased compartment pressure. Correct answer: FDS. Clinical: Early decompression prevents ischemic contracture (Volkmann’s).
Keyword Definitions
• Dual nerve supply – Muscle receiving motor innervation from two different nerves.
• Subscapularis – Supplied by upper and lower subscapular nerves.
• Pectoralis major – Supplied by medial and lateral pectoral nerves.
• Pronator teres – Supplied by median nerve only.
• Flexor digitorum profundus – Medial half by ulnar nerve, lateral half by anterior interosseous branch of median nerve.
• Brachial plexus – Network of nerves supplying upper limb, roots C5–T1.
• Median nerve – Formed by medial and lateral cords, supplies most forearm flexors.
• Ulnar nerve – Arises from medial cord, supplies intrinsic hand muscles and medial FDP.
• Subscapular nerves – Branches of posterior cord, innervate subscapularis.
• Clinical correlation – Knowledge of dual supply important in nerve lesions and recovery.
• Muscle palsy – Weakness pattern helps localize lesion to specific nerve or part of plexus.
Chapter: Anatomy / Upper Limb
Topic: Brachial Plexus and Muscle Innervation
Subtopic: Dual nerve supply of upper limb muscles
Lead Question – 2013
All of the following muscles have dual nerve supply except?
a) Subscapularis
b) Pectoralis major
c) Pronator teres
d) Flexor digitorum profundus
Explanation: Subscapularis has dual supply (upper and lower subscapular nerves). Pectoralis major has dual supply (medial and lateral pectoral nerves). FDP has dual supply (median and ulnar nerves). Pronator teres has single supply (median nerve). Correct answer: (c) Pronator teres. Clinical: Isolated median injury can paralyze pronator teres completely.
Guessed Questions for NEET PG
1) Which muscle among the following is supplied by both ulnar and median nerves?
a) Flexor pollicis longus
b) Flexor carpi radialis
c) Flexor digitorum profundus
d) Pronator quadratus
Explanation: Flexor digitorum profundus has dual supply – medial half by ulnar, lateral half by anterior interosseous (median). Correct answer: FDP. Clinical: Explains partial preservation in isolated lesions.
2) Subscapularis is supplied by?
a) Upper and lower subscapular nerves
b) Thoracodorsal nerve
c) Lateral pectoral nerve
d) Axillary nerve
Explanation: Subscapularis is innervated by both upper and lower subscapular nerves from posterior cord. Correct answer: Upper and lower subscapular nerves. Clinical: Injury leads to weak internal rotation.
3) Which muscle receives innervation from both medial and lateral pectoral nerves?
a) Pectoralis major
b) Pectoralis minor
c) Subclavius
d) Serratus anterior
Explanation: Pectoralis major is supplied by medial and lateral pectoral nerves. Correct answer: Pectoralis major. Clinical: Paralysis leads to weak adduction and internal rotation.
4) A patient with ulnar nerve lesion at wrist retains partial flexion of DIP of ring finger due to?
a) Median nerve supply
b) Radial nerve supply
c) Musculocutaneous nerve supply
d) Axillary nerve supply
Explanation: Lateral half of FDP (index and middle fingers) supplied by median, medial half (ring and little fingers) by ulnar. Correct answer: Median nerve supply. Clinical: Explains incomplete loss in ulnar palsy.
5) Which of the following has single nerve supply?
a) FDP
b) Pectoralis major
c) Pronator teres
d) Subscapularis
Explanation: Pronator teres is solely supplied by median nerve. Others have dual innervation. Correct answer: Pronator teres. Clinical: Useful in lesion localization.
6) Which nerve supplies medial half of flexor digitorum profundus?
a) Ulnar
b) Median
c) Radial
d) Musculocutaneous
Explanation: Medial half (ring and little fingers) of FDP is innervated by ulnar nerve. Correct answer: Ulnar nerve. Clinical: Explains weakness of DIP flexion in ulnar palsy.
7) Damage to lateral pectoral nerve causes weakness in?
a) Shoulder abduction
b) Arm adduction
c) Elbow flexion
d) Wrist extension
Explanation: Lateral pectoral nerve innervates pectoralis major, main action is adduction and internal rotation of arm. Correct answer: Arm adduction. Clinical: Loss of powerful adduction in lesion.
8) A patient with lesion of posterior cord affecting both upper and lower subscapular nerves shows weakness in?
a) Internal rotation
b) External rotation
c) Abduction
d) Supination
Explanation: Subscapularis performs internal rotation of humerus, supplied by both upper and lower subscapular nerves. Correct answer: Internal rotation. Clinical: Shoulder stability is also reduced.
9) Median nerve injury at elbow spares which of the following?
a) Pronator teres
b) Flexor digitorum profundus (medial half)
c) Flexor digitorum superficialis
d) Flexor pollicis longus
Explanation: Medial half of FDP is supplied by ulnar nerve, hence spared in median nerve injury at elbow. Correct answer: FDP (medial half). Clinical: Explains partial preservation of finger flexion.
10) Which of the following combinations represent dual innervation correctly?
a) FDP – Median & Ulnar
b) Pectoralis major – Medial & Lateral pectoral
c) Subscapularis – Upper & Lower subscapular
d) All of the above
Explanation: All mentioned muscles are examples of dual innervation. Correct answer: All of the above. Clinical: Important for understanding muscle function in partial nerve injuries.
Keyword Definitions
• Pectoral region – The area on the anterior chest wall related to pectoral muscles.
• Pectoralis major – Large superficial chest muscle aiding arm adduction and medial rotation.
• Pectoralis minor – Lies deep to pectoralis major, attaches coracoid process to ribs.
• Subclavius – Small muscle beneath clavicle, stabilizes clavicle.
• Infraspinatus – Rotator cuff muscle of scapula, not part of pectoral region.
• Axilla – Space beneath pectoral region transmitting vessels and nerves.
• Thoracoacromial artery – Main arterial supply of pectoral region.
• Medial and lateral pectoral nerves – Innervation of pectoralis muscles.
• Clinical correlation – Trauma or surgery of chest may injure pectoral nerves.
• Rotator cuff – Group of muscles stabilizing shoulder joint, includes infraspinatus.
• Breast surgery relevance – Pectoralis muscles form bed of breast and are key in mastectomy.
Chapter: Anatomy / Upper Limb
Topic: Pectoral Region and Axilla
Subtopic: Muscles of pectoral region
Lead Question – 2013
Which of the following muscle is not in the pectoral region?
a) Pectoralis major
b) Infraspinatus
c) Pectoralis minor
d) Subclavius
Explanation: The pectoral region consists of pectoralis major, pectoralis minor, and subclavius. Infraspinatus is located on posterior aspect of scapula, part of rotator cuff, not pectoral region. Correct answer: (b) Infraspinatus. Clinical: Important to distinguish anterior chest muscles from posterior scapular muscles.
Guessed Questions for NEET PG
1) Which nerve supplies pectoralis major?
a) Thoracodorsal nerve
b) Medial and lateral pectoral nerves
c) Axillary nerve
d) Suprascapular nerve
Explanation: Pectoralis major is supplied by both medial and lateral pectoral nerves. Correct answer: Medial and lateral pectoral nerves. Clinical: Nerve injury may weaken adduction and medial rotation of arm.
2) Which structure lies deep to pectoralis minor?
a) Brachial plexus cords
b) Cephalic vein
c) Basilic vein
d) Ulnar nerve
Explanation: Pectoralis minor is a landmark for cords of brachial plexus and axillary vessels. Correct answer: Brachial plexus cords. Clinical: Used as a guide in axillary dissections.
3) Subclavius muscle function is?
a) Elevates scapula
b) Depresses clavicle
c) Flexes humerus
d) Extends arm
Explanation: Subclavius depresses and stabilizes the clavicle during shoulder movements. Correct answer: Depresses clavicle. Clinical: Provides protection to subclavian vessels during clavicular fracture.
4) Which artery mainly supplies pectoralis major?
a) Subclavian artery
b) Thoracoacromial artery
c) Radial artery
d) Subscapular artery
Explanation: The thoracoacromial artery, a branch of axillary artery, supplies pectoralis major. Correct answer: Thoracoacromial artery. Clinical: Important during reconstructive flap surgeries.
5) A breast carcinoma infiltrating deep fascia can involve which muscle first?
a) Serratus anterior
b) Pectoralis major
c) Infraspinatus
d) Latissimus dorsi
Explanation: Pectoralis major forms the bed of breast, hence infiltrated in advanced carcinoma. Correct answer: Pectoralis major. Clinical: Explains fixation of breast mass to chest wall.
6) Which muscle is part of rotator cuff but not pectoral region?
a) Subscapularis
b) Infraspinatus
c) Supraspinatus
d) Teres minor
Explanation: Infraspinatus is part of rotator cuff, not pectoral region. Correct answer: Infraspinatus. Clinical: Weakness causes loss of external rotation at shoulder.
7) Injury to medial pectoral nerve causes weakness of?
a) Deltoid
b) Pectoralis minor
c) Latissimus dorsi
d) Teres major
Explanation: Medial pectoral nerve supplies pectoralis minor and part of pectoralis major. Correct answer: Pectoralis minor. Clinical: May weaken scapular protraction.
8) Which lymph nodes lie deep to pectoralis minor?
a) Apical
b) Central
c) Lateral
d) Subscapular
Explanation: Apical group of axillary lymph nodes lie deep to pectoralis minor. Correct answer: Apical. Clinical: Important in breast cancer spread and axillary clearance surgeries.
9) A patient with winging of scapula likely has injury to?
a) Medial pectoral nerve
b) Long thoracic nerve
c) Axillary nerve
d) Suprascapular nerve
Explanation: Winging of scapula occurs due to serratus anterior paralysis from long thoracic nerve injury. Correct answer: Long thoracic nerve. Clinical: Seen in radical mastectomy complications.
10) Which of the following is not a muscle of anterior axillary fold?
a) Pectoralis major
b) Pectoralis minor
c) Subclavius
d) Latissimus dorsi
Explanation: Anterior axillary fold is formed by lower border of pectoralis major. Latissimus dorsi forms posterior fold. Correct answer: Latissimus dorsi. Clinical: Used in surgical identification of axillary folds.
Keyword Definitions
• Bursa – Fluid-filled sac reducing friction between tendon and bone.
• Synovial sheath – Tubular bursa surrounding a tendon for smooth gliding.
• Radial bursa – Synovial sheath enclosing flexor pollicis longus tendon.
• Ulnar bursa – Common flexor sheath for FDP and FDS tendons.
• Flexor pollicis longus (FPL) – Muscle flexing thumb distal phalanx.
• Flexor digitorum profundus (FDP) – Muscle flexing distal phalanges of fingers.
• Flexor digitorum superficialis (FDS) – Muscle flexing middle phalanges.
• Flexor carpi radialis (FCR) – Wrist flexor inserting into 2nd metacarpal.
• Thenar space – Potential space in palm communicating with radial bursa.
• Midpalmar space – Potential space in palm communicating with ulnar bursa.
• Clinical relevance – Infections of synovial sheaths may spread rapidly to palm and forearm.
Chapter: Anatomy / Upper Limb
Topic: Hand and Forearm Structures
Subtopic: Synovial Sheaths and Bursae of Hand
Lead Question – 2013
Radial bursa is the synovial sheath covering the tendon of ?
a) FDS
b) FDP
c) FPL
d) FCR
Explanation: The radial bursa is the synovial sheath of flexor pollicis longus (FPL) tendon. It extends from wrist into the thumb. Correct answer: FPL. Clinical: infection here (tenosynovitis) may spread into the forearm and cause “horseshoe abscess” by communicating with the ulnar bursa.
Guessed Questions for NEET PG
1) Ulnar bursa covers tendons of:
a) FPL
b) FDP and FDS
c) FCR
d) EPL
Explanation: Ulnar bursa is the common flexor sheath enclosing tendons of FDP and FDS to fingers. Correct answer: FDP and FDS. Clinical: infections here can spread into midpalmar space, causing swelling and impaired finger movements.
2) Horseshoe abscess occurs due to communication between:
a) Radial and ulnar bursa
b) Ulnar bursa and carpal tunnel
c) Radial bursa and midpalmar space
d) Thenar and hypothenar spaces
Explanation: Radial bursa of thumb communicates with ulnar bursa of little finger, producing a characteristic “horseshoe-shaped abscess.” Correct answer: Radial and ulnar bursa. Clinical: requires early drainage to prevent spread to forearm.
3) Infection of thumb flexor tendon sheath may spread into:
a) Thenar space
b) Midpalmar space
c) Parona’s space
d) Dorsum of hand
Explanation: FPL tendon sheath infection spreads through radial bursa into Parona’s space (forearm). Correct answer: Parona’s space. Clinical: severe swelling of forearm seen in advanced tenosynovitis.
4) Which tendon passes separately in its own sheath within carpal tunnel?
a) FPL
b) FDP
c) FDS
d) Palmaris longus
Explanation: FPL passes in its own synovial sheath (radial bursa) through the carpal tunnel. Correct answer: FPL. Clinical: inflammation here may cause isolated thumb pain in carpal tunnel syndrome.
5) Parona’s space is located:
a) Between palmar aponeurosis and flexor tendons
b) Between pronator quadratus and flexor tendons
c) In dorsal hand
d) In thenar eminence
Explanation: Parona’s space is between pronator quadratus and flexor tendons in distal forearm. Correct answer: Between pronator quadratus and flexor tendons. Clinical: serves as pathway for spread of infection from radial or ulnar bursa.
6) Which of the following muscles inserts into the distal phalanx of thumb?
a) FPL
b) FDS
c) FDP
d) EPL
Explanation: Flexor pollicis longus (FPL) inserts into the base of distal phalanx of thumb, flexing IP joint. Correct answer: FPL. Clinical: important in pinch grip strength, loss indicates anterior interosseous nerve palsy.
7) Ulnar bursa commonly extends up to which finger?
a) Index
b) Middle
c) Ring
d) Little
Explanation: The ulnar bursa extends into the little finger flexor sheath. Correct answer: Little finger. Clinical: explains why infections of little finger flexor sheath can spread to common flexor sheath and palm.
8) Which structure is enclosed within both radial bursa and carpal tunnel?
a) FCR
b) FPL
c) EPL
d) Lumbricals
Explanation: FPL tendon passes through carpal tunnel inside its radial bursa sheath. Correct answer: FPL. Clinical: tenosynovitis here may mimic carpal tunnel syndrome with isolated thumb symptoms.
9) A 25-year-old presents with swelling of thumb and little finger tendon sheaths with forearm spread. Most likely condition?
a) Thenar abscess
b) Midpalmar abscess
c) Horseshoe abscess
d) Carpal tunnel syndrome
Explanation: Simultaneous infection of radial and ulnar bursae produces characteristic horseshoe abscess. Correct answer: Horseshoe abscess. Clinical: requires surgical drainage through palmar incisions.
10) Which flexor tendon does not pass through the carpal tunnel?
a) FCR
b) FDP
c) FDS
d) FPL
Explanation: Flexor carpi radialis (FCR) passes in its own canal, not inside carpal tunnel. Correct answer: FCR. Clinical: helps distinguish isolated FCR tenosynovitis from carpal tunnel pathologies.
Keyword Definitions
• Cardiac muscle – Specialized involuntary muscle forming the myocardium; responsible for heart contractions.
• Myocardium – Thick middle layer of heart wall made of cardiac muscle; contracts to pump blood.
• Intercalated discs – Specialized junctions connecting cardiac muscle cells; contain desmosomes and gap junctions.
• Gap junctions – Channels allowing electrical coupling between cardiac myocytes for synchronized contraction.
• Nucleus – Cardiac muscle cells typically have one centrally located nucleus; occasionally two.
• Striations – Alternating light and dark bands in cardiac muscle due to organized sarcomeres.
• Sheet arrangement – Cardiac muscle fibers arranged in branching sheets for efficient contraction.
• Spindle-shaped – Characteristic of smooth muscle, not cardiac muscle.
• Clinical relevance – Dysfunction of cardiac muscle leads to arrhythmias, heart failure, or cardiomyopathies.
• Histology – Cardiac muscle cells are short, branched, striated, with intercalated discs and central nuclei.
Chapter: Histology / Muscular System
Topic: Cardiac Muscle
Subtopic: Structure, Histology, and Clinical Relevance
Lead Question – 2013
True about cardiac muscle is?
a) Spindle shaped
b) Large central nucleus
c) No gap junctions
d) Arranged in sheets
Explanation: Cardiac muscle fibers are arranged in branching sheets, enabling coordinated contractions. Correct answer: Arranged in sheets. They are striated, short, branched, with one central nucleus and intercalated discs containing gap junctions. Spindle-shaped is smooth muscle. Clinically, the sheet arrangement ensures synchronized myocardial contraction and efficient blood pumping.
Guessed Questions for NEET PG
1) Cardiac muscle fibers are:
a) Striated
b) Non-striated
c) Voluntary
d) Spindle-shaped
Explanation: Cardiac muscle is striated due to sarcomere organization. Correct answer: Striated. Clinical: Striations are essential for contraction; histological changes indicate cardiomyopathies.
2) Intercalated discs contain:
a) Desmosomes and gap junctions
b) Tight junctions only
c) Hemidesmosomes only
d) None
Explanation: Intercalated discs connect cardiac myocytes via desmosomes for mechanical strength and gap junctions for electrical coupling. Correct answer: Desmosomes and gap junctions. Clinical: defects cause arrhythmias.
3) Nucleus in cardiac myocytes is typically:
a) Single and central
b) Multiple and peripheral
c) Absent
d) Peripheral only
Explanation: Cardiac myocytes usually have a single centrally located nucleus. Correct answer: Single and central. Clinical: nuclear abnormalities may indicate hypertrophy or cardiomyopathy.
4) Cardiac muscle is controlled by:
a) Autonomic nervous system
b) Somatic nervous system
c) Hormones only
d) Voluntary control
Explanation: Cardiac muscle contracts involuntarily under autonomic nervous system regulation. Correct answer: Autonomic nervous system. Clinical: autonomic dysfunction affects heart rate and rhythm.
5) Branching of cardiac fibers allows:
a) Efficient force distribution
b) Limited contraction
c) No contraction
d) Smooth movement only
Explanation: Branching ensures synchronized contraction and effective pumping. Correct answer: Efficient force distribution. Clinical: branching abnormalities reduce cardiac efficiency.
6) Gap junctions allow:
a) Electrical coupling
b) Structural support only
c) Nutrient diffusion only
d) No function
Explanation: Gap junctions permit ion flow between cardiac cells for coordinated contraction. Correct answer: Electrical coupling. Clinical: defective gap junctions cause arrhythmias.
7) Spindle-shaped cells are characteristic of:
a) Smooth muscle
b) Cardiac muscle
c) Skeletal muscle
d) Connective tissue
Explanation: Spindle-shaped cells belong to smooth muscle. Correct answer: Smooth muscle. Cardiac cells are branched. Clinical: smooth muscle pathology affects vessel tone.
8) Clinical significance of cardiac muscle sheets:
a) Coordinated contraction of myocardium
b) Voluntary movements
c) Endocrine secretion
d) Filtration
Explanation: Sheet arrangement allows synchronized myocardial contraction. Correct answer: Coordinated contraction of myocardium. Clinical: disruption leads to inefficient pumping and heart failure.
9) Cardiac muscle striations are due to:
a) Sarcomeres
b) Fibrocartilage
c) Elastic fibers
d) Gap junctions
Explanation: Striations arise from organized sarcomeres with actin and myosin filaments. Correct answer: Sarcomeres. Clinical: sarcomere disruption occurs in cardiomyopathy.
10) Cardiac muscle differs from skeletal muscle in:
a) Branching and intercalated discs
b) Voluntary control
c) Peripheral nuclei
d) Non-striated appearance
Explanation: Cardiac muscle is branched, striated, and has intercalated discs, unlike skeletal muscle which is unbranched, striated, and multinucleated. Correct answer: Branching and intercalated discs. Clinical: these features enable synchronized contractions and resistance to mechanical stress.
Keyword Definitions
• Tetanic contraction: Sustained maximal muscle tension produced by rapid repetitive stimulation.
• Calcium (Ca2+): Intracellular ion that enables cross-bridge cycling by binding to regulatory proteins.
• Summation: Successive twitches adding to increase tension when stimuli arrive before full relaxation.
• Sarcoplasmic reticulum: Organelle storing Ca2+ in muscle fibres and releasing it on stimulation.
• Excitation–contraction coupling: Sequence linking membrane depolarization to Ca2+ release and contraction.
Lead Question - 2012
Tetanic contraction is due to accumulation of ?
a) Na+
b) Ca2+
c) K+
d) Cl-
Explanation:
Tetanic contraction results when repeated stimuli cause intracellular calcium to accumulate because release from the sarcoplasmic reticulum outpaces reuptake; sustained Ca2+ keeps cross-bridges cycling and prevents relaxation. This leads to fused tetanus. Correct answer: Ca2+ (option b). Clinically relevant in high-frequency stimulation and some toxins.
Guessed Questions for NEET PG
1. Which process increases during unfused tetanus compared with single twitch?
a) Ca2+ reuptake
b) Summation of tension
c) Complete relaxation
d) Decreased cross-bridge cycling
Explanation:
Unfused tetanus arises when stimuli occur before full relaxation, so individual twitches summate producing increased mean tension. This summation occurs because residual Ca2+ persists between stimuli, increasing cross-bridge availability; it differs from fused tetanus where relaxation is absent. Correct answer: Summation of tension.
2. A toxin that blocks Ca2+ release would cause?
a) Enhanced tetanus
b) Flaccid paralysis
c) Spastic paralysis
d) Increased muscle tone
Explanation:
Blocking Ca2+ release from the sarcoplasmic reticulum prevents actin-myosin interaction, producing inability to generate force and flaccid paralysis. Without Ca2+, tetanic contraction cannot occur. Clinically, agents that prevent Ca2+ release produce muscle weakness rather than spasm. Correct answer: Flaccid paralysis.
3. Which ionic change primarily terminates contraction between twitches?
a) Increased intracellular Na+
b) Decrease in intracellular Ca2+ via reuptake
c) Increase in intracellular K+
d) Chloride influx
Explanation:
Contraction ends when Ca2+ is actively pumped back into the sarcoplasmic reticulum, lowering cytosolic Ca2+ and allowing regulatory proteins to inhibit cross-bridge formation, enabling relaxation. Efficient reuptake is essential to prevent summation. Correct answer: Decrease in intracellular Ca2+ via reuptake.
4. Which clinical condition produces sustained involuntary tetanic contractions?
a) Hypocalcemia increasing neuronal excitability
b) Hyperkalemia causing flaccidity
c) Botulism blocking ACh release
d) Myasthenia gravis reducing receptor sensitivity
Explanation:
Hypocalcemia lowers threshold for neuronal firing, potentially increasing neurotransmitter release and producing muscle hyperexcitability and tetany. These involuntary sustained contractions differ from tetanic contraction produced experimentally by high-frequency stimulation. Correct answer: Hypocalcemia increasing neuronal excitability.
5. During high-frequency stimulation fused tetanus occurs because?
a) SR reuptake accelerates
b) Cytosolic Ca2+ remains elevated between stimuli
c) Sarcomeres shorten beyond overlap optimum
d) ATP becomes unavailable instantly
Explanation:
Fused tetanus results when stimuli arrive so rapidly that cytosolic Ca2+ remains high continuously, preventing relaxation. Sustained cross-bridge cycling produces maximal, smooth contraction until fatigue or stimulus stops. Correct answer: Cytosolic Ca2+ remains elevated between stimuli.
6. Which mechanism contributes to muscle fatigue during prolonged tetanic contraction?
a) Unlimited ATP supply
b) Accumulation of inorganic phosphate and H+ impairing Ca2+ release and cross-bridges
c) Increased Ca2+ sensitivity of troponin
d) Enhanced SR Ca2+ content
Explanation:
Fatigue during sustained tetanus involves metabolic changes including inorganic phosphate and proton accumulation, which reduce calcium release and cross-bridge function, and impair ATPase activity, leading to declining force despite continued stimulation. Correct answer: Accumulation of inorganic phosphate and H+.
7. Which drug would reduce tetanic contraction by decreasing available Ca2+?
a) Ryanodine receptor agonist opening SR Ca2+ release
b) Dantrolene reducing SR Ca2+ release
c) ACh esterase inhibitor increasing ACh
d) Ca2+ ionophore increasing cytosolic Ca2+
Explanation:
Dantrolene inhibits calcium release from the sarcoplasmic reticulum, reducing intracellular Ca2+ and preventing sustained contractions, clinically used to treat malignant hyperthermia by suppressing excessive muscle contraction. Correct answer: Dantrolene reducing SR Ca2+ release.
8. In skeletal muscle physiology, accumulation of which ion extracellularly tends to reduce excitability rather than cause tetanus?
a) Extracellular K+ accumulation causing depolarization and inactivation of Na+ channels
b) Extracellular Ca2+ accumulation increasing contraction
c) Extracellular Na+ accumulation causing tetanus
d) Extracellular Cl- causing summation
Explanation:
Prolonged activity can raise extracellular K+, depolarizing the membrane and inactivating Na+ channels, reducing excitability and causing weakness, not tetanic contraction; tetanus arises from intracellular Ca2+ accumulation and high-frequency stimulation. Correct answer: Extracellular K+ accumulation causing depolarization and inactivation.
9. Which structural element directly binds Ca2+ to initiate contraction in skeletal muscle?
a) Troponin C on thin filaments
b) Myosin light chain kinase
c) Titin
d) Nebulin
Explanation:
In skeletal muscle, Ca2+ binds to troponin C in the thin filament regulatory complex causing conformational changes that allow myosin–actin interaction and force production; increased Ca2+ concentration from SR release initiates contraction and tetanus under repetitive stimuli. Correct answer: Troponin C.
10. Which experimental manipulation would most directly create fused tetanus in an isolated muscle preparation?
a) Low-frequency single pulses
b) High-frequency electrical stimulation preventing relaxation between pulses
c) Cooling the muscle to stop enzymatic activity
d) Blocking ACh receptors at the neuromuscular junction
Explanation:
Fused tetanus is produced experimentally by applying high-frequency stimuli such that Ca2+ accumulation prevents relaxation between pulses, resulting in sustained maximal tension, an established method to study contractile properties in isolated muscle. Correct answer: High-frequency electrical stimulation.
Chapter: Muscle Physiology
Topic: Length–Tension Relationship
Subtopic: Muscle Fibre Mechanics
Keyword Definitions:
• Optimum length – Muscle fibre length at which maximum tension is generated.
• Equilibrium length – Resting length without external load.
• Initial length – Starting length before contraction begins.
• Sarcomere – Basic contractile unit of a muscle fibre.
• Active tension – Force produced by cross-bridge cycling.
• Passive tension – Force developed when muscle is stretched beyond resting length.
Lead Question - 2012
When the tension in a muscle fibre is maximum, its length is called as ?
a) Equilibrium length
b) Optimum length
c) Initial length
d) None
Explanation:
Maximum muscle tension occurs at the optimum length, where actin–myosin overlap allows maximal cross-bridge formation. Too much shortening or stretching reduces tension. Hence, the correct answer is Optimum length.
Guessed Questions for NEET PG
1. At optimum length, sarcomere overlap is?
a) Complete overlap
b) Partial overlap allowing maximal cross-bridges
c) No overlap
d) Overlap beyond actin filaments
Explanation:
At optimum length, actin and myosin filaments overlap ideally, allowing maximum cross-bridge formation without hindrance. This ensures maximal tension development. Correct answer: Partial overlap allowing maximal cross-bridges.
2. Clinical: A patient with muscle spasm shows reduced force at very short length. Why?
a) Decreased calcium release
b) Overlap of actin filaments
c) Increased ATP consumption
d) Sarcomere hyperextension
Explanation:
At very short sarcomere lengths, actin filaments overlap excessively, blocking cross-bridge binding and reducing force generation. Correct answer: Overlap of actin filaments.
3. Passive tension in muscle arises mainly from?
a) Myosin heads
b) Actin filaments
c) Titin protein
d) Sarcoplasmic reticulum
Explanation:
When a muscle is stretched beyond resting length, passive tension develops due to titin and connective tissue elasticity. Correct answer: Titin protein.
4. Clinical: During physiotherapy, stretching muscles beyond resting length increases?
a) Active tension
b) Passive tension
c) Both active and passive equally
d) No change
Explanation:
Excessive stretching does not increase active tension but raises passive tension through elastic elements like titin. Correct answer: Passive tension.
5. Initial length of cardiac muscle fibres determines?
a) Contractility
b) Preload and stroke volume
c) Afterload
d) Heart rate
Explanation:
In the heart, initial muscle length (preload) sets sarcomere stretch, influencing stroke volume via the Frank–Starling mechanism. Correct answer: Preload and stroke volume.
6. In skeletal muscle, maximum active tension is observed at?
a) 1.6 µm sarcomere length
b) 2.0–2.2 µm sarcomere length
c) 3.5 µm sarcomere length
d) 1.0 µm sarcomere length
Explanation:
Studies show skeletal muscle develops maximal active tension when sarcomere length is 2.0–2.2 µm. Correct answer: 2.0–2.2 µm sarcomere length.
7. Clinical: Reduced cardiac contractility in dilated cardiomyopathy occurs because?
a) Sarcomeres overstretched
b) Increased overlap
c) ATP deficiency
d) Titin rupture
Explanation:
In dilated cardiomyopathy, sarcomeres are excessively stretched, reducing actin–myosin overlap and contractile efficiency. Correct answer: Sarcomeres overstretched.
8. What happens to active tension when muscle is stretched beyond optimum length?
a) Increases further
b) Decreases
c) Remains same
d) Shifts to passive
Explanation:
Beyond optimum length, overlap reduces, leading to fewer cross-bridges and decreased active tension, while passive tension increases. Correct answer: Decreases.
9. In isometric contraction at optimum length?
a) Tension is maximum
b) Length shortens maximally
c) Passive tension decreases
d) ATP consumption is minimal
Explanation:
During isometric contraction, muscle length is fixed but tension peaks at optimum sarcomere length. Correct answer: Tension is maximum.
10. Clinical: After immobilization, muscles lose optimum length efficiency due to?
a) Reduced sarcomere number
b) Increased titin length
c) Hyperplasia of fibres
d) Myelin degeneration
Explanation:
Immobilization leads to sarcomere loss, altering optimum length and reducing force output. Correct answer: Reduced sarcomere number.
Chapter: Physiology
Topic: Muscle Physiology
Subtopic: Smooth vs Skeletal Muscle
Keyword Definitions:
• Smooth muscle – Involuntary muscle controlled by autonomic nervous system and hormones.
• Skeletal muscle – Voluntary muscle controlled by somatic nervous system.
• Calcium ions – Required for contraction in both smooth and skeletal muscle, though mechanism differs.
• Troponin – Regulatory protein present in skeletal but absent in smooth muscle.
• Myosin – Thick filament protein essential for contraction in all muscles.
Lead Question - 2012
Smooth muscle physiology different from skeletal muscle
a) K⁺ requires for contraction
b) Ca²⁺ required for contraction
c) Troponin is absent
d) Myosin is required for contraction
Explanation:
Smooth muscle differs from skeletal muscle mainly due to the absence of troponin. Instead, smooth muscle contraction is regulated by calmodulin–myosin light chain kinase pathway. Calcium still plays a key role, but the regulatory protein troponin is absent. Correct answer: Troponin is absent.
Guessed Questions for NEET PG
1. Smooth muscle contraction is regulated by?
a) Troponin
b) Calmodulin
c) Tropomyosin
d) Titin
Explanation:
Unlike skeletal muscle, smooth muscle lacks troponin. Instead, contraction is controlled by calcium–calmodulin complex which activates myosin light chain kinase. Correct answer: Calmodulin.
2. Which protein is absent in smooth muscle fibers?
a) Actin
b) Troponin
c) Myosin
d) Tropomyosin
Explanation:
Actin, myosin, and tropomyosin are present in smooth muscle. Troponin, however, is absent, which differentiates it from skeletal muscle. Correct answer: Troponin.
3. Calcium in smooth muscle binds to?
a) Troponin C
b) Calmodulin
c) Tropomyosin
d) Actinin
Explanation:
In skeletal muscle, calcium binds troponin C, but in smooth muscle it binds to calmodulin, initiating contraction via MLCK. Correct answer: Calmodulin.
4. Patient with asthma receives bronchodilator. Relaxation of airway smooth muscle occurs by?
a) Increased cAMP
b) Increased IP₃
c) Increased calcium
d) Decreased cAMP
Explanation:
β₂ agonists increase cAMP, which inhibits MLCK, leading to relaxation of smooth muscle in bronchi. Correct answer: Increased cAMP.
5. Dense bodies in smooth muscle are functionally similar to?
a) T-tubules
b) Z-lines
c) Sarcoplasmic reticulum
d) Myosin heads
Explanation:
Dense bodies in smooth muscle anchor thin filaments, serving a role similar to Z-lines in skeletal muscle. Correct answer: Z-lines.
6. Which of the following best describes latch phenomenon in smooth muscle?
a) Sustained contraction with high ATP
b) Sustained contraction with low ATP
c) Rapid relaxation
d) Skeletal-type tetany
Explanation:
Latch phenomenon allows smooth muscle to maintain tension for long periods with minimal ATP use. Correct answer: Sustained contraction with low ATP.
7. In smooth muscle, myosin light chain kinase (MLCK) is activated by?
a) Troponin C
b) Calcium–calmodulin
c) Tropomyosin
d) ATP alone
Explanation:
Calcium binds calmodulin which activates MLCK, phosphorylating myosin light chains to initiate contraction. Correct answer: Calcium–calmodulin.
8. Clinical: A hypertensive patient takes a calcium channel blocker. Effect on vascular smooth muscle?
a) Increased contraction
b) Decreased contraction
c) Increased MLCK activity
d) Increased calcium influx
Explanation:
Calcium channel blockers reduce calcium influx into smooth muscle, lowering contraction and relaxing vessels. Correct answer: Decreased contraction.
9. Skeletal and smooth muscle differ because skeletal muscle contraction is?
a) Involuntary and calmodulin-mediated
b) Voluntary and troponin-mediated
c) Involuntary and troponin-mediated
d) Voluntary and calmodulin-mediated
Explanation:
Skeletal muscle is voluntary and regulated by troponin–tropomyosin complex, unlike smooth muscle. Correct answer: Voluntary and troponin-mediated.
10. Which structural arrangement is absent in smooth muscle compared to skeletal?
a) Sarcomere
b) Myosin
c) Actin
d) Intermediate filaments
Explanation:
Smooth muscle lacks sarcomere arrangement, hence appears non-striated. Myosin, actin, and intermediate filaments are present. Correct answer: Sarcomere.
Chapter: Lower Limb Anatomy
Topic: Hip Joint & Thigh Muscles
Subtopic: Hip Flexors — Actions and Clinical Relevance
Keyword Definitions
Psoas major — Primary hip flexor arising from lumbar vertebrae; strong hip flexion and trunk flexion contribution.
Iliacus — Joins psoas major to form iliopsoas, major hip flexor inserting on lesser trochanter.
Iliopsoas — Combined tendon of psoas major and iliacus; principal flexor of hip.
Biceps femoris — Hamstring muscle; knee flexor and hip extensor (long head) — not primary hip flexor.
Gluteus maximus — Principal hip extensor and external rotator; active in rising from sitting.
Tensor fasciae latae (TFL) — Assists hip flexion, abduction, and medial rotation; acts via IT band.
Hip flexion — Movement decreasing angle between thigh and trunk; important for walking, climbing stairs.
Femoral nerve — Supplies iliacus and provides motor to quadriceps; psoas major supplied by lumbar plexus.
Hip pathology — Iliopsoas bursitis or tendonitis causes anterior hip pain aggravated by flexion.
Clinical test — Straight leg raise and resisted hip flexion assess iliopsoas function.
Lead Question - 2012
Muscle causing flexion of hip ?
a) Biceps femoris
b) Psoas major
c) Gluteus maximus
d) TFL
Explanation: The psoas major, together with iliacus (forming iliopsoas), is the principal hip flexor producing powerful flexion at the hip and contributing to trunk flexion. Biceps femoris and gluteus maximus are extensors; TFL assists flexion but is not the primary flexor. Answer: b) Psoas major.
1. Primary nerve supply to psoas major is from:
a) Femoral nerve
b) Lumbar plexus (L1–L3)
c) Sciatic nerve
d) Superior gluteal nerve
Explanation: Psoas major receives direct branches from the anterior rami of lumbar spinal nerves (L1–L3) constituting part of the lumbar plexus. Femoral nerve supplies iliacus; sciatic and superior gluteal do not innervate psoas. Clinically lumbar radiculopathy affects hip flexion. Answer: b) Lumbar plexus (L1–L3).
2. Iliopsoas tendonitis typically causes pain at:
a) Posterior thigh
b) Anterior groin/hip
c) Lateral knee
d) Medial ankle
Explanation: Iliopsoas tendonitis presents with anterior groin or hip pain aggravated by hip flexion, resisted straight leg raise, and climbing stairs. It may mimic intra-articular hip pathology; ultrasound or MRI helps confirm tendon inflammation. Answer: b) Anterior groin/hip.
3. Which muscle is a synergist to psoas major in hip flexion?
a) Gluteus maximus
b) Rectus femoris
c) Adductor magnus
d) Piriformis
Explanation: Rectus femoris (part of quadriceps) crosses the hip and assists in hip flexion, acting as a synergist to iliopsoas during activities requiring powerful hip flexion like kicking. Gluteus maximus is an extensor. Answer: b) Rectus femoris.
4. A patient cannot flex hip against resistance but can on gravity-eliminated plane. This suggests:
a) Complete nerve transection
b) Pain inhibition or partial weakness
c) Intact motor function
d) Labral tear
Explanation: Inability to perform resisted hip flexion with preserved movement on gravity-eliminated testing suggests pain inhibition or partial weakness (Grade 3/5), not complete denervation. Further neuro exam and imaging are warranted. Answer: b) Pain inhibition or partial weakness.
5. InTrendelenburg gait is due to weakness of:
a) Hip flexors (psoas)
b) Hip abductors (gluteus medius/minimus)
c) Knee extensors (quadriceps)
d) Ankle dorsiflexors
Explanation: Trendelenburg gait arises from weak hip abductors (gluteus medius/minimus) causing pelvic drop on contralateral side during stance. Psoas weakness causes hip flexion issues but not Trendelenburg. Examination differentiates abductor from flexor pathology. Answer: b) Hip abductors (gluteus medius/minimus).
6. Which motion is produced when iliopsoas acts bilaterally on fixed femur?
a) Trunk extension
b) Trunk flexion (sit-up)
c) Lateral rotation of femur
d) Hip abduction
Explanation: With femur fixed, bilateral contraction of iliopsoas flexes the trunk at the hip (raises the torso), assisting in sit-up movements. It also stabilizes lumbar spine during posture. Dysfunction impairs rising from supine. Answer: b) Trunk flexion (sit-up).
7. Which muscle primarily extends the hip and opposes psoas major?
a) Iliacus
b) Gluteus maximus
c) Sartorius
d) Pectineus
Explanation: Gluteus maximus is the main hip extensor providing power for rising, climbing, and sprinting, functionally opposing the flexion produced by iliopsoas. Injury to gluteus maximus alters gait and seated-to-standing mechanics. Answer: b) Gluteus maximus.
8. Tight psoas major may cause which postural change?
a) Increased lumbar lordosis
b) Flattened lumbar curve
c) Thoracic kyphosis reduction
d) Knee hyperextension
Explanation: A tight psoas pulls lumbar spine into increased lordosis and anterior pelvic tilt, contributing to low back pain and altered gait mechanics. Stretching and posture correction are part of management. Answer: a) Increased lumbar lordosis.
9. Hip flexion power is most compromised by lesion of which structure?
a) Femoral nerve root (L2–L4)
b) Sciatic nerve
c) Obturator nerve
d) Superior gluteal nerve
Explanation: The femoral nerve innervates iliacus and rectus femoris; lesion reduces hip flexion strength. Psoas major is from lumbar plexus; combined lesions of L2–L4 significantly impair hip flexion. Sciatic/obturator/gluteal affect other functions. Answer: a) Femoral nerve root (L2–L4).
10. Clinical test for iliopsoas strength: patient performs?
a) Hip abduction against resistance
b) Resisted hip flexion in sitting or supine
c) Heel raise
d) Knee flexion
Explanation: Resisted hip flexion (patient attempts to lift thigh against resistance) assesses iliopsoas/rectus femoris. Positive pain or weakness suggests tendonitis, nerve lesion, or muscular tear and guides further imaging or EMG. Answer: b) Resisted hip flexion in sitting or supine.
Chapter: Head & Neck Anatomy
Topic: Facial Muscles
Subtopic: Risorius — anatomy, function, and clinical relevance
Keyword Definitions
Risorius — thin superficial facial muscle that retracts the angle of the mouth producing a smile or grimace.
Facial expression muscles — group of muscles innervated by the facial nerve (CN VII) that move the skin of the face.
Mastication muscles — muscles of chewing (masseter, temporalis, pterygoids) innervated by V3, not risorius.
Buccinator — deep cheek muscle that compresses the cheek and assists in mastication; distinct from risorius.
Facial nerve (CN VII) — motor nerve supplying muscles of facial expression including risorius; vulnerable in parotid surgery.
Parotid gland/plexus — risorius lies superficially near parotid region; surgical injury may affect branches of CN VII causing asymmetry.
Botulinum toxin — used cosmetically/therapeutically on facial muscles; improper injection into risorius can alter smile.
Deglutition — swallowing; risorius is not a primary deglutition muscle.
Orbicularis oris — sphincter muscle of the mouth working with risorius for lip movement.
Clinical sign — loss of risorius function causes inability to retract mouth corner, mild asymmetrical smile.
Lead Question - 2012
Risorius is a muscle of ?
a) Mastication
b) Deglutition
c) Facial expression
d) Eye movement
Explanation: Risorius is a superficial muscle of facial expression that retracts the angle of the mouth laterally, contributing to smiling or grimacing. It is innervated by branches of the facial nerve (CN VII) and not involved in mastication, swallowing, or eye movement. Answer: c) Facial expression.
1. Which nerve supplies the risorius muscle?
a) Mandibular division of trigeminal (V3)
b) Facial nerve (CN VII)
c) Hypoglossal nerve (XII)
d) Glossopharyngeal nerve (IX)
Explanation: The risorius receives motor innervation from the facial nerve (CN VII) via its buccal or zygomatic branches. Damage to these branches (eg. parotid surgery) produces weakness of mouth corner retraction and a flattened or asymmetrical smile. Answer: b) Facial nerve (CN VII).
2. Primary action of risorius is to:
a) Elevate mandible
b) Protrude tongue
c) Retract angle of mouth laterally
d) Close eyelids
Explanation: Risorius retracts the angle of the mouth laterally, creating a grin or stretched smile. It acts with levator and depressor muscles to modulate facial expression. It does not elevate the mandible, move the tongue, or close eyelids. Answer: c) Retract angle of mouth laterally.
3. Injury to the facial nerve branch supplying risorius produces which clinical sign?
a) Difficulty in chewing
b) Loss of forehead wrinkling
c) Asymmetrical smile with inability to retract mouth corner
d) Diplopia
Explanation: Paralysis of the branch to risorius causes inability to retract the mouth corner on the affected side, producing an asymmetrical smile. Chewing is mainly V3, forehead wrinkling involves temporal branch, and diplopia relates to ocular muscles. Answer: c) Asymmetrical smile with inability to retract mouth corner.
4. During parotid surgery, which precaution least helps preserve risorius function?
a) Identify and protect facial nerve branches
b) Minimize traction on superficial fascia
c) Avoid deep incisions through masseter
d) Only operate via transoral approach crossing buccinator
Explanation: Preserving facial nerve branches and superficial fascia protects risorius. A transoral approach through buccinator (d) risks injuring branches and the duct, making it least protective. Avoiding deep masseter incisions is sensible. Answer: d) Only operate via transoral approach crossing buccinator.
5. Which muscle lies deep to risorius and assists cheek flattening during mastication?
a) Buccinator
b) Masseter
c) Platysma
d) Levator labii superioris
Explanation: Buccinator lies deep to risorius and compresses the cheek to keep food between teeth, aiding mastication. Masseter is a primary masticator on lateral face. Risorius superficially retracts mouth corner and does not assist primary chewing. Answer: a) Buccinator.
6. Cosmetic injection of botulinum toxin into risorius is used to treat:
a) Jaw claudication
b) Gummy smile due to hyperactive smiling muscles
c) Orbital cellulitis
d) Vocal cord paralysis
Explanation: Botulinum toxin into risorius and adjacent smile muscles can reduce hyperactive lateral mouth retraction contributing to a gummy or asymmetrical smile. Careful dosing prevents excessive weakness and smile distortion. It is not used for orbital cellulitis or vocal cord issues. Answer: b) Gummy smile due to hyperactive smiling muscles.
7. Anatomical variation: risorius commonly inserts into:
a) Skin at angle of mouth
b) Mandibular ramus
c) Zygomatic arch
d) Hyoid bone
Explanation: Risorius inserts into the skin at the angle of mouth (modiolus area), pulling it laterally. It does not attach to bony structures like the mandible, zygoma, or hyoid. Variations in origin exist but insertion is cutaneous. Answer: a) Skin at angle of mouth.
8. Which clinical test assesses risorius function specifically?
a) Puff cheeks against closed lips
b) Ask patient to show teeth and retract mouth corners laterally
c) Test tongue protrusion
d) Assess shoulder shrug
Explanation: Asking the patient to smile broadly and retract mouth corners tests risorius among other smile muscles; inability to retract the affected corner suggests weakness. Puffing cheeks tests buccinator and orbicularis, tongue relates to XII, shoulder shrug to accessory nerve. Answer: b) Ask patient to show teeth and retract mouth corners laterally.
9. In facial nerve palsy localized to the buccal branches, which action is most compromised?
a) Eye closure
b) Nasolabial fold flattening and mouth corner retraction
c) Head rotation
d) Tongue movement
Explanation: Buccal branches supply muscles of the midface including risorius, buccinator, and orbicularis oris; palsy flattens nasolabial fold and impairs mouth corner retraction and labial competence. Eye closure is temporal/zygomatic, head rotation accessory. Answer: b) Nasolabial fold flattening and mouth corner retraction.
10. Which statement about risorius is FALSE?
a) It is a muscle of facial expression
b) It aids chewing by elevating mandible
c) It is innervated by CN VII
d) It inserts into skin at mouth angle
Explanation: Risorius does not aid mastication or elevate the mandible; that is the role of masseter and temporalis. Risorius retracts mouth angle, is innervated by facial nerve, and inserts into the skin at the angle of the mouth. Answer: b) It aids chewing by elevating mandible (FALSE).
Chapter: Pharynx
Topic: Pharyngeal Muscles
Subtopic: Passavant's Ridge
Keyword Definitions:
Passavant's Ridge: A mucosal ridge on the posterior pharyngeal wall during swallowing.
Superior Constrictor: Muscle forming posterior pharyngeal wall, contributes to ridge formation.
Palatopharyngeus: Elevates pharynx and contributes fibers to ridge.
Palatoglossus: Muscle connecting tongue to palate, not involved in ridge.
Inferior Constrictor: Pharyngeal constrictor muscle, not part of ridge formation.
Soft Palate: Muscular fold that closes nasopharynx during swallowing.
Lead Question – 2012
Passavant ridge ?
a) Superior constrictor and palatopharyngeus
b) Inferior constrictor and palatopharyngeus
c) Superior constrictor and palatoglossus
d) Inferior constrictor and palatoglossus
Explanation: Passavant’s ridge is a mucosal prominence formed by contraction of the superior constrictor and palatopharyngeus during swallowing. It helps close the nasopharyngeal isthmus against the soft palate, preventing nasal regurgitation. Correct answer: a) Superior constrictor and palatopharyngeus.
Question 2. A patient presents with nasal regurgitation of liquids. Dysfunction of which structure is most likely?
a) Palatoglossus
b) Passavant’s ridge
c) Cricopharyngeus
d) Stylopharyngeus
Explanation: Failure of Passavant’s ridge to form properly prevents closure of nasopharynx, causing nasal regurgitation. This occurs with superior constrictor or palatopharyngeus weakness. Correct answer: b) Passavant’s ridge.
Question 3. Which muscle is primarily responsible for preventing food from entering the nasopharynx?
a) Tensor veli palatini
b) Levator veli palatini
c) Palatopharyngeus
d) Stylopharyngeus
Explanation: Levator veli palatini elevates the soft palate, sealing the nasopharynx against Passavant’s ridge. This prevents regurgitation during swallowing. Correct answer: b) Levator veli palatini.
Question 4. A lesion of glossopharyngeal nerve will most likely cause?
a) Absent gag reflex
b) Nasal regurgitation
c) Hoarseness
d) Shoulder droop
Explanation: Glossopharyngeal nerve supplies sensory input for gag reflex. Its lesion abolishes gag reflex while motor defects are due to vagus/accessory nerve. Correct answer: a) Absent gag reflex.
Question 5. Inadequate closure of the nasopharynx during swallowing is due to paralysis of?
a) Palatopharyngeus
b) Stylopharyngeus
c) Masseter
d) Cricopharyngeus
Explanation: Palatopharyngeus elevates pharynx and forms Passavant’s ridge. Its dysfunction leads to failure of nasopharyngeal closure. Correct answer: a) Palatopharyngeus.
Question 6. A child with cleft palate develops hypernasal speech. The main cause is?
a) Weak palatal closure
b) Weak tongue movement
c) Laryngeal stenosis
d) Weak jaw closure
Explanation: Hypernasality results from failure of palatal muscles and Passavant’s ridge to close the nasopharyngeal opening during speech. Correct answer: a) Weak palatal closure.
Question 7. Which nerve mediates motor supply to palatal muscles except tensor veli palatini?
a) Glossopharyngeal
b) Vagus via cranial accessory
c) Trigeminal mandibular
d) Hypoglossal
Explanation: All palatal muscles except tensor veli palatini are supplied by vagus through cranial part of accessory nerve. Correct answer: b) Vagus via cranial accessory.
Question 8. Which muscle is supplied by mandibular nerve among palatal muscles?
a) Levator veli palatini
b) Tensor veli palatini
c) Palatopharyngeus
d) Palatoglossus
Explanation: Tensor veli palatini is the only palatal muscle supplied by mandibular division of trigeminal nerve. Correct answer: b) Tensor veli palatini.
Question 9. A 40-year-old develops difficulty swallowing with nasal escape of food. Which nerve is most likely affected?
a) Vagus
b) Hypoglossal
c) Facial
d) Glossopharyngeal
Explanation: Vagus nerve supplies motor fibers to palatal muscles (via cranial accessory). Its lesion causes palatal paralysis and nasal regurgitation. Correct answer: a) Vagus.
Question 10. Which muscle contributes to Passavant’s ridge along with superior constrictor?
a) Palatopharyngeus
b) Palatoglossus
c) Stylopharyngeus
d) Tensor veli palatini
Explanation: Passavant’s ridge is formed by palatopharyngeus fibers joining superior constrictor contraction. Correct answer: a) Palatopharyngeus.
Question 11. A surgeon notes poor closure of nasopharynx after adenoidectomy. Which structure is compromised?
a) Passavant’s ridge
b) Cricopharyngeus
c) Stylopharyngeus
d) Tensor veli tympani
Explanation: If Passavant’s ridge or its contributing muscles are injured, nasopharyngeal closure fails, leading to regurgitation or hypernasal speech. Correct answer: a) Passavant’s ridge.
Triangle of Auscultation: A small triangular area on the back where the thoracic wall is thin, allowing clear lung auscultation.
Trapezius: Large superficial muscle of the back responsible for moving the scapula and supporting arm movements.
Latissimus Dorsi: Broad, flat muscle covering lower back; involved in arm adduction, extension, and internal rotation.
Rhomboid Major: Muscle connecting spine to scapula, retracting and stabilizing the scapula.
Scapula: Shoulder blade bone, forms part of the shoulder girdle; not a muscular boundary.
Chapter: Anatomy
Topic: Back Muscles
Subtopic: Triangle of Auscultation
Lead Question 2012: All form boundaries of triangle of auscultation except:
a) Trapezius
b) Latissmusdorsi
c) Scapula
d) Rhomboid major
Answer: c) Scapula
Explanation: The triangle of auscultation is bordered by the **trapezius** medially, **rhomboid major** laterally, and **latissimus dorsi** inferiorly. The scapula is a bone overlying the area, not a muscular boundary. Clinically, this triangle is important because placing a stethoscope here allows better auscultation of lung sounds due to reduced muscular thickness.
1. Which muscle forms the medial border of the triangle of auscultation?
a) Rhomboid major
b) Trapezius
c) Latissimus dorsi
d) Serratus anterior
Answer: b) Trapezius
Explanation: The medial border of the triangle of auscultation is formed by the **trapezius**. This muscle extends from the occipital bone to the lower thoracic vertebrae, covering part of the back. Its lateral border is formed by rhomboid major, and inferiorly by latissimus dorsi.
2. The inferior boundary of the triangle of auscultation is formed by:
a) Latissimus dorsi
b) Trapezius
c) Rhomboid minor
d) Erector spinae
Answer: a) Latissimus dorsi
Explanation: The **latissimus dorsi** forms the inferior boundary of the triangle. It is a broad, flat muscle involved in arm adduction, extension, and medial rotation. Its position makes the area thin and suitable for auscultating lung sounds clinically.
3. Lateral border of the triangle of auscultation is formed by:
a) Trapezius
b) Rhomboid major
c) Serratus posterior superior
d) Levator scapulae
Answer: b) Rhomboid major
Explanation: The **rhomboid major** forms the lateral border of the triangle of auscultation. It retracts and stabilizes the scapula, and its thin overlay in this area allows better lung auscultation. Trapezius is medial, latissimus dorsi is inferior.
4. Clinically, why is the triangle of auscultation important?
a) Muscle injection site
b) Better lung auscultation
c) Site for nerve block
d) Bone palpation
Answer: b) Better lung auscultation
Explanation: The triangle of auscultation is a clinically significant area because muscular thickness is minimal, allowing stethoscopes to detect lung sounds clearly. Physicians often ask patients to fold arms over chest to enlarge the triangle for auscultation.
5. Which posture enlarges the triangle of auscultation for examination?
a) Arms at sides
b) Hands behind head
c) Hands folded across chest
d) Forward bending
Answer: c) Hands folded across chest
Explanation: Folding the arms across the chest rotates the scapula laterally, widening the triangle of auscultation. This allows easier placement of the stethoscope for clear lung sound assessment. Forward bending or arms behind head do not effectively enlarge the area.
6. Which nerve is closely related to the triangle of auscultation?
a) Long thoracic nerve
b) Thoracodorsal nerve
c) Spinal accessory nerve
d) Axillary nerve
Answer: a) Long thoracic nerve
Explanation: The **long thoracic nerve** runs along the lateral chest wall near the triangle, innervating serratus anterior. Injury can cause winged scapula. Awareness of nearby nerves is important during clinical procedures in the area.
7. Which imaging modality can show muscles forming the triangle of auscultation?
a) X-ray
b) MRI
c) Ultrasound
d) CT scan
Answer: b) MRI
Explanation: MRI can clearly visualize soft tissues including trapezius, rhomboid major, and latissimus dorsi, demonstrating the triangle of auscultation. Ultrasound can also be used, but X-ray shows only bones. CT scan is less commonly used for muscle visualization.
8. Which condition can make auscultation through this triangle more difficult?
a) Muscle hypertrophy
b) Muscle atrophy
c) Thin body habitus
d) Forward flexion
Answer: a) Muscle hypertrophy
Explanation: Hypertrophy of trapezius, rhomboid, or latissimus dorsi increases muscle thickness, making lung auscultation more difficult. Thin body habitus enhances auscultation. Clinical positioning can compensate partially, but hypertrophy reduces the acoustic window.
9. Which muscle is superficial and must be displaced to access the triangle?
a) Trapezius
b) Rhomboid minor
c) Levator scapulae
d) Serratus anterior
Answer: a) Trapezius
Explanation: The trapezius is superficial and overlies part of the triangle. During clinical examination, scapular rotation moves the trapezius laterally, allowing access to the triangle of auscultation. Deep muscles like rhomboid minor remain under trapezius.
10. Which movement of the scapula enlarges the triangle laterally?
a) Elevation
b) Retraction
c) Protraction
d) Depression
Answer: c) Protraction
Explanation: Lateral protraction of the scapula moves it away from the vertebral column, widening the triangle of auscultation. This facilitates auscultation. Retraction or elevation reduces the area, making examination more difficult.
Keyword Definitions:
Pelvic Stability: Ability to maintain pelvis alignment during walking or standing to prevent tilting.
Gluteus Medius & Minimus: Hip abductor muscles crucial for stabilizing pelvis during gait.
Trendelenburg Gait: Abnormal gait caused by weakness of gluteus medius/minimus.
Hip Abduction: Movement of the leg away from the midline, important in walking stability.
Gait Cycle: The sequence of motions during walking, divided into stance and swing phases.
Quadriceps: Group of thigh muscles involved in knee extension and stabilization during walking.
Adductors: Muscles bringing the thigh towards the body's midline.
Pelvic Tilt: Movement of the pelvis in the sagittal or frontal plane affecting gait mechanics.
Chapter: Locomotion and Movement
Topic: Gait and Muscle Actions
Sub-topic: Pelvic Stability During Walking
Lead Question (2012):
42. In walking, gravity tends to tilt pelvis and trunk to the unsupported side, major factor in preventing this unwanted movement is?
a) Adductor muscles
b) Quadriceps
c) Gluteus maximus
d) Gluteus medius and minimus
Explanation: The gluteus medius and minimus are primary hip abductors that stabilize the pelvis during the stance phase of gait. When one leg is lifted, these muscles on the opposite side contract to prevent pelvic drop. Weakness results in Trendelenburg gait. Adductors, quadriceps, and gluteus maximus have other roles in walking but do not provide primary pelvic stability in this context. Correct Answer: d) Gluteus medius and minimus
1. A patient presents with a waddling gait and difficulty maintaining pelvic level while walking. Which muscle weakness is most likely responsible?
a) Gluteus medius
b) Rectus femoris
c) Sartorius
d) Iliopsoas
Explanation: Waddling gait and pelvic drop point to gluteus medius weakness. This muscle functions as a pelvic stabilizer during the stance phase. Rectus femoris is mainly for knee extension and hip flexion, sartorius assists in flexion/abduction, and iliopsoas is the primary hip flexor. Correct Answer: a) Gluteus medius
2. Trendelenburg test is used to assess the integrity of which nerve?
a) Femoral nerve
b) Obturator nerve
c) Superior gluteal nerve
d) Inferior gluteal nerve
Explanation: The superior gluteal nerve innervates the gluteus medius and minimus. Damage causes positive Trendelenburg sign due to pelvic drop on the contralateral side during single-leg stance. The femoral nerve supplies anterior thigh muscles, obturator nerve supplies adductors, and inferior gluteal nerve supplies gluteus maximus. Correct Answer: c) Superior gluteal nerve
3. During walking, in which phase do the gluteus medius and minimus contract most strongly?
a) Swing phase
b) Stance phase
c) Toe-off phase
d) Double support phase
Explanation: Hip abductors like gluteus medius and minimus are most active during the stance phase when the opposite leg is in the swing phase. This activity prevents pelvic drop toward the unsupported side. In the swing phase, these muscles are less active. Correct Answer: b) Stance phase
4. A clinical student notes that a patient’s pelvis drops to the left during right leg stance. This indicates weakness in:
a) Left gluteus medius
b) Right gluteus medius
c) Right gluteus maximus
d) Left gluteus maximus
Explanation: Pelvic drop occurs on the side opposite to the weak muscle. If pelvis drops to the left, the right gluteus medius is weak. This is because the right hip abductors should hold the pelvis level when the right leg bears weight. Correct Answer: b) Right gluteus medius
5. Which of the following muscles assists the gluteus medius in pelvic stabilization?
a) Tensor fasciae latae
b) Biceps femoris
c) Semitendinosus
d) Gastrocnemius
Explanation: The tensor fasciae latae assists the gluteus medius in hip abduction and pelvic stabilization during gait. Biceps femoris and semitendinosus are hamstring muscles, while gastrocnemius is a calf muscle. Correct Answer: a) Tensor fasciae latae
6. In a patient with a superior gluteal nerve injury, which gait abnormality is most expected?
a) Steppage gait
b) Antalgic gait
c) Trendelenburg gait
d) Ataxic gait
Explanation: Superior gluteal nerve injury weakens the gluteus medius and minimus, leading to Trendelenburg gait — characterized by pelvic drop on the unsupported side during walking. Steppage gait is seen in foot drop, antalgic gait in pain, and ataxic gait in cerebellar disorders. Correct Answer: c) Trendelenburg gait
7. Which muscle is primarily responsible for preventing excessive pelvic tilt in the coronal plane during single-leg stance?
a) Piriformis
b) Gluteus maximus
c) Gluteus medius
d) Iliacus
Explanation: The gluteus medius is the primary stabilizer preventing excessive pelvic tilt in the coronal plane during single-leg stance. Piriformis assists in lateral rotation, gluteus maximus in hip extension, and iliacus in hip flexion. Correct Answer: c) Gluteus medius
8. A 45-year-old patient has difficulty climbing stairs after hip surgery. Which muscle weakness is most likely if pelvic stability is also compromised?
a) Quadriceps femoris
b) Gluteus medius
c) Adductor magnus
d) Sartorius
Explanation: Gluteus medius weakness affects both stair climbing and pelvic stability. Quadriceps mainly extend the knee, adductor magnus brings the leg toward midline, and sartorius flexes the hip and knee but does not stabilize pelvis effectively. Correct Answer: b) Gluteus medius
9. Which muscle group counteracts gravity’s tendency to tilt the pelvis during the gait cycle?
a) Hip abductors
b) Hip adductors
c) Knee extensors
d) Ankle plantarflexors
Explanation: Hip abductors (mainly gluteus medius and minimus) counteract gravity’s pull, keeping pelvis level during stance phase. Adductors, knee extensors, and ankle plantarflexors serve other roles in gait mechanics. Correct Answer: a) Hip abductors
10. A physiotherapist is retraining a stroke patient’s gait. To strengthen pelvic stability, which exercise is most appropriate?
a) Side-lying hip abduction
b) Leg press
c) Calf raises
d) Hamstring curls
Explanation: Side-lying hip abduction specifically targets the gluteus medius and minimus, enhancing pelvic stability during walking. Leg press works mainly quadriceps, calf raises target gastrocnemius/soleus, and hamstring curls strengthen posterior thigh muscles. Correct Answer: a) Side-lying hip abduction
Keywords & Definitions:
Popliteus muscle: A small muscle located at the back of the knee joint, playing a role in unlocking the knee from a fully extended position.
Knee joint: A hinge-type synovial joint connecting the femur, tibia, and patella, allowing flexion and extension.
Intracapsular origin: A muscle origin point located within the fibrous capsule of a joint.
Tibial nerve: A branch of the sciatic nerve supplying motor and sensory innervation to parts of the leg and foot.
Knee locking: A mechanism that stabilizes the knee in extension by medial rotation of the femur on the tibia.
Flexor muscles of knee: Muscles that bend the knee joint, reducing the angle between the thigh and leg.
Posterior cruciate ligament: A ligament inside the knee joint that prevents posterior displacement of the tibia.
Meniscus: C-shaped fibrocartilage structures that provide cushioning and stability in the knee joint.
Lateral rotation of femur: Outward movement of the femur around its longitudinal axis, important in knee biomechanics.
Bursitis: Inflammation of the bursa, a small fluid-filled sac reducing friction between tissues.
Chapter: Locomotor System – Lower Limb Anatomy
Topic: Muscles of the Leg
Sub-topic: Popliteus Muscle Anatomy and Function
Lead Question – NEET PG 2012:
True about popliteus are all except?
a) Flexor of knee
b) Intracapsular origin
c) Supplied by tibial nerve
d) Causes locking of knee
Explanation:
The popliteus muscle is a flexor of the knee, originates intracapsularly from the lateral femoral condyle, and is supplied by the tibial nerve. However, it causes unlocking of the knee by laterally rotating the femur on the tibia during initiation of flexion. The locking function is opposite to its role. Correct answer: d) Causes locking of knee.
Guess Question 1:
Which nerve injury can lead to weakness in unlocking the knee?
a) Common peroneal nerve
b) Tibial nerve
c) Femoral nerve
d) Obturator nerve
Explanation: The tibial nerve innervates the popliteus muscle, crucial for unlocking the knee. Injury to this nerve impairs flexion initiation from a fully extended position, causing difficulty in walking downstairs or on slopes. Correct answer: b) Tibial nerve.
Guess Question 2:
The popliteus muscle primarily acts by:
a) Medially rotating the tibia
b) Laterally rotating the femur
c) Abducting the tibia
d) Extending the knee
Explanation: The popliteus muscle unlocks the knee by laterally rotating the femur on the tibia when the foot is fixed, or medially rotating the tibia when the foot is free. This breaks the 'screw-home' mechanism of knee locking. Correct answer: b) Laterally rotating the femur.
Guess Question 3:
Which structure is located anterior to the popliteus muscle?
a) Lateral meniscus
b) Posterior cruciate ligament
c) Medial meniscus
d) Patellar ligament
Explanation: The posterior cruciate ligament lies anterior to the popliteus muscle within the knee joint capsule. This anatomical relationship is important during knee surgeries to avoid injury. Correct answer: b) Posterior cruciate ligament.
Guess Question 4:
Popliteus muscle origin is:
a) Medial femoral condyle
b) Lateral femoral condyle
c) Tibial plateau
d) Fibular head
Explanation: The popliteus originates from the lateral femoral condyle and adjacent lateral meniscus. Its tendon passes inferomedially through the capsule to insert on the posterior tibia. Correct answer: b) Lateral femoral condyle.
Guess Question 5:
Which artery supplies the popliteus muscle?
a) Femoral artery
b) Popliteal artery
c) Anterior tibial artery
d) Posterior tibial artery
Explanation: The popliteal artery, a continuation of the femoral artery, gives muscular branches to the popliteus. Adequate blood supply is essential for muscle function and healing post-injury. Correct answer: b) Popliteal artery.
Guess Question 6 (Clinical):
A patient with a posterior knee stab wound presents with difficulty in initiating knee flexion. Which muscle is likely injured?
a) Gastrocnemius
b) Popliteus
c) Semitendinosus
d) Biceps femoris
Explanation: The popliteus initiates flexion from full extension by unlocking the knee. Injury here causes functional impairment despite intact hamstrings. Correct answer: b) Popliteus.
Guess Question 7:
Which of the following is NOT an action of the popliteus muscle?
a) Unlocking the knee
b) Assisting knee flexion
c) Lateral rotation of femur
d) Knee extension
Explanation: The popliteus assists in flexion and unlocking the knee but does not extend the knee. Extension is achieved mainly by the quadriceps femoris. Correct answer: d) Knee extension.
Guess Question 8 (Clinical):
Damage to which ligament may impair popliteus function due to its attachment?
a) Anterior cruciate ligament
b) Posterior cruciate ligament
c) Lateral meniscus
d) Patellar ligament
Explanation: The popliteus has a fibrous attachment to the lateral meniscus. Damage here may affect both structures, leading to locking symptoms. Correct answer: c) Lateral meniscus.
Guess Question 9:
In a weight-bearing knee, the popliteus unlocks the joint by:
a) Medial rotation of femur
b) Lateral rotation of femur
c) Lateral rotation of tibia
d) Hyperextension
Explanation: In a fixed-foot condition, the popliteus laterally rotates the femur to unlock the knee. This movement is reversed when the tibia is free. Correct answer: b) Lateral rotation of femur.
Guess Question 10 (Clinical):
A patient recovering from knee arthroscopy has pain and weakness during the first 10° of flexion. Which muscle is primarily responsible for this movement?
a) Gastrocnemius
b) Hamstrings
c) Popliteus
d) Quadriceps femoris
Explanation: The popliteus is critical for initiating knee flexion from a locked position. Weakness here manifests as difficulty in starting flexion. Correct answer: c) Popliteus.
Keywords:
Heel Touch Phase - Initial phase of gait cycle when the heel makes contact with the ground.
Calf Compartment - Posterior compartment of the leg containing gastrocnemius, soleus, and other muscles, enclosed by fascia.
Resting Pressure - The baseline pressure within a closed anatomical compartment at rest.
Compartment Pressure - Pressure within an enclosed myofascial compartment, influenced by muscle activity and blood flow.
Gait Cycle - Sequence of movements during walking, from heel strike to the next heel strike of the same foot.
Muscle Pump - The action of muscles in aiding venous return to the heart.
Q1 (Lead - NEET PG 2012): During heel touch phase of walking, pressure in calf compartment is?
a) More than resting pressure
b) Less than resting pressure
c) No change in pressure
d) First rises and then falls
Explanation: The heel touch phase activates the gastrocnemius and soleus muscles to control foot placement and initiate push-off preparation. This contraction compresses the posterior compartment, briefly elevating intracompartmental pressure above resting level. The pressure rise aids venous return. Thus, the correct answer is a) More than resting pressure. Clinically, this principle explains why dynamic muscle use prevents venous stasis.
Q2: Which phase of the gait cycle involves maximum activity of the tibialis anterior muscle?
a) Heel strike
b) Toe-off
c) Mid-stance
d) Swing phase
Explanation: The tibialis anterior is most active during heel strike and swing phase, preventing foot slap and aiding dorsiflexion clearance. However, EMG studies show peak activation at a) Heel strike to control plantar flexion. Dysfunction may cause foot drop, requiring orthotic support.
Q3: In compartment syndrome, which parameter is most important for deciding fasciotomy?
a) Duration of symptoms
b) Pressure exceeding 30 mmHg
c) Presence of pain on passive stretch
d) Sensory loss
Explanation: While clinical signs are critical, the gold standard decision criterion is b) Pressure exceeding 30 mmHg or within 30 mmHg of diastolic BP. Persistent ischemia beyond 6 hours risks irreversible muscle and nerve damage.
Q4: Which muscle group forms the superficial layer of the posterior calf compartment?
a) Soleus and gastrocnemius
b) Tibialis posterior
c) Flexor hallucis longus
d) Peroneus longus
Explanation: The superficial posterior calf compartment includes gastrocnemius, soleus, and plantaris. The correct answer is a) Soleus and gastrocnemius. These muscles share the Achilles tendon and function mainly in plantar flexion.
Q5: A 45-year-old runner develops pain and swelling in the calf after prolonged exercise. Which is the most likely cause?
a) Acute arterial occlusion
b) Chronic exertional compartment syndrome
c) Deep vein thrombosis
d) Muscle strain
Explanation: Chronic exertional compartment syndrome (CECS) causes reversible intracompartmental pressure rise during activity, leading to pain and swelling. The hallmark is resolution with rest. Thus, b) Chronic exertional compartment syndrome fits the presentation better than DVT or strain.
Q6: The pressure in calf compartment is lowest during which gait phase?
a) Heel strike
b) Toe-off
c) Swing
d) Mid-stance
Explanation: During swing phase, calf muscles relax, reducing intracompartmental pressure below resting levels. This recovery allows reperfusion of muscles. Correct answer: c) Swing.
Q7: The primary artery supplying the posterior calf compartment is?
a) Anterior tibial artery
b) Fibular artery
c) Posterior tibial artery
d) Popliteal artery
Explanation: The posterior tibial artery supplies most of the posterior compartment, branching into medial and lateral plantar arteries in the foot. Hence, answer: c) Posterior tibial artery.
Q8: Which condition results from unrelieved acute compartment syndrome?
a) Volkmann's ischemic contracture
b) Sudeck’s atrophy
c) Osgood-Schlatter disease
d) Shin splints
Explanation: Prolonged compartment syndrome leads to ischemic necrosis and fibrosis of muscles, resulting in permanent flexion deformity known as Volkmann's ischemic contracture. Timely fasciotomy prevents this outcome.
Q9: Which nerve is most at risk in posterior calf compartment syndrome?
a) Tibial nerve
b) Superficial peroneal nerve
c) Deep peroneal nerve
d) Sural nerve
Explanation: The tibial nerve passes through the posterior compartment with posterior tibial vessels, making it most susceptible to compression in elevated pressures. Hence, a) Tibial nerve is correct.
Q10: A patient with calf pain has resting compartment pressure of 12 mmHg and post-exercise pressure of 40 mmHg. Diagnosis?
a) Acute compartment syndrome
b) Chronic exertional compartment syndrome
c) DVT
d) Muscle tear
Explanation: Resting pressure <15 mmHg is normal, but post-exercise >30-35 mmHg confirms CECS. The pattern here matches b) Chronic exertional compartment syndrome.
Q11: Which maneuver increases venous return from the calf during walking?
a) Plantar flexion
b) Dorsiflexion
c) Both plantar and dorsiflexion
d) None
Explanation: The calf muscle pump works during both plantar flexion (push-off) and dorsiflexion (relaxation), propelling blood towards the heart and preventing stasis. Correct: c) Both plantar and dorsiflexion.
Chapter: Anatomy
Topic: Upper Limb
Subtopic: Muscles of Shoulder and Arm
Keyword Definitions:
Intertubercular sulcus: Groove between the greater and lesser tubercles of the humerus, lodging the tendon of the long head of biceps brachii.
Latissimus dorsi: Large back muscle inserting into the floor of intertubercular sulcus, responsible for adduction, extension, and medial rotation of the arm.
Teres major: Muscle of scapular region inserting into medial lip of intertubercular sulcus.
Pectoralis major: Large anterior chest muscle inserting into lateral lip of intertubercular sulcus.
Deltoid: Shoulder muscle inserting into deltoid tuberosity of humerus.
Medial rotation: Movement turning the anterior surface of limb towards the midline.
Adduction: Movement of limb towards the midline.
Extension (shoulder): Movement of arm backwards in sagittal plane.
Axillary nerve: Nerve supplying deltoid and teres minor muscles.
Thoracodorsal nerve: Nerve supplying latissimus dorsi.
Lead Question - 2012:
Which muscle is inserted into the floor of the intertubercular sulcus of the humerus?
a) Latissimus dorsi
b) Teres major
c) Pectoralis major
d) Deltoid
The correct answer is a) Latissimus dorsi. This large muscle arises from the lower thoracic vertebrae, lumbar fascia, iliac crest, and lower ribs. It passes superiorly and laterally to insert into the floor of the intertubercular sulcus of the humerus. Its insertion site allows it to act as a powerful adductor, extensor, and medial rotator of the arm.
1. The medial lip of the intertubercular sulcus gives insertion to:
a) Teres major
b) Latissimus dorsi
c) Pectoralis major
d) Subscapularis
Answer: Teres major. This muscle assists in medial rotation and adduction of the humerus. Its medial lip insertion is an important anatomical relationship in shoulder surgery.
2. The lateral lip of the intertubercular sulcus is the insertion site for:
a) Latissimus dorsi
b) Pectoralis major
c) Teres major
d) Deltoid
Answer: Pectoralis major. This muscle flexes, adducts, and medially rotates the humerus. Its lateral lip insertion is important in differentiating from latissimus and teres major attachments.
3. Clinical: Injury to the thoracodorsal nerve will impair:
a) Lateral rotation of arm
b) Abduction of arm
c) Adduction and extension of arm
d) Flexion of forearm
Answer: Adduction and extension of arm. The thoracodorsal nerve innervates latissimus dorsi, essential for swimming and climbing movements.
4. Which muscle originates from the iliac crest and inserts into humerus?
a) Latissimus dorsi
b) Teres major
c) Deltoid
d) Pectoralis minor
Answer: Latissimus dorsi. Its broad origin includes the iliac crest, lumbar fascia, and lower thoracic spinous processes.
5. The intertubercular sulcus lodges the tendon of:
a) Long head of triceps
b) Short head of biceps
c) Long head of biceps
d) Brachialis
Answer: Long head of biceps. This tendon passes within the sulcus, stabilized by the transverse humeral ligament.
6. Clinical: During axillary lymph node dissection, damage to which nerve can cause difficulty climbing?
a) Thoracodorsal nerve
b) Axillary nerve
c) Long thoracic nerve
d) Radial nerve
Answer: Thoracodorsal nerve. It lies near axillary lymph nodes and damage impairs latissimus dorsi function.
7. Which movement will be most affected if latissimus dorsi is paralyzed?
a) Shoulder flexion
b) Shoulder extension
c) Elbow extension
d) Wrist flexion
Answer: Shoulder extension. Latissimus dorsi extends the humerus, essential in rowing and pulling actions.
8. The nerve supply of teres major is:
a) Axillary nerve
b) Lower subscapular nerve
c) Thoracodorsal nerve
d) Radial nerve
Answer: Lower subscapular nerve. This innervates teres major, contributing to adduction and medial rotation.
9. Clinical: Weakness in using crutches may be due to paralysis of:
a) Latissimus dorsi
b) Supraspinatus
c) Infraspinatus
d) Subscapularis
Answer: Latissimus dorsi. It plays a role in lifting the body during crutch walking.
10. Which muscle is known as the "swimmer's muscle"?
a) Latissimus dorsi
b) Teres major
c) Deltoid
d) Trapezius
Answer: Latissimus dorsi. Its powerful adduction, extension, and medial rotation are essential for swimming strokes.